You are on page 1of 214

2017/

2018
MODUL PERSIAPAN UN &
SBMPTN BAHASA INGGRIS

Khurriyah Nurbaety, S.Pd

2017/ 2018
MODUL PERSIAPAN UN & SBMPTN BAHASA INGGRIS 2017/ 2018

LETTER/ E-MAIL
A. Ringkasan Materi
A letter is a written message from one party to another, containing information. Letters guarantee the
preservation of communication between both parties. They bring friends or relatives closer together, enrich
professional relationships and provide a satisfying mean of self-expression.
 Personal letter
Personal letter is a letter that is used for personal gain. The letter relates to personal affairs.
For example: the letter of a child to his parents or a letter to a friend.

The characteristics of a personal letter as follows:


(1) Not using letterhead/head letter
(2) Not using the same letter
(3) Hail vary the opening and closing letters
(4) The use of language freely, in accordance with the wishes of the writer of the letter.

 Official letter
An official letter is a letter that is used for official purposes, either written from individuals, agencies,
institutions and organizations. Examples: job application letter, invitation letter, letter of notification and
circulars. The characteristics of a formal letter, as follows:
1) Using the letterhead if the issuing institution or organization
2) Using the number of letters, attachments, and subject
3) Using the opening greeting and closing the common or official, such as: Assalamualikum, with
respect, we respectfully
4) Using the language with a variety of official or standard
5) Using the seal / stamp if it comes from an organization or official agency
6) Writing a letter to follow a particular letter format

Official letter is a letter that is used for the benefit of the work, duties of the office, or official activities.
This letter comes from the agency or institution, either private or public. Example: the letter task, letter orders,
memoranda, and the decision letter. The letter is official who berifat individual job application letter, letter of
application for license, and the leave application letter. The characteristics of official letters, as follows:
1) Using the header / head of the letter and the relevant agencies or institutions
2) Using the number of letters, attachments, and subject
3) Using the opening greeting and closing a standard or official, such as: with respect, we respectfully
4) Using a standard language or official variety
5) Using the seal / stamp of agency or office letter maker

 Application letter
Job cover letter can be handwritten or typed. Sometimes a particular company or agency specifically
requires a cover letter sent to applicants be handwritten or typed. Even if the letter will be handwritten
job applications, the text should be clear, readable, and neat. The letter, written as it will help people
who read it.
Application letter has a number of purposes:
 It allows you to sell yourself by pointing out key features of your CV
 It gives you the opportunity to include material that is not in the CV, especially personal qualities
that you listed when making your preparations.
 It shows a prospective employer that you know how to write a letter. While this may be of
decreasing importance in an electronic age, many employers still value it highly, both as a skill in
its own right and as a test of your ability to communicate clearly and effectively.

The letter of application should follow the general guidelines for all business letters. It should have an
introduction, a body, and a conclusion.
 The introduction

SMA Alfa Centauri Bandung Halaman 1


MODUL PERSIAPAN UN & SBMPTN BAHASA INGGRIS 2017/ 2018

In the introduction you should detail the job you are applying for, and, if relevant, the circumstances
that have led to this (for example an advertisement, or the recommendation of an agency).

 The body
The body of the letter provides you with an opportunity to present yourself to the employer:
- Tell the reader about your present job and why you are looking to move on.
- Explain why you are suitable for this job
- Emphasize the skills you have which make you particularly suitable for the job
- List briefly the personal qualities posed by the advert or job details

It is important not to write too much, however. Two, or at most three, short punchy paragraphs are
much more effective than two sides of rambling prose.

 The conclusion
The conclusion should round the letter off, leaving the reader with a positive image. It should sum up
briefly the selling points made in the body of the letter, mention any items (including the CV) you are
enclosing, and express willingness to provide any further information that the reader may want.

B. Contoh Soal dan Pembahasan

Dear Mr. Green,

I am writing concerning claim #760. The amount that your adjusters have set for the damage
caused by Hurricane Harry to my barn is unacceptably low. The amount that your company has offered
would not even allow doing the work myself as the materials alone would cost almost $300 more than
your estimate.
I am including the estimate from our independent contractors that I have asked to inspect the
damage to my barn, and their estimates vary between $1,500 and $1,750.
I would like your company to take into consideration the estimates I have provided and
reevaluate your settlement offer. If you continue to deny the fact that your offer is unacceptably low, I
will need to seek legal counsel.
I will appreciate your immediate attention to this matter.

Yours sincerely,

Mathew Liverpool

1. What is the cost of the materials needed to fix the damage according to Mr. Liverpool?
a. Between $1.500 and $1.750.
b. Mr. Liverpool has not decided yet.
c. Much more expensive than expected.
d. 300% less than Mr. Green's estimate.
e. $300 more than Mr. Green's estimate.

Pembahasan:
Paragraf 2 memuat informasi rinci mengenai perkiraan biaya yang diperlukan untuk memperbaiki kerusakan
menurut penuturan Mr. Liverpool (... and their estimates vary between $1,500 and $1,750).
Jawaban : A

2. It can be inferred from this letter that ....


a. Mr.Green estimated the amount payable to Mr. Liverpool
b. Mr.Green failed to send contractors to inspect the barn

SMA Alfa Centauri Bandung Halaman 2


MODUL PERSIAPAN UN & SBMPTN BAHASA INGGRIS 2017/ 2018

c. Mr. Liverpool was frustrated by the damage caused by Hurricane Harry


d. Mr. Liverpool was disappointed by the estimate made by Mr. Green's adjusters
e. Mr. Green estimated higher than what Mr. Liverpool had expected.

Pembahasan:
Paragraf pertama menjelaskan tujuan Mr. Liverpool menulis surat yaitu memberitahukan bahwa ada
kesalahan perhitungan perkiraan biaya yang dibuat oleh pegawai Mr. Green sehingga hal ini membuat Mr.
Liverpool kecewa.
Jawaban : D

C. Latihan Soal

Text 1
The following text is for questions 1 to 2

25th January, 2012


Mr. Eduard Clinton
1785 Honeycutt Boulevard
Marysville, IL 53028

Dear Mr. Clinton,

We are pleased to offer you the position of Budget Assistant at Systems, Inc. We would like you to start
working on Monday, November 10. You will report to your supervisor, Ms. Kho, in room 44. We are happy to
offer you the salary that you discussed with Ms. Shirley Smith in your interview. Other benefits include health
insurance for you and your family. We are sorry that we cannot offer life or dental insurance and
automobile insurance at this time, but we may be able to in the future. You will also get 20 vacation days
and 5 sick days per year. Stock options and a pension plan are also available. I would suggest that
you make an appointment to come in and speak with Linda Brown, our benefits expert, very soon, and she
can explain the full benefits package to you. All the same time you can fill out some paperwork for us.

Welcome to Systems, Inc. We look forward to working with you.

Sincerely,

Bob Cartwright

1. What job was offered to Mr. Eduard Clinton? 2. How many vacation days will he get?
a. Interviewer. a. 5
b. Benefit expert. b. 10
c. Budget Assistant. c. 20
d. Budget Supervisor. d. 25
e. Human Resources Director. e. 44

SMA Alfa Centauri Bandung Halaman 3


MODUL PERSIAPAN UN & SBMPTN BAHASA INGGRIS 2017/ 2018

Text 2
The following text is for questions 3 to 5

32 Alberta Way
Kingston, ON K7P OLO
Dear Dave Smith,

On behalf of your brother John, and all of the people of Alberta, I would like to personally invite you to
visit Alberta in 2010 to help mark the province‘s Centennial year.
A number of activities have been planned in the province throughout the year, and I know John would
love to have your visit to help mark this important milestone in Alberta‘s history.
To help you plan your travel, details of some of the events can be found on our website at
www.albertcicentennial.ca, and additional information can be found at Travel Alberta‘s website at
www.travelalberta.com. I am also enclosing a special gift for you, so when you do visit Alberta you can
celebrate our Centennial year for learning more about our past at one of Alberta‘s museums, historic sites or
interpretative centers.
Please come join us as we celebrate our past and imagine our future.

Sincerely yours,

Ralph Klein

3. What is the letter mainly about?


a. Important websites.
b. A travel plan to Alberta.
c. Activities during celebration.
d. An invitation to attend a celebration.
e. A celebration of Alberta‘s Centennial year.

4. Why does the sender mention the websites in his letter?


a. To see important events.
b. To browse interesting events.
c. To keep up with the current news.
d. To help the receiver enjoy his travel.
e. To facilitate the receiver to plan his trip.

5. ―Please come join us as we celebrate our past and imagine our future‖. (Paragraph 4). The sentence means
….
a. do together and play together
b. combine together and celebrate it
c. join together and do something right
d. merge together and hope to raise the product
e. let‘s commemorate the centennial year together

SMA Alfa Centauri Bandung Halaman 4


MODUL PERSIAPAN UN & SBMPTN BAHASA INGGRIS 2017/ 2018

Text 3
The following text is for questions 6 to 8

Dear Mr. Frank Sinatra,

The Fun World Co. would like to welcome you as a new customer to our firm. We know that you will be
extremely satisfied with our line of products and the service we provide to our customers.
You are invited to purchase our merchandise on our regular open account terms, from April 1 until June
30th, 2010. Our credit manager, Andi Nico, will be happy to discuss any aspect of our credit policy with you
at your convenience.
I am enclosing our catalog and price list for your review. I believe that you will find our prices
competitive and in keeping with industry trends. Throughout the year we offer our valued customers frequent
discount as an incentive and as a showing of our appreciation.
I do hope you will afford us the opportunity to serve you in the near future.

6. The letter is about.... d. merchandise and catalogs


a. a new customer welcome e. merchandise and catalogs
b. discounts offered to a new customer
c. a description about services provided 8. "I do hope you will afford us the opportunity to
d. information on prices of new products serve you in the near future." (Paragraph 4).
e. an appeal to a new customer to buy the The sentence means....
company's products a. I will send the product soon
b. I hope you enjoy having our product
7. Besides sending the letter, the company also c. I give you a chance to join the company
attached ... to its customers. d. I give you discount if you buy our product
a. catalogs and price lists e. I hope we could give you the best service
b. products and price lists soon
c. incentives and products

Text 4
The following text is for questions 9 to 10

Dear Liza,
I have received your letter asking me to meet you at your house this Sunday to discuss the urgent matter
you have. I regret however to say that it will not be possible for me to see you on Sunday as I already have a
previous appointment. I shall see you on Monday, next week, at 5 p.m. Hoping the change of date will be
convenient to you.

9. What does the letter tell about?


a. Liza‘s problem.
b. Liza‘s appointment.
c. Daisy‘s previous letter to Liza.
d. A meeting held at Liza‘s house.
e. Daisy‘s regret for not being able to come.

10. Why did Liza ask Daisy to come to her house?


a. Daisy would discuss her problem with Liza.
b. Liza persuaded Daisy to go somewhere.
c. Liza wanted daisy to solve her problem.
d. Daisy wanted to meet Liza‘s family.
e. Liza wanted to chat with Daisy.

SMA Alfa Centauri Bandung Halaman 5


MODUL PERSIAPAN UN & SBMPTN BAHASA INGGRIS 2017/ 2018

Text 5
The following text is for questions 11 to 13.

May 28, 2010

THE HUMAN RESOURCES & ADMINISTRATION HEAD


Inara Lightings, Ltd. Abu Dhabi
United Arab Emirates

Dear Sir or Madam,

I am writing to explore the possibility of employment as Document Controller in your reputable firm.

I am a Computer Science graduate of the University of the Philippines, Diliman, Quezon City.

I bring with me several years of experience as information system analyst of leading companies that include
Nissan Motor Philippines, Inc. My strong computer skills, knowledge of modern word processing software, and
project management tools are but some of the things that make me an asset to your company.

Attached is my resume for your perusal. Should you need any further information, I can be reached at 632-555-
1234 (during regular business hours), or at 632-555-6789 (during regular business hours).

Sincerely yours,

RINI ROSYIDAH
(http://educendika.blogspot.co.id/2009/12/writing-application-letter.html)

11. What position is Rini applying for? d. The applicant has not only computer skills
a. Employment. but also project management tools
b. Computer analyst. e. The applicant has a lot of work
c. Document controller. experiences since her study in the
d. Computer project manager. University of the Philippines
e. Information system analyst.
13. ―I am writing to explore the possibility of
12. These following statements are true according employment as Document Controller in your
to the text, except …. reputable firm.‖ (paragraph 1)
a. The applicant has worked for some years The underline word has similar meaning to ...
before a. Sensible
b. The applicant attaches her curriculum b. Famous
vitae in her application letter c. irresistable
c. The applicant can be contacted through d. prestigious
two different contact numbers e. disrespectable

SMA Alfa Centauri Bandung Halaman 6


MODUL PERSIAPAN UN & SBMPTN BAHASA INGGRIS 2017/ 2018

ADVERTISEMENT
A. Ringkasan materi
 Advertising is a form of communication for marketing (a picture, short film, song, etc.) which tries to
encourage, persuade, or manipulate an audience (viewers, readers, or listeners) to buy a product or
service or join in an event.
 The purpose is to inform the public or community about a certain product or service
 There are four important strategic questions in any advertisement:
a. What products will be offered?
b. Who will be the target customers?
c. How will the products reach them?
d. At what price should the products be offered?
 An advertisement must be evocative, informative and persuasive. It uses short (easy to understand) but
powerful language.
 A brochure is a flyer, pamphlet, or leaflet that is used to pass information about something. Brochures
are advertising pieces mainly used to introduce a company or organization and inform about products
and/or services to a target audience.
 Brochures are distributed by radio, handed personally or placed in brochure racks. They may be
considered as grey literature. They are usually present near tourist attractions.
 The most common types of single-sheet brochures are the bi-fold (a single sheet printed on both sides
and folded into halves) and the tri-fold (the same, but folded into thirds). A bi-fold brochure results in four
panels (two panels on each side), while a tri-fold results in six panels (three panels on each side).

B. Contoh soal dan pembahasan

Text 1

Happy Mart
Our Customers in Jakarta are welcome to enjoy
Our closing down sale on June 29,2013
50% off for all items
Please be noted that Happy Mart at Senayan Plaza Mall
Will cease its operation on June 30,2013 and
At Shangri-La Mall on August 31, 2013.
If you have any inquiries, please contact us at +622188888667677
Toll-free + 6221767679769 in West Jakarta only
Or send email to Happy Mart@ymail.com

1. If customers in West Jakarta have a question, they should contact a toll-free at ….


a. Shangri-La Mall
b. Senayan Plaza Mall
c. + 62(21)7676979769
d. +62(21)88888667677
e. Happy Mart@ymail.com

Pembahasan:
Sudah jelas tercantum di baris ke 9
Jawaban: C

SMA Alfa Centauri Bandung Halaman 7


MODUL PERSIAPAN UN & SBMPTN BAHASA INGGRIS 2017/ 2018

2. From the text we know that ….


a. Happy Mart will be sold 50% of the normal price
b. Happy Mart are selling all its items on June 28, 2013
c. Happy Mart at Senayan Plaza will close down on June 29, 2013
d. Happy Mart at Shangri-La Mall will close down before the one in Senayan Plaza
e. Happy Mart will have a big sale one day before the closing date at Senayan Plaza Mall

Pembahasan:
Dari informasi yang terdapat pada iklan diatas, kita bisa menarik kesimpulan bahwa inti dari iklan tersebut
adalah Happy Mart akan mengadakan obral besar selama satu hari yang bertempat di Senayan Plaza Mall.
Jawaban : E

Text 2

Hotel Work :
One Month Training Course
Suitable young men and women are invited to apply for places on the one month training course on
hotel work organized by Hong Kong Institute of Hotel Management. Tuition is free of charge and students who
successfully complete the course will be offered employment in the Colony‘s leading hotels.
The Training Course will take place from Monday 21st July to Friday 22nd August, from 9 a.m to 4.30
p.m. daily , except Saturdays and Sundays.
Applications for places on the course are welcome for students now in their third year at secondary
school, who have good knowledge of English, and have interest in hotel work.

Application forms may be obtained from :


The Hong Kong Institute of Management,
Box 948,
The South China Times.

The closing date for applications is April 29th

3. Those who successfully complete the course Jawaban: C


will be given ....
A. Free tuition 4. What requirement is needed by an applicant to
B. A further training course apply for the hotel work training ?
C. Jobs in big hotels in the colony A. Able to speak Mandarin.
D. A chance to stay in the colony‘s leading B. Secondary school graduate.
hotels for one month C. Good knowledge of English.
E. Membership of the Hong Kong Institute D. Reputable university graduate.
of Hotel management E. 2-year experience in hotel work.

Pembahasan: Pembahasan:
Informasi rinci mengenai pertanyaan diatas Informasi rinci mengenai persyaratan yang
tercantum pada paragraf pertama baris ketiga, dipenuhi oleh pelamar terdapat pada paragraf
yaitu (... students who successfully complete ketiga.
the course will be offered employment in the Jawaban : C
Colony‘s leading hotels). The sentence ―will be
offered employment‖ means will be offered job
in the colony.

SMA Alfa Centauri Bandung Halaman 8


MODUL PERSIAPAN UN & SBMPTN BAHASA INGGRIS 2017/ 2018

C. Latihan Soal

Text 1
The following text is for questions 1 to 2

We, a mining consultant company, are looking for …

A SENIOR SECRETARY
 Female, graduated from a reputable university
 Min. 5 years of work experience
 Good command of English (oral & written)
 Computer literate (min. Word, Excel, Internet)
Please submit your complete application, resume & recent photograph not more than 10 days after these ads
to:
HRD PT CITRA ENERGY DEVELOPMENT
Wisma Emha Jln. Wijaya I No. 11A, Kebayoran Baru, Jakarta Selatan 12170
Fax no. 021-7207978
e-mail address: mitraenergy@cbn.net.id
only short listed candidates will be notify

1. What is the text about? The following advertisement is for questions


a. Wisma Emha. 3 to 5
b. A job vacancy.
c. A senior secretary.
d. An application letter.
e. PT Citra Energy Development.

2. The following are the requirements asked


EXCEPT ….
a. five years experience.
b. able to operate a computer.
c. able to speak and write in English.
d. a graduate from a reputable university.
e. application letter should be sent ten days
after these ads.

3. How does the seller persuade the consumers?


a. By offering pizzas.
b. By giving 20% discounts.
c. By opening from 3 to 6 pm.
d. By opening every Monday to Thursday.
e. By giving 20% discount from 3 to 6 pm
every Monday to Thursday.

SMA Alfa Centauri Bandung Halaman 9


MODUL PERSIAPAN UN & SBMPTN BAHASA INGGRIS 2017/ 2018

4. What kind of pizza is offered based on the 5. ―… on our mouthwatering pizza…‖, the
advertisement? underline has the similar meaning with,
a. Indonesian pizza. EXCEPT….
b. American pizza. a. neglecting
c. Canadian pizza. b. tempting
d. Mexican pizza. c. tasty
e. Italian pizza. d. yummy
e. delicious

Text 3
The following advertisement is for questions 6 to 7

FOR SALE GREENWOOD


a. 3-bedroom family home, double garage. New condition. Large garden. Short walk to city and schools.
$74.000
b. Modern 2-storey 6-bedroom home, large living room, separate dining room, 3 bathrooms, basement, and
garage. Beautiful big garden and swimming pool. Excellent condition. $150.000.
c. Luxury 2 bedroom apartment, near ocean. Quiet location. 45 minutes from city. $55.000.
d. Attractive 3-bedroom family home, separate dining room, 2 bathrooms. Opposite bus-stop, 5 minutes from
town. Easy to look after (brick, with small garden). $80.000.
e. Available immediately. Large l-bedroom city apartment, fully furnished with quality furnishing. Large living
area, dishwasher, refrigerators $35.000
f. Attractive l-bedroom apartment in new building. Only 30 minutes from city centre and 10'minutes from
airport. Unfurnished. $29.000

6. The topic of the text is about .... 7. A home which has a swimming pool is priced at
a. Greenwood ....
b. types of homes a. $29.000
c. residences for sale b. $55.000
d. prices of the apartments c. $74.000
e. location of the residences d. $80.000
e. $150.000

SMA Alfa Centauri Bandung Halaman 10


MODUL PERSIAPAN UN & SBMPTN BAHASA INGGRIS 2017/ 2018

The following advertisement is for questions 8 to 10

8. What is the advertisement about?


a. Air Asia gives 200,000 free seats to Philippines.
b. Air Asia performs their first flight to Philippines.
c. Air Asia informs the flight schedule to Philippines.
d. Air Asia holds an event in Philippines.
e. Air Asia flies to Philippines with love.

9. When did the free seats booking end?


a. 1 April 2014.
b. 30 April 2014.
c. 1 December 2013.
d. 30 December 2013.
e. 24 November 2013.

10. Based on the advertisement above, these are the destinations if you fly from Cebu, EXCEPT….
a. Cagayan de Oro
b. Puerto Princesa
c. Kuala Lumpur
d. Davao
e. Seoul

SMA Alfa Centauri Bandung Halaman 11


MODUL PERSIAPAN UN & SBMPTN BAHASA INGGRIS 2017/ 2018

ANNOUNCEMENT
A. Ringkasan Materi

Announcement berasal dari kata announce; dalam bahasa Inggris kata kerja announce bermakna
"make a formal public declaration about a fact, occurrence, or intention. Jadi bisa dikatakan announcement
adalah sebuah teks yang menerangkan sebuah pemberitahuan tentang sebuah acara yang akan
diselenggarakan dan lebih bersifat umum.
An announcement is divided into written announcement and spoken announcement. We can find a
written announcement on the wall magazine or on the public place, and also we can find a spoken
announcement when teacher call chief of the students or when there is an event in the school.

Generic structures of announcement are:


1. Head (the title or type of event)
2. Body or content (date, place, program, etc)
3. Closing (For more information,….)
4. Writer (someone or an institution who make the announcement)

B. Contoh Soal dan Pembahasan

ANNOUNCEMENT
In accordance with the international security regulations, the following items are never allowed to be
taken onto a plane by passengers, either in their carry-on bags or in their checked luggage: weapons,
including knives and guns; explosives, including dynamite and fireworks.
The following items may be placed in checked luggage but not on carry-on bags. Tools, including
hammers, screwdrivers and wrenches; sports equipment such as golf clubs, baseball bats, skis and ski
poles.
When you pass through the security line, all bags will go through our X ray machines and some bags
will be manually checked by personnel, as well.
Thank you for your cooperation. Have a safe and pleasant flight.

1. What is the announcement about?


a. Weapons and explosives.
b. Welcoming foreign passengers.
c. Items brought on carry-on bags.
d. An international airport security regulation.
e. A warning to passengers from different countries.

Pembahasan:
Dari paragraph pertama jelas disebutkan bahwa pengumuman diatas mengenai an international airport
security regulation.
Jawaban: D

SMA Alfa Centauri Bandung Halaman 12


MODUL PERSIAPAN UN & SBMPTN BAHASA INGGRIS 2017/ 2018

C. Latihan Soal

Text 1
The following text is for questions 1 and 2.

ANNOUNCEMENT
Pay attention !
Our school will have a debate competition
 Participants : All students in our school
 It will be held from 10 – 12 August 2008
 Prize : I Rp 2.000.000,-
II Rp 1.500.000,-
III Rp 1.000.000,-

Please join us!

1. What is the announcement about? 2. According to the text the competition ….


a. A school debate a. is only for students with good English
b. A school competition b. is in the form of spoken arguments
c. A debate competition c. is in the form of written arguments
d. Extracurricular activities d. will be held after school hours
e. Participants of the debate e. will run for two days

Text 2
The following text is for questions 3 to 4

To All Third Grade Students


All library books have to be returned on the twenty-second of May, 2009. They should be covered with non-
colorful wrapping plastic. Lost books must be replaced with the ones of similar subjects. Fine will be charged
to the late return of the books. Students who have handed on all books will get receipts that have to be
submitted to the administration officers.

Library Staff

3. What is informed on the announcement? 4. What will happen if the book you borrowed is
a. A request to return library books. lost?
b. The subjects of the library books. a. You will get receipts.
c. A compulsory to replace lost books. b. It should be covered by non-colorful
d. The library administration officers. plastic.
e. Submission of receipts. c. The librarian will be fined by a certain
amount of money.
d. You should cover with wrapping plastic.
e. It should be replaced with a similar book.

SMA Alfa Centauri Bandung Halaman 13


MODUL PERSIAPAN UN & SBMPTN BAHASA INGGRIS 2017/ 2018

Text 2
The following text is for questions 5 to 7.

Hi guys! Our neighborhood will hold a Fund Raising Activity next month. It is intended to help victims of the
landslide which happened yesterday. Everyone who is interested in this activity, please join us. You are also
invited to donate your used clothes. If you have any questions, please feel free to contact Ms. Maryam as
chief program coordinator at her house from 7 a.m to 3 p.m. from Monday to Friday.

Committee

1. What is the announcement about?


a. A neighborhood‘s activity.
b. Ms. Maryam‘s program.
c. Used clothes donation.
d. Victims of a landslide.
e. Fund raising activity.

2. Where can Ms. Maryam accept her neighbors‘ questions?


a. At her office.
b. At her house.
c. At the landslide area.
d. At the neighborhood.
e. At the committee office.

3. If you have any question, please feel free to contact Ms. Maryam.
This sentence has the same meaning as ….
a. Ms. Maryam is always ready to give information you need
b. Ms. Maryam is always ready to ask your questions
c. You are not charged if you contact Ms. Maryam
d. Ms. Maryam will give you questions freely
e. Ms. Maryam has questions to ask

SMA Alfa Centauri Bandung Halaman 14


MODUL PERSIAPAN UN & SBMPTN BAHASA INGGRIS 2017/ 2018

RECOUNT TEXT
A. Ringkasan Materi

Recount text is a text that telling the reader about one story, action or activity. Its goal is to entertaining or
informing the reader.

Generic structure:
 Orientation: provides the setting and introduces participants
 Events: tells what happened, in what sequences
 Re-orientation: optional-closure of events

Significant Lexicogrammatical Features:


 Focus on specific participants
 Use of material processes
 Circumstances of time and place
 Use of past tense
 Focus on temporal sequences

B. Contoh Soal dan Pembahasan

I usually woke up at 8 a.m. and went to the press center to check the daily schedule of briefings and
press conferences, usually held by the United Nations officials or disaster mitigation team chief, Alwi
Shihab.
It was challenging to visit different refugee to find soft stories, human interest stories. Then, I went
back to the press center in between to cover the press conferences of the day.
It was heart breaking when I saw these survivors fight for food and secondhand clothing, which they
said were limited and inadequate. Emerging to a glaring, full noon, it was time to go back to the press
center to write stories and race against time, always fearing that the internet connection would come
crashing down.
And after everything was done, only then I remembered to eat. Most times, I only ate once a day
because you always had to rush and it was difficult to find food. You had to travel quite far, about a 30-to
45- minute trip by car to find fresh food.

1. How often did the writer eat a day? e. writer


a. Three times. Jawaban: D
b. Seldom.
c. Twice. 3. What is the writer‘s occupation?
d. Never. a. A volunteer.
e. Once. b. A journalist.
c. A presenter.
Pembahasan: d. A TV reporter.
Jawaban terdapat pada paragraf keempat baris e. A social worker.
pertama, ―… I only ate once a day …‖
Jawaban: E Pembahasan:
Pada paragraph ketiga baris ketiga terdapat
kalimat “… it was time to go back to the press
2. ―… mitigation team chief Alwi Shihab.‖ center to write stories and race against time,
(Paragraph 1) …”. Dari kalimat tersebut dapat kita simpulkan
The synonym of the underlined word is …. bahwa pekerjaan penulis adalah seorang
a. developer jurnalis.
b. decreaser Jawaban: B
c. lightener
d. reliever

SMA Alfa Centauri Bandung Halaman 15


MODUL PERSIAPAN UN & SBMPTN BAHASA INGGRIS 2017/ 2018

C. Latihan Soal

Text 1
The following text is for questions 1 to 3
One day in July 2005, I went fishing on Sodus bay. I casted under our own dock with a rubber worm
and BOOM!

A big bass took off to the other side of the dock and I was sure the line was going to break because
his pulling was rubbing the line against sharp rocks. Three times I pulled him back to my side of the dock and
three times he got back to the other side of the dock. Finally, I got him to my side and he was tired.

I knew that this was the biggest bass I had ever seen. I worked him towards me and got down on my
belly, reached down and gripped him. However, I didn't have a good grip at first and I let him down to the water
for a few seconds to get a better grip. The next thing I knew he was swimming away. No! I screamed louder
than I ever had. It was the first time I cried over a fish. What a huge fish. This is a true story and a sad one for
me.

1. Where did the writer cast his line? d. the bass made the writer tired
a. Under his own dock. e. a big bass broke the writer‘s line.
b. On his own dock.
c. At his cottage. 3. What happened after the writer could grip the
d. On a bay. big bass?
e. At home. a. He stored the big bass in his dock.
b. He cleaned the big bass up.
2. The main idea of paragraph 2 is.... c. He laid down the big bass.
a. the writer got a big bass d. He picked the big bass up.
b. the writer pulled the bass e. He lost the big bass.
c. a big bass took off to the sea

Text 2
The following text is for questions 4 to 6.

I, Milton Friedman was born on July 31, 1921, in Brooklyn, N.Y., the fourth and last child and first son
of Sarah Ethel (Landau) and Jeno Saul Friedman. My parents were born in Carpatho-Ruthenia of the
Soviet Union. They emigrated to the U.S. in their teens, meeting in New York. When I was a year old,
my parents moved to Rahway, N.J,, a small town about 20 miles from New York City.
I was awarded a competitive scholarship to Rutgers University. I graduated from Rutgers in 1932.
I financed the rest of my college expenses by the usual mixture of waiting at tables, clerking in a retail
store, occasional entrepreneurial ventures, and summer earnings. Shortly, however, I became interested
in economics.
In economics, I had the good fortune to be exposed to two remarkable men: Arthur F. Burns
and Homer Jones. Arthur Burns shaped my understanding of economic research, introduced me to the
highest scientific standards, and became a guiding influence on my subsequent career. Homer Jones
introduced me to rigorous economic theory, made economics exciting and relevant, and encouraged me
to go on to graduate work. On his recommendation, the Chicago Economics Department offered me a
tuition scholarship, As it happened, I was also offered a scholarship by Brown University in Applied
Mathematics, but, by that time, I had definitely transferred my primary allegiance to economics. In 1971
won the Nobel Memorial Prize in Economics for my achievements in the fields of consumption analysis,
monetary history and theory and for his demonstration of the complexity of stabilization policy, ln 1977, at
age 65, I retired from the University of Chicago after teaching there for 30 years.

SMA Alfa Centauri Bandung Halaman 16


MODUL PERSIAPAN UN & SBMPTN BAHASA INGGRIS 2017/ 2018

4. Who influenced Milton Friedman on d. Brown University offered Milton


economic research? Friedman a tuition scholarship.
a. Sarah Ethel. e. Milton Friedman was offered a
b. Jeno Friedman. scholarship by the Chicago
c. Arthur F. Burns. Economics Department in Applied
d. Flomer.Tones. Mathematics'
e. Alfied Nobel.
6. What did Milton Friedman do in financing
5. What is the main idea of paragraph 3? his college expenses?
a. Milton Friedman was very committed a. He was a teacher during summer.
in his interest and talent in b. He had a job in entrepreneur
economics. company
b. Homer Jones shaped Milton Friedman c. He worked as a clerk in a retail
understanding of economic research. store.
c. Arthur Burns introduced Milton d. He sold tables to people.
Friedman to rigorous economic theory. e. He taught economics in the
university.

Text 3
The following text is for questions 7 to 9.

Ingrid Schuster, Austria‘s top woman skier, won the World Cup downhill race here today – her
first race since her terrible fall at wengen nine months ago where she broke both her legs in a 90 kph
crash. Doctors said she would never ski again, but 22-year- old Ingrid didn‘t believe them. ―I was sure I would ski
again. I just knew it,‘‘ she told me after she had won the race. Getting fit again was very hard work and it took a
long time. ―But I knew that one day I would race again. I wanted my first race to be here at Kizbuhel, in front of all
my fans, but I didn‘t really expect to win. I just wanted to take part. Winning today was great. I made all the hard
work worthwhile.‖

7. What‘s the main idea of the text? b. Ingrid Schuster had faith that she
a. Ingrid won her ski race competition. could ski again
b. Ingrid took part in the World Cup c. The accident paralyzed her both legs
downhill race. d. She gave up skiing after the accident
c. The world Cup downhill race was her e. It didn‘t take a long time for her to
first race. recover
d. Ingrid won the World Cup downhill race
after nine months of absence. 9. In the passage, what is used to show the real
e. Ingrid is an Australian top woman skier. words of the speakers?
a. Passive voice
8. Which statement is true according to the text? b. Direct speech
a. The doctors said she could ski again c. Reported speech
after the accident d. Simple present tense
e. Simple past tense

Text 4
The following text is for questions 10 to 13.

I went on a nice picnic on Saturday with some friends from school. We went to lovely lake
about a hundred miles north of here. We left on the bus at seven o‘clock in the morning and arrived at the
lake at eight- thirty. We spent the morning swimming and rowing boats on the lake. After lunch we
climbed a large hill behind the lake and saw the beautiful view from the top. We got back to the bus around
four o‘clock and then we came back a town.

SMA Alfa Centauri Bandung Halaman 17


MODUL PERSIAPAN UN & SBMPTN BAHASA INGGRIS 2017/ 2018

10. What does the text tell us about? c. Seeing the beautiful seashore.
a. The writer‘s activities on Saturday. d. Going on a picnic alone.
b. The writer‘s planning to go to the lake. e. Enjoying his holiday.
c. The writer‘s satisfaction with his holiday.
d. The writer‘s preparation for spending 12. The following are about the lake, except ….
holiday. a. it was very danger
e. The writer‘s experience when he went b. it has a beautiful scenery
on a picnic. c. there is a hill behind the lake
d. there is a small restaurant near by
11. Which is true about the writer? e. it was a hundred miles far from the
a. Spending his time by fishing. city
b. Building a tent near the lake.

Text 5
The following text is for questions 13 to 16.

When the earthquake happened, Tom was on his car. He was driving home from his work. Suddenly
his car lurched to one side to the left. He thought he got a flat tire. He didn‘t know that it was an earthquake. He
knew that it was an earthquake when he saw some telegraph poles falling to the ground, like match
sticks. Then he saw lot of rocks tumbling across the road. He was trapped by the rocks. He couldn‘t move his
car at all. There were rocks everywhere. There was nothing he could do but left his car and walked a long way to
his house, in town.
When he got there, he was very surprised that there was almost nothing left. The earthquake
made a lot of damage to his town.

13. What does the text tell us about?


a. Earthquake. 15. When he realized it, he ….
b. Tom‘s experience. a. was very surprised
c. Car‘s accident. b. left his car
d. How to escape from danger. c. stayed inside his car
e. Trapped on the road. d. kept watching the disaster
e. tried hard to make his car move
14. How did he know that it was an earthquake?
a. When he saw a lot of rocks tumbling 16. Then he saw lot of rocks tumbling across the
across the road road. The underlined word is similar to ….
b. When he saw some telegraph poles a. moving inside
falling to the ground b. lying
c. Because he couldn‘t move his car at c. falling
all d. jumping
d. Because there were rocks everywhere e. rolling
e. Because his car lurched to one side to
the left

SMA Alfa Centauri Bandung Halaman 18


MODUL PERSIAPAN UN & SBMPTN BAHASA INGGRIS 2017/ 2018

Text 6
The following text is for questions 17 to 19.

Alfred Bernhard Nobel was a Swedish chemist, engineer, innovator, and ornament manufacturer. He
was the inventor of dynamite. He also owned Bofors, which he had redirected from its previous role as
primarily an iron and steel producer to a major manufacturer of cannons and other ornaments. He held
355 different patents, dynamite being the most famous. In his last will, he used his enormous fortune to institute
the Nobel Prizes. The synthetic element nobelium was named after him. He was the third son of Immanuel
Nobel and Andriette Ahlsell Nobel. Born in Stockholm on 21 October 1833, lie went with my family to Saint
Petersburg in 1842, where his father invented modern plywood. He studied chemistry with Professor Nikolay
Nikolaevich Zinin. When he was 18, he went to the United States to study chemistry for four years and
worked for a shore period under John Ericsson, who designed the American Civil War ironclad USS
Monitor.
Returning to Sweden with his father after the bankruptcy of his family business, he then devoted
himself to the study of explosives, and especially to the save manufacture and use of nitroglycerine
(discovered in 1847 by Ascanio Sobrero, one of his fellow students under Theophile-Jules Pelouze at the
University of Turin). A big explosion occurred on 2 September 1864 at his factory in Heleneborg in Stockholm,
killing five people. Among them was his younger brother, Emil. The foundations of the Nobel Prize were laid in
1895 when Alfred Nobel wrote his last will, leaving much of his wealth for its establishment. Since 1901,
the prize has honored men and women for outstanding achievements in physics, chemistry, medicine
literature and for work in peace.

17. Where did Alfred Nobel work with John c. A big explosion in Helenborg in Stockholm
Ericsson? killed many people.
a. In Sweden d. A big explosion destroyed his factory in
b. In the USA Heleneborg Stockholm.
c. In Stockholm e. Alfred Nobel planned the safe manufacture
d. In Heleneborg and use of nitroglycerine.
e. In Saint Petersburg
19. What was Bofors's main business under Alfred
18. What is the main idea of paragraph 2? Nobel?
a. Alfred Nobel devoted himself to the study a. It manufactured cannons and other
of chemistry. ornaments.
b. His success with explosive finally led to b. It designed the ironclad monitor.
the 1864 tragedy. c. It produced iron and steel.
d. It produced dynamite.
e. It invented nobelium.

SMA Alfa Centauri Bandung Halaman 19


MODUL PERSIAPAN UN & SBMPTN BAHASA INGGRIS 2017/ 2018

NARRATIVE TEXT
A. Ringkasan Materi

Jika melihat pada kamus bahasa Inggris, secara harfiah narrative bermakna (1) a spoken or written account
of connected events; a story. (2) the narrated part of a literary work, as distinct from dialogue. (3) the
practice or art of narration. Jika disimpulkan, maka sebuah narrative text adalah teks yang berisi sebuah
cerita baik tertulis ataupun tidak tertulis dan terdapat rangkaian peristiwa yang saling terhubung.

The purpose of the narrative text is to amuse or to entertain the reader with the story.
The generic structures of the narrative text are:
1. Orientation:
It is about the opening paragraph where the characters of the story are introduced.
2. Complication:
Where the problems in the story developed.
3. Resolution:
Where the problems in the story is solved.
4. Re-orientation:
Conclusion of the ending of the story

B. Contoh Soal dan Pembahasan

The following text is for questions 1 to 3.

Once upon a time there lived as a neighbor, a bear and a rabbit. The rabbit was a good shot, and the
bear, being very clumsy, could not use the arrow to good advantage. The bear would call over the rabbit and
asked the rabbit to take his bow and arrows and came with the bear to the other side of the hill. The rabbit,
fearing to arouse the bear‘s anger by refusing, consented and went with the bear and shot enough buffalo to
satisfy the hungry family. Indeed, he shot and killed so many that there was lots of meat left after the bear and
his family had loaded themselves, and packed all they could carry home. The bear was very gluttonous and did
not want the rabbit to get any of the meat, so the poor rabbit could not even taste the blood from the butchering,
as the bear would throw earth on the blood and dry it up. Poor rabbit would have to go home hungry after his
hard day‘s work.
The bear was the father of five children. The youngest child was very kind to the rabbit. The mother
bear, knowing that her youngest child was very eater, always gave him an extra large piece of meat, but the
youngest child didn‘t eat. He would take it outside with him and pretend to play ball with it, kicking it toward the
rabbit‘s house, and when he got close to the door he would give the meat with such a great kick, that it would fly
into the rabbit‘s house and in this way the poor rabbit would get his unknown to the papa bear.

1. The poor rabbit didn‘t taste any of the meat because ….


a. they are already given to the butcher
b. they are eaten by the youngest bear
c. the bear carried all the meat home
d. they are already dried up
e. the bear ate all the meat

Pembahasan:
Pada paragraph 1 terdapat kalimat ―… had loaded themselves, and packed all they could carry home. The bear
was very gluttonous and did not want the rabbit to get any of the meat, so the poor rabbit could not even taste
the blood from the butchering,…‖ Dari kalimat tersebut bisa kita simpulkan bahwa si kelinci malang tidak bisa
merasakan daging karena si beruang sudah membawa semua daging ke rumahnya.
Jawaban: C

SMA Alfa Centauri Bandung Halaman 20


MODUL PERSIAPAN UN & SBMPTN BAHASA INGGRIS 2017/ 2018

2. Which statement is NOT TRUE according to the text?


a. The papa bear was not very kind to the rabbit.
b. The rabbit got nothing from his shooting.
c. The mother bear always gives her youngest extra meat.
d. The papa bear didn‘t like giving the rabbit some meat.
e. The papa bear knew that his youngest child gave the rabbit some meat.

Pembahasan:
Dari semua pilihan jawaban yang tidak sesuai adalah pernyataan E karena pernyataan tersebut bertentangan
dengan pernyataan yang terdapat pada paragraph 2 kalimat ―… the poor rabbit would get his unknown to the
papa bear.‖ Kalimat tersebut menyatakan bahwa papa beruang tidak mengetahui bahwa anaknya memberikan
daging kepada si kelinci.
Jawaban: E

The following text is for questions 3 to 5.

The Man and the Monster


Once upon a time there lived a young man, named Nazar. He lived a very poor life. One day, Nazar
went to town to look for a job. He had to climb up the hill and (3) … a thick jungle to get to a nearest town.
Suddenly, he heard (4) … from a bush. When he approached, he found some people gathering a big tree. Nazar
realized that the people were doing against God order. Then he took out a big (5) … axe and planned to cut
down the tree.

3. …. 5. ….
A. checked A. sharp D. rough
B. examined B. edge E. thick
C. inspected C. dull
D. went through
E. hunted through Pembahasan:
Pada kalimat ini terdapat noun phrase ― a big
Pembahasan: … axe‖ sehingga kita butuh adjective untuk
Semua pilihan jawaban adalah kata kerja bentuk lebih mendeskripsikan kata bendanya (axe).
lampau (verb 2). Kalimat pada nomor ini sebetulnya Adjective yang tepat untuk mendeskripsikan
terdiri dari dua klausa yang dihubungkan oleh kata ‗axe‘ adalah sharp.
connector ‗and‘, yaitu: Jawaban: A
He had to climb up the hill
He … a thick jungle to get to a nearest town. (Objek 6. Arrange the following jumbled sentences into
pada kalimat ini adalah a thick jungle sehingga bisa a meaningful paragraph!
kita simpulkan kata kerja yang tepat adalah went
through) 1. Finally, one of the frogs listened to what
Jawaban: D the other frogs were saying and gave up.
He fell down and died.
4. …. 2. When the other frogs saw how deep the pit
A. trembles D. gestures was, they told the two frogs that they were
B. shadows E. sounds as good as dead. The two frogs ignored
C. movement the comments and tried to jump up out of
the pit with all their might.
Pembahasan: 3. The other frogs continued to jump as hard
Kata kerja pada kalimat ini adalah ‗heard‘ sehingga as he could. Once again the crowd of frogs
objek yang tepat adalah sounds. yelled at him to stop the pain and just die.
Jawaban: E 4. The other frogs kept telling them to stop,
that they were as good as dead.
5. He jumped even harder and finally made it
out. When he went out, the other frogs

SMA Alfa Centauri Bandung Halaman 21


MODUL PERSIAPAN UN & SBMPTN BAHASA INGGRIS 2017/ 2018

said, ―Didn‘t you hear us; the frogs The good arrangement is ….
explained to them that he was deaf. He a. 6 – 2 – 4 – 3 – 1 – 5
thought they were encouraging him the b. 6 – 2 – 4 – 1 – 5 – 3
entire time. c. 6 – 2 – 4 – 1 – 3 – 5
6. A group of frogs were travelling through d. 6 – 2 – 1 – 4 – 3 – 5
the woods, and some of them fell into a e. 6 – 2 – 1 – 3 – 4 – 5
deep pit. Jawaban: A

C. Latihan Soal

Text 1
The following text is for questions 1 to 3.

Once upon a time rabbit wanted to cross a river but he could not swim. He had an idea. He saw a boss
of crocodiles swimming in the river. The rabbit asked the boss of crocodile. ―How many crocodile are there in the
river?‖ ―Where are they?‖ the rabbit asked for the second time.
―All of you are good, nice, gentle and kind, so I want to make a line in order. Later, I will know how kind
you are‖, said the rabbit. Then, the boss of the crocodile called all his friends and asked them to make a line in
order from one side to the other side of the river. Just then, the rabbit started to count while jumping from one
crocodile to another one. One … two … there … four … until twenty, and finally, he thanked all crocodile
because he had crossed the river.

1. The story mainly tells us about …. 2. They explored places and they got a place
a. Twenty crocodiles called sima-sima lanungun meaning quiet
b. The boss of the crocodiles place.
c. A rabbit and twenty crocodiles 3. Now the place has become simalungun.
d. A rabbit and the boss of crocodile 4. Long long ago, there were 3 kingdoms in
e. The boss of the crocodile and all his North Sumatra, namely Silau Kongdom,
friends Tanah Djawo Kingdom, and Raya
Kingdom. They lived prosperously and
2. We know from the first paragraph that the peacefully.
rabbit actually wanted …. 5. It made other kingdoms so jealous that
a. to cross the river they planned to conquer them.
b. to swim across the river 6. They helped each other in sweeping the
c. to meet the boss of crocodile enemy. The country had welfare and the
d. to know where the crocodiles are citizens were happy.
e. to know the number of crocodiles there 7. They arranged a strategy to defeat them.
Majapahit Kingdom attacked Tanah Djawo
3. ―All of you are good, nice, gentle and kind ….‖ Kingdom, but the other kingdoms helped
(Paragraph 2) Tanah Djawo Kingdom so Majapahit failed
The underlined word is synonymous with …. too.
a. wild 8. They forced the three kingdoms into
b. diligent leaving their homeland.
c. cheerful
d. easy going A. 4-6-5-7-1-8-2-3
e. honorable B. 4-5-6-7-1-8-2-3
C. 4-6-5-1-7-8-2-3
4. Arrange the following sentences into a D. 4-6-5-7-1-2-8-3
good paragraph. E. 4-6-5-7-1-8-3-2
1. Soon after they forced Majapahit to retreat,
another enemy appeared and attacked
them.

SMA Alfa Centauri Bandung Halaman 22


MODUL PERSIAPAN UN & SBMPTN BAHASA INGGRIS 2017/ 2018

The following text is for questions 5 to 7.

One Sunday morning, a proud fly flew around to find someone to talk to. Seeing a bull grazing he
approached the bull to talk.
The small fly flew down and droned around the bull‘s head, but the bull did not care about him. He kept
munching grass. Then fly, then, (5) … right inside the bull‘s ear, but the bull did not give any different reaction.
Seeing the bull‘s reaction, the fly concluded that the bull was stupid.
The fly decided to land on the bull‘s nose, still the bull ignored him. Then, the fly shouted, ―What a dumb
(6) … you are! If am too heavy for you, just spill it out and I‘ll go!‖
The bull smiled and calmly responded, ―O, tiny fly! Whether you stay or go, it doesn‘t matter. I won‘t be
(7) … with your presence as long as you keep your mouth shut up!‖

5. ….
a. murmured
b. whispered
c. whizzed
d. purred
e. sang

6. ….
a. soul
b. thing
c. ghost
d. insect
e. creature

7. ….
a. interrupted
b. interfered
c. bothered
d. annoyed
e. riled

SMA Alfa Centauri Bandung Halaman 23


MODUL PERSIAPAN UN & SBMPTN BAHASA INGGRIS 2017/ 2018

SPOOF TEXT
A. Ringkasan Materi
Definition:
Spoof text is a text which tells the events of the past with an unexpected ending and funny.

Purpose:
To entertain the readers with funny story.

Generic Structures:
1. Orientation: Introduction of the participant of the story, where it happens and when it happens. By giving
the orientation, reader will recognize the story.
2. Events: Tells about the chronological of the story. Several events are explored in chronological way which
able to arrange the story read nicely.
3. Twist: An unpredictable plot in the end of the story which amuse the reader. Readers even didn‘t predict
before that it would be.

Language Features:
1. Focusing on individual participant like as people, animals or certain things
2. Using past sentence
3. Using action verb (ate, ran, was walking, etc.)
4. The use of adverbial phrase of time and place (in the garden, two days ago)
5. Told in chronological order
6. Use of direct speech or indirect speech for the dialogues
7. The use of connective (first, then, finally)

B. Contoh Soal dan Pembahasan

The Blonde and the Lawyer

A blonde and a lawyer were seated next to each other on a flight from LA to NY.
The lawyer asked if she would like to play a fun game. The blonde, tired, just wanted to take a nap, politely
declined and rolled over to the window to catch a few winks.
The lawyer persisted and explained that the game was easy and a lot of fun.
He explained, "I ask you a question, and if you don't know the answer, you pay me $5.00, and vise versa ."
Again, she declined and tried to get some sleep. The lawyer, now agitated, said, "Okay, if you don't
know the answer you pay me $5.00, and if I don't know the answer, I will pay you $500.00."
This catched the blonde's attention and, figuring there would be no end to this torment unless she
played, agreed to the game. The lawyer asked the first question. "What's the distance from the earth to the
moon?" The blonde didn't say a word, reached into her purse, pulled out a $5.00 bill and handed it to the lawyer.
"Okay" said the lawyer, "your turn."
She asked the lawyer, "What goes up a hill with three legs and comes down with four legs?" The
lawyer, puzzled, took out his laptop computer and searched all his references, no answer. He taped into the air
phone with his modem and searched the net and the library of congress, no answer. Frustrated, he sent e-mails
to all his friends and coworkers, to no avail. After an hour, he woke the blonde up, and handed her $500.00. The
blonde said, "Thank you", and turned back to get some more sleep.
The lawyer, who is more than a little miffed, woke the blonde up and asked, "Well, what's the answer?"
Without a word, the blonde reached into her purse, handed the lawyer $5.00, and went back to sleep.

1. The story tells us about …. e. a blonde who lost her money


a. a plane which a blonde and a lawyer took
b. a lawyer who played a game with a blonde Pembahasan:
c. a lawyer who bet on something with a Dari judul teks diatas kita bisa mengetahui
blonde bahwa cerita tersebut mengenai a lawyer and a
d. a blonde who was tricked by a lawyer

SMA Alfa Centauri Bandung Halaman 24


MODUL PERSIAPAN UN & SBMPTN BAHASA INGGRIS 2017/ 2018

blonde. The lawyer asked the blonde to play blonde akhirnya mau bermain game dengan
the game with him. menyetujui peraturan yang diajukan oleh the
Jawaban: B lawyer, yaitu jika the blonde tidak bisa
menjawab pertanyaan, maka dia harus
2. What did the lawyer want to play with the membayar $5.00. Namun, jika the lawyer tidak
blonde? bisa menjawab pertanyaan, maka dia harus
a. Hide and seek. membayar $500.00.
b. Chicken limbo. Jawaban: B
c. Jump rope.
d. Guessing game. 4. How much money did the blonde benefit from
e. Scrabble. the game?
a. $5.00
Pembahasan: b. $50.00
Dari kalimat "I ask you a question, and if you c. $500.00
don't know the answer, you pay me $5.00, and d. $510.00
vise versa”, kita bisa mengetahui bahwa e. $490.00
permainan yang ditawarkan the lawyer adalah
tebak-tebakan (guessing game). Pembahasan:
Jawaban: D The blonde paid $5.00 for not being able to
answer the questions. The lawyer paid $500.00
3. What was the rule of the game finally? for not being able to answer the questions. And
a. If the blonde could not answer the lawyer‘s when the lawyer asked for the answer, the
question, she paid him $5.00, and vice blonde herself didn‘t know it. Therefore, she
versa. paid $5.00. It means the blonde got $490.00 as
b. If the blonde could not answer the the benefit from the game.
question, she should pay $5.00. If the Jawaban: E
lawyer could not answer her question, he 5. Who was cleverer, the blonde or the lawyer?
should pay $500.00. a. The blonde.
c. If the blonde could not answer the lawyer‘s b. The lawyer.
question, she paid him $500.00, and vice c. None of them was clever.
versa. d. Both were clever.
d. If the blonde could not answer the lawyer‘s e. They were the same.
question, she paid him $500.00. If the
lawyer could not answer her question, he Jawaban: A
paid her $5.00.
e. If the lawyer could not answer her 6. The word ―miffed‘ in the last paragraph has the
question, he paid her $500.00. If the similar meaning to…
blonde could not answer his question, she a. shy
did not have to pay him. b. happy
c. generous
Pembahasan: d. doubt
Perhatikan pernyataan, "Okay, if you don't e. angry
know the answer you pay me $5.00, and if I
don't know the answer, I will pay you $500.00." Pembahasan:
This catches the blonde's attention and, Kata ‗miffed‘ yang bermakna jengkel memiliki
figuring there will be no end to this torment kesamaan makna dengan kata ‗angry‘.
unless she plays, agrees to the game.‖ Jawaban: E
Pernyataan tersebut menyatakan bahwa the

SMA Alfa Centauri Bandung Halaman 25


MODUL PERSIAPAN UN & SBMPTN BAHASA INGGRIS 2017/ 2018

C. Latihan Soal

The following text is for questions 1 to 3.

Last week I took my five-year old son, Willy, to a musical instrument store in my hometown. I
wanted to buy him a set of junior drum because his drum teacher advised me to buy him one. Willy likes
listening to music very much. He also likes asking me everything he wants to know. Even his questions
sometimes seem precocious for a boy of his age. He is very inquisitive.
We went there by car. On the way, we saw a policeman standing near a traffic light regulating the
passing cars and other vehicles. He blew his whistle now and then.
Seeing the policeman blowing his whistle, Willy asked me at once, "Dad, why is the policeman
using a whistle, not a drum?"
Hearing his unexpected question I answered reluctantly, "Because he is not Phil Collins!"

1. What does the text talk about ….


a. Willy and his new drum
b. Phil Collins and his drum
c. A policeman and his whistle
d. Willy's drum private teacher
e. The writer's five-year old son

2. From the text above we know that Willy is a/n … boy.


a. smart
b. funny
c. stupid
d. childish
e. annoying

3. Which sentence makes the text a funny story?


a. He is very inquisitive.
b. "Because he is not Phil Collins".
c. He blew his whistle now and then.
d. He also likes asking me everything he wants to know.
e. "Dad, why is the policeman using a whistle not a drum?"

SMA Alfa Centauri Bandung Halaman 26


MODUL PERSIAPAN UN & SBMPTN BAHASA INGGRIS 2017/ 2018

ANECDOTE TEXT
A. Ringkasan Materi

The roots of anecdote lie in the Greek word anekdota, meaning ―unpublished.‖ The word‘s original sense in
English was ―secret or private stories‖. But the most common sense today is that of a funny story about
something that happened.‖
Anecdote is a text which retells funny and unusual incidents in fact or imagination. Its purpose is to
entertain the readers.

Generic Structure of Anecdote


1. Abstract
Pada bagian ini, penulis akan memulai tulisan dengan mengenalkan sesuatu yang unik atau ganjil
secara singkat sehingga mengundang perhatian pendengar atau pembaca dan membuat penasaran bagaimana
itu bisa terjadi. Biasanya – walaupun tidak mutlak – penulis akan memulainya dengan mengajukan sebuah
pertanyaan.

2. Orientation
Setelah mampu membuat pendengar atau pembaca penasaran, kemudian penulis akan mulai dengan
memperkenalkan setting tempat, pelaku, waktu dari sebuah cerita. Bagian ini memiliki fungsi yang sama seperti
pada Narration Text.

3. Crisis
Bagian ketiga generic structure dari anecdote text adalah Crisis. Setelah pada bagian abstrack penulis
menceritakan hal unik secara singkat, pada bagian inilah kemudian penulis mempertegas keunikan dan
keganjilan yang terjadi sehingga grafik curiosity (penasaran) pembaca mengalami peningkatan.

4. Incident.
Pada bagian inilah penulis mengakhiri rasa penasaran akan hal ganjil atau unik dalam cerita tersebut
dengan menceritakan bagaimana pelaku memecahkan masalah atau keganjilan yang terjadi. Dan pada bagian
inilah biasanya pembaca atau pendengar mendapatkan pemecahan masalah secara menggelitik atau lucu.

5. Coda (optional/tidak harus)


Coda adalah kesimpulan yang bisa ditarik dalam cerita lucu tersebut yang biasanya akan mengundang
tawa pendengar atau pembaca yang lebih besar lagi. Namun bagian generic structure anecdote text ini tidak
selalu ada dalam cerita anecdote.

Language Feature of Anecdote


1. Using exclamation words; it‘s awful!, it‘s wonderful!, etc
2. Using imperative; listen to this
3. Using rhetoric question; do you know what?
4. Using action verb; go, write, etc
5. Using conjunction of time; then, afterward
6. Using simple past tense

B. Latihan Soal

One day our school organized a natural walk to Barely forest. We were very excited with this outing as
this was our first time participating in a nature walk with our seniors. It was a bright and sunny day, and we
enjoyed the tranquility of nature. It was a contrast from the hustle and bustle of city life.
Suddenly, James, a brave boy who always loved to perform daring stunts in class, caught up with us. He
whispered that he had found something exciting to show us. Naturally, we were curious, so we followed James.

SMA Alfa Centauri Bandung Halaman 27


MODUL PERSIAPAN UN & SBMPTN BAHASA INGGRIS 2017/ 2018

James showed us a pit, and next to it we found a rope and a flashlight. James, who was rather
mischievous, thought of an idea. He suggested that he would go down in a pit to see what was in there. We were
very excited and agreed to help him.
We tied the rope around his waist and started to lower him into the pit bit by bit. Just before James
reached the bottom, he shouted out,‖ Stop! Let me see if there is anything below me.‖
―All right, all clear!‖ shouted James as he landed safely. Then, without any warning, the flashlight
suddenly went out. James was standing gingerly at the bottom of the deep pit, plunged in total darkness with no
light. Suddenly, he heard a hissing sound next to him. He was rooted to the spot, frozen in fear. James cried for
help and followed by a loud painful scream. We could not see James, but we were horrified by his scream. We
quickly pulled the rope up.
Once James got out from the pit, trembling with fear, he shouted, ―Help! I have been bitten by a snake!‖
We burst into laughter when we saw the snake was nothing more than a plastic bag coiled tightly around James‘
ankle. James was embarrassed.

1. What is the main idea of the text above?


a. James and his friends wandered off from nature walk and explored the pit.
b. James learnt a lesson not to do silly things that put his classmates into trouble.
c. James‘ mischief had cost his classmates a great deal of trouble.
d. The writer and his friends were terrified by James‘ painful scream.
e. James will not pull any stupid stunts anymore.

2. Which of the statements is true?


a. The nature walk was organized by James.
b. The nature walk was held on Sunday.
c. James saw a snake in the pit.
d. James was fearful when the flashlight went off.
e. James and his friends were discovered by their teachers.

3. Which statement in the text suggested that the pit was explored before James discovered it?
a. James showed us the pit, and next to it we found a rope and a flashlight.
b. Naturally, we were curious, so we followed James.
c. We tied the rope around his waist and started to lower him into the pit.
d. He whispered that he had found something exciting to show.
e. We were very excited and agreed to help him.

4. Which of the following is not true according to the text?


a. James was a brave boy.
b. James and his friends were horrified.
c. James was not allowed to participate in nature walk.
d. James found the pit.
e. James went down in the pit.

SMA Alfa Centauri Bandung Halaman 28


MODUL PERSIAPAN UN & SBMPTN BAHASA INGGRIS 2017/ 2018

DESCRIPTIVE TEXT
A. Ringkasan Materi

Descriptive text is a text which says what a person or a thing is like. Its purpose is to describe and reveal a
particular person, place or thing.

Generic structure:
1. Identification (identifikasi) adalah pendahuluan , berupa gambaran umum tentang suatu topik.
2. Description (deskripsi) adalah berisi ciri-ciri khusus yang dimiliki benda, tempat, atau orang yang
dideskripsikan.

Language features:
1. Menggunakan simple present tense
2. Menggunakan attribute verb, seperti be (am, is, are)
3. Hanya fokus pada satu objek tersebut.

B. Contoh Soal dan Pembahasan


My Pet

I have a pet. It is a dog and I call it Dolgy. Dolgy is a Chinese breed. It is small, fluffy, and cute. It
has got thick black fur. When I cuddle it, the fur feels soft. Dolgy does not like bones. Every day it eats soft
food like steamed rice, fish, or bread. Every morning I give him milk and bread. When I am at school, Dolgy
plays with my rabbit. They get along well, and never fight maybe because Dolgy does not bark a lot. It treats
the other animals in our house gently, and it never eats shoes. Dolgy is reallt sweet and friendly animal.

1. The communicative purpose of this text is …. 3. Whom Dolgy plays with when the writer goes to
a. To describe a particular animal school?
b. To share an amusing incident with others a. The writer‘s sister.
c. To present two points of view about an b. The writer‘s mother.
issue c. The writer‘s cat.
d. To inform the readers about the beauty of d. The writer‘s other dog.
Dolgy e. The writer‘s rabbit.
e. To retell events for the purpose of
informing and entertaining Pembahasan:
Jelas tertulis pada kalimat, ―When I am at
Pembahasan: school, Dolgy plays with my rabbit‖.
Teks diatas merupakan teks deskripsi yang Jawaban: E
bertujuan untuk mendeskripsikan seekor
binatang yaitu anjing bernama Dolgy. 4. Waht does Dolgy eat every morning?
Jawaban: A a. fish and rice
b. bread and milk
2. How does Dolgy look like? c. bone and milk
a. Big, fierce, and cute. d. rice and bone
b. Big, fluffy, and fierce. e. milk and fish
c. Small, fierce, and stink.
d. Small, fluffy, and cute. Pembahasan:
e. Small, fluffy, and stink. Jelas tertulis pada kalimat, “Every day it eats
Pembahasan: soft food like steamed rice, fish, or bread.
Jelas tertulis pada kalimat ―It is small, fluffy, Every morning I give him milk and bread‖.
and cute‖. Jawaban: B
Jawaban: D

SMA Alfa Centauri Bandung Halaman 29


MODUL PERSIAPAN UN & SBMPTN BAHASA INGGRIS 2017/ 2018

C. Latihan Soal

This text is for questions 1-5


Orchard Road

Orchard Road is a Boulevard which becomes business and entertainment center in Singapore. Orchard
Road is surrounded by a lush tropical and flower gardens which are beautiful. At first, Orchard Road is just a
suburban street lined with orchards, plantations nutmeg, and pepper farming. However, in the 1970s, it turned
into a shopping center in Singapore. In 1960 and 1970 entertainment industries began to enter this road.
Shopping centers such as mall and Plaza was built in 1974.
Orchard Road runs along about 2.2 km. This road is one-way street flanked by a variety of shopping
malls, hotels and restaurants. The shopping area which is nearly 800,000 square meters provides a wide range
of Things, food, and entertainment. In this area there are many options that can satisfy visitors from all walks of
life starting from the luxury branded things to the Popular branded, from exclusive restaurants to fast food.
There are so many ways that can be accessed to get to Orchard road such as: by taxi, bus or drive your
own car. For those who are driving to Orchard Road can be entered from the west through the Napier Road.
Vehicles from Dunearn Road can turn to left at the intersection of the Marriott Hotel junction. Vehicles that come
from Paterson can turn right onto Orchard Road. Orchard is always crowded so you have to be careful in order
not to get lost.

1. The text mainly focuses on …. 3. In the third paragraph the writer describes
a. Singapore about?
b. Orchard Plantation a. The location of Orchard Road.
c. Plaza and Mall b. The things that we can see at Orchard
d. Orchard road as business and road.
entertainment center c. The direction to get to Orchard road.
e. Shopping Center d. The history of Orchard road.
e. The distance of Orchard road.
2. Which statement is TRUE?
a. At first Orchard Road is a crowded 4. Words ―it‖ in line 4 refers to?
settlement. a. The plantation.
b. Orchard Road became business and b. Luxury branded things.
entertainment center since 1974. c. The plaza.
c. Vehicles from Dunrean road turn to the left d. Singapore.
at intersection of the Marriot Hotel junction. e. Suburban street.
d. Orchard road is infamous place at
Singapore. 5. The word ―satisfy‖ in line eighth has the closet
e. Orchard road is not surrounded by flower meaning with?
garden. a. Pleased.
b. Free.
c. Frightem.
d. Threat
e. Loved.

SMA Alfa Centauri Bandung Halaman 30


MODUL PERSIAPAN UN & SBMPTN BAHASA INGGRIS 2017/ 2018

This text is for questions 6 to10.

Petruk Cave

Petruk cave is one of the leading tourist attractions in Kebumen, Central Java. The cave is located in
the dukuh Mandayana Candirenggo Village, Ayah District, Kebumen regency. In the petruk cave there is no
lighting that illuminates the cave. It is still very natural cave so that petruk cave is very dark to be entered. Petruk
cave‘s name is taken from the punokawan of puppet characters that is Petruk. The cave named Petruk cave
because the length of cave is as long as petruk‘s nose.
In the cave there are 3 floors that are the first is a basic cave, Hindu caves and Petruk cave. The base
cave is a short cave which is just 100 meters away. The cave is used for tourist attractions. Hindu cave is part of
the cave that is usually used to put offerings to the ancestor. Inside Petruk cave there are so many stalactites
and stalagmites which are really awesome. If you want to explore this cave, you must be led by guides who are
ready to take you through the cave. After arriving at the end of the cave, you can see the beach or waterfall
located near at the end of the cave.

6. What is the purpose of the text?


a. To inform readers about tourism in Kebumen.
b. To entertain reader about Petruk Cave.
c. To explain the reader about floors in Petruk Cave.
d. To Introduce The Local tourism in Kebumen.
e. To describe Petruk Cave.

7. Why did Petruk cave named as one of character in Punokawan puppet?


a. Because the cave is belong to Petruk.
b. Because Petruk is the first explorer of the cave.
c. Because Petruk is buried at the cave.
d. Because the cave‘s length is as long as Petruk‘s nose.
e. Because the cave‘s depth is as deep as Petruk‘s hair.

8. Which part of Petruk Cave which used for place to put foods for ancestor?
a. In the basic cave.
b. In petruk cave.
c. In Hindu cave.
d. In front of the cave.
e. Inside the cave.

9. What is stalactites means?


a. A type of formation that hangs from the ceiling of caves.
b. Types of formation that lay on the floor of caves.
c. Types of food given to ancestor.
d. Someone who guide the visitor in the cave.
e. Kind of animals in the cave.

10. What is ―lead‖ means in paragraph 2?


a. Guide.
b. Take.
c. Bring.
d. Put.
e. Place.

SMA Alfa Centauri Bandung Halaman 31


MODUL PERSIAPAN UN & SBMPTN BAHASA INGGRIS 2017/ 2018

REPORT TEXT
A. Ringkasan Materi

Report text is a text which presents information about something, as it is. It is as a result of systematic
observation and analysis.

Social function:
To describe the way things are with reference to a range of natural, man-made and social phenomena in our
environment.

Generic structures:
1. General classification: Stating classification of general aspect of thing; animal, public place, plant, etc
which will be discussed in general.
2. Description: Describing the thing which will be discussed in detail; part per part , customs or deed for
living creature and usage for materials.

Language Feature of Report:


1. Introducing group or general aspect
2. Using conditional logical connection; when, so, etc
3. Using simple present tense

Example of report text:

THE PELICAN REPORT


General Clasification The white pelican is one of the most successful fish-eating birds.
Description The success is largely due to its command hunting behaviour. A group, perhaps two
dozen birds, will gather in a curved arc some distance offshore. The birds then begin to
move forward towards the shore, beating the water furiously with their wings, driving the
fish before them.
When the water is shallow enough for the birds to reach the fish, the formation breaks
up as each bird dips its bill into the water to scoop up its meal. As the bird lifts its head,
the water drains from its bill leaving the fish which are then swallowed.
Pelicans are among the oldest group of birds, Fossils of this genus have been found
dating back 40 million years.

B. Contoh Soal dan Pembahasan

Belize, capital of British Honduras, has always been the colony‘s administrative, cultural, and
geographic center. It is a unique waterfront community characterized by large frame house with rambling,
screened verandas. Located on the periphery of the hurricane zone, and at an average elevation of two feet
above sea level, the town is vulnerable to any feet tidal wave. Hence most buildings are on stilts, and many
others have Spartan furnishings at the street level.
The cooling effect of sea breezes in a community surrounded on three sides by salt water relieves the
otherwise oppresive climate. Located on the Caribbean coast of central America slightly more than eight hndred
miles south of New Orleans and about the same distance West Jamaica. Belize had a population of nealy six
thousand in 1859, ten thousand in 1900, and reached thirty thousand in 1960‘s.

1. The following statements are TRUE about e. it is not located on the periphery of the
Belize, EXCEPT …. hurricane zone
a. geographical center
b. cultural center Pembahasan:
c. the colony‘s administrative Pada paragraph pertama jelas terdapat
d. capital of British Honduras pernyataan tentang Belize bahwa Belize
adalah capital of British Honduras, the colony‘s

SMA Alfa Centauri Bandung Halaman 32


MODUL PERSIAPAN UN & SBMPTN BAHASA INGGRIS 2017/ 2018

administrative, cultural, and geographical Spartan furnishings at the street level‖. Dari
center and located on the periphery of the pernyataan tersebut kita bisa mengetahui
hurricane zone. Dari pernyataan tersebut kita jawaban yang tepat adalah D.
bisa mengetahui bahwa pernyataan yang tidak Jawaban: D
tepat adalah E.
Jawaban: E 3. The word ‗vulnerable‘ (paragraph 1) means
2. Because of the dangers from storms and most nearly ….
waves, the houses of Belize …. a. protected against
a. are on the waterfront b. immune to
b. have large, screen verandas c. attracted to
c. enjoy cooling sea breezes d. defenseless against
d. are built on stilts e. provide something
e. are dirty
Pembahasan:
Pembahasan: ―… the town is vulnerable to any feet tidal
Pada paragraph pertama terdapat pernyataan ― wave‖. Kata vulnerable pada kalimat tersebut
… at an average elevation of two feet above bermakna rentan yang juga sama artinya
sea level, the town is vulnerable to any feet dengan defenseless against.
tidal wave. Hence (oleh karenanya) most Jawaban: D
buildings are on stilts, and many others have

C. Latihan Soal

Text 1
The following text is for questions 1 to 2.

Cameroon is a West African country of ten million people which has been very successful in
growing food for its people, unlike many other West African countries. Since 1971 it has doubled its
output of major foodstuffs as maize and potatoes. Now it is one of the few African countries able to feed
itself. This was not the cast five years ago when the country was only 75-80% self-sufficient in food.
Although isolated pockets of hunger still remain, the World Bank nowadays gives Cameroon money to
sustain, not to achieve, self-sufficiency in food production.

1. The text tells us about …. b. Cameroon is the biggest and the richest
a. West African countries country among West African Country.
b. Cameroon‘s ability to feed its people c. It was the World Bank which financed the
c. Major foodstuffs food production production of food in Cameroon
d. Cameroon‘s food production d. Similar to other West African countries,
e. Food self- sufficiency Cameroon has always been self-sufficient.
e. Cameroon has succeeded in its food
2. Which of the following statements is TRUE production because of the fertility of its
about Cameroon? soil.
a. The World Bank supports Cameroon to
maintain its self- sufficiency.

Text 2
The following text is for questions 3 to 4.

Although tame, pigeons are believed to be dangerous birds. A person died after suffering for two
weeks from a brain disease caused by the fungus on the droppings or pigeons. Fungus is a tiny plant
which grows on the droppings of pigeons. When the droppings dry up, the fungus is carried by the wind,
and those who breathes in the fungus can get a disease of the brain and may die. Only a strong person
may eventually recover.

SMA Alfa Centauri Bandung Halaman 33


MODUL PERSIAPAN UN & SBMPTN BAHASA INGGRIS 2017/ 2018

3. The topic of the text is …. 4. The text suggests that pigeons may cause
a. the cause of human brain disease people‘s death when ….
b. pigeons, a potential source of brain a. their droppings are making the area dirty
disease b. the wind spread the smell of their
c. how fungus contaminates human being droppings all over the area
d. fungus, a deadly tiny plant c. people inhale the fungus carried by the
e. the danger of pigeons droppings wind
d. their droppings fertilize and make
fungus grow
e. pigeon droppings directly harm people‘s
brain

Text 3
The following text is for questions 5 to 7.

Two new species of dinosaurs, one quick – moving meat-eater and the other a giant plant- eater,
have been discovered in Antarctica. The 70 million – year- old fossil of the carnivore would have rested
for millenniums at the bottom of the Antarctic sea, while the remain of the 30- meter long plant-eater
were found on the top of a mountain.
The little carnivore- about 1.8 meters tall –was found on James Ross Island, off the coast of the Antarctic
Peninsula.
Not yet named, the animal probably floated out to the sea after it died and settled to the bottom of what
was then a shallow area of the Weddell Sea. Its bones and teeth suggest it may represent a population of
two –legged carnivores that survived in the Antarctic long after other predators took over elsewhere on the
globe.‖ For whatever reason, they were still hanging out on the Antarctic continent,‖ Case said in a
statement.
A second team led by William hammer of Augustana college in Rock Island, Illinois, found the 200 million –
year- old plant-eater‘s fossils on a mountaintop 13.000 feet (3,900 meters) high near the Beardmore glacier.
Now, known as Mt. Kirk Patrick, the area was once a soft riverbed.

5. The article informs us about ….


a. two species of dinosaurs
b. the newest discovery in Antarctica
c. the two kinds of new dinosaurs
d. the discovery of two species of dinosaurs in Antarctica
e. a team of researchers was funded by the National Science Foundation

6. What is the main idea of the first paragraph?


a. Two species of dinosaurs are carnivores and herbivores.
b. The two species were funded by the National Science Foundation.
c. The two species are a quick moving meat- eater and a giant –eater.
d. The two species of dinosaurs were floating at the bottom of the Weddel Sea.
e. Two new species of dinosaurs have been discovered in Antarctica.

7. The following information is about the giant plant- eater dinosaur, EXCEPT ….
a. it is 30 meters long
b. it is 200 million years old
c. it is 1.8 meters tall
d. it is found on the top of a mountain
e. it is found by William Hammer

SMA Alfa Centauri Bandung Halaman 34


MODUL PERSIAPAN UN & SBMPTN BAHASA INGGRIS 2017/ 2018

EXPLANATION TEXT
A. Ringkasan Materi

Definition:
Explanation text (text explanation) is a text that contains the processes associated with natural phenomena,
social, science, culture and more. An Explanation text is usually derived from the corresponding author
questions 'why' and 'how' of a phenomenon.

Social Function:
To explain the processes involved in the formation or workings of natural or sociocultural phenomena.

Generic structure:
 A general statement:
It contains a general explanation of the phenomenon that will be discussed. This phenomenon could be
the introduction or explanation.
 A sequence explanation:
It contains an explanation of why the phenomenon may occur or be created. A sequenced of
explanation in the form of answers to the question 'why' and 'how' the author when making an
Explanation text. In sequenced of explanation may consist of more than one paragraph.

Significant Lexico grammatical Features:


 Focus on generic non-human participants
 Use mainly of material and relational processes
 Use mainly of temporal and causal circumstances and conjunctions
 Some use of passive voice to get theme right
 Use simple present tense
 Use Action verbs

The example of Explanation text:


General statement Speech production is made possible by the specialized movements of or vocal organs
that generate speech sound waves.
Explanation Like all sound production, speech production requires a source of energy. The source of
energy for speech production is the steady stream of air that comes from the lungs as we
exhale. When we breathe normally, the aim stream is inaudible. To become audible, the
air stream must vibrate rapidly. The vocal cords cause the air stream to vibrate.
Explanation As we talk, the vocal cords open and close rapidly, chopping up the steady air stream into
a series of puffs. These puffs are heard as a buzz. But this buzz is still not speech.
Explanation To produce speech sounds, the vocal tract must change shape. During speech we
continually alter the shape of the vocal tract by moving the tongue and lips, etc. these
movements change the acoustic properties of the vocal tract, which in turn produce the
different sounds of speech.

B. Contoh Soal dan Pembahasan

Photosynthesis is the process of converting solar energy to chemical energy and storing it in bonds of
sugar. This process occurs in plants and some algae (Kingdom Protista). Plants need only solar energy,
CO2 and H20 to make sugar. The process of photosynthesis takes in the chloroplasts, specifically using
chlorophyll, the green pigment involved in photosynthesis.
Photosynthesis takes place primarily in plant leaves, and little to none occurs in stems, etc. The parts of
a typical leaf include the upper and lower epidermis, the mesophyll, the vascular bundle(s) (veins), and the
stomates. The upper and lower epidermal cells do not have chloroplasts, thus photosynthesis does not
occur there. They serve primarily as protection for the rest of the leaf. The stomates are holes which occur
primarily in the lower epidermis and are for air exchange: they let CO2 in and O2 out. The vascular bundles

SMA Alfa Centauri Bandung Halaman 35


MODUL PERSIAPAN UN & SBMPTN BAHASA INGGRIS 2017/ 2018

or veins in a leaf are part of the plant‘s transportation system, moving water and nutrients around the plant
as needed. The mesophyll cells have chloroplasts and this is where photosynthesis occurs.
As you hopefully recall, the parts of a chloroplast include the outer and inner membranes, inter
membrane space, stroma, and thylakoids stacked in grana. The chlorophyll is built into the membranes of
the thylakoids.
Chlorophyll looks green because it absorbs the red and blue light, making these colors unavailable to be
seen by our eyes. It is the green light which is not absorbed that finally reaches our eyes, making chlorophyll
appear green. However, it is the energy from the red and blue light that are absorbed that is, thereby, able to
be used to do photosynthesis. The green light we can see is not/cannot be absorbed by the plant, and thus
cannot be used to do photosynthesis.

1. What is the function of the upper and lower


epidermis? Pembahasan:
a. To let CO2 in and O2 out Kata sambung ‗however‘ = akan tetapi,
b. To enable photosynthesis bagaimanapun juga, dapat diganti dengan kata
c. To absorb leaf green lights sambung yang bersifat ‗contradictive‘ =
d. To protect the rest of the leaf berlawanan/bertentangan seperti but,
e. To transport the nutrient nevertheless, yet, still.
Jawaban: A
Pembahasan:
Fungsi utama dari epidermis atas dan bawah 3. The passage indicates that photosynthesis
adalah melindungi daun. Informasi tersebut takes place in ….
dapat kita temukan pada paragraph kedua dari a. the veins
kalimat ―They serve primarily as protection for b. the stomates
the rest of the leaf‖. c. the membranes
Jawaban: D d. the epidermis
e. the mesophyll
2. However, it is the energy from the red and blue
light that are absorbed that is, thereby, able to Pembahasan:
be used to do photosynthesis. The underlined proses fotosintesis terjadi pada ‗mesophyll‘.
conjunction can also be replaced by …. Informasi tersebut dapat kita temukan pada
a. but . pragraf kedua kalimat terakhir yaitu ―The
b. moreover mesophyll cells have chloroplasts and this is
c. furthermore where photosynthesis occurs‖
d. therefore Jawaban: E
e. hence

C. Latihan Soal

Text 1 is for questions 1 to 6

There are three reasons why solar energy generation has not development more rapidly. First, the
cost per watt of solar cell generation is more expensive than that steam power or nuclear power
generation. Therefore, researchers are still looking for ways to make solar cells cheaper. Second,
nature plays a large part in solar cell generation. For example, some days are cloudy and rainy; because
there is not always fine weather, solar energy cannot be generated every day. No solar energy is
generated at night. Consequently, solar cells must have the capacity to store energy for use during
these times, and these storage cells are very expensive. As a result, only in low latitude areas can
enough solar energy be generated effectively at these times. Finally, building plants for solar cell
generation is extremely expensive. A very large space is needed, and the needed, and the need for
maintenance is constant. To illustrate, the surface of the solar cell plants has to be cleaned daily.
For all these reason, it is very difficult to develop solar cell plants and makes solar energy available to
the general public for a competitive price. Despite all these problems, I believe the day will come when we
use solar energy cells because they are a form of clean energy with no pollution.

SMA Alfa Centauri Bandung Halaman 36


MODUL PERSIAPAN UN & SBMPTN BAHASA INGGRIS 2017/ 2018

1. The text describes … of the slow development 4. The development of solar energy for public use
of solar cell energy. has been rather slow due to the following,
a. the process EXCEPT ….
b. the causes a. the nature of solar energy itself
b. the high cost of production
c. the purpose c. the condition of nature
d. the benefit and harm d. the high construction costs of the plant
e. the condition e. the daily maintenance of the plant

2. Efforts have been made to develop solar cell 5. The author expects that ….
energy because …. a. effort to develop solar cell energy will be
a. it can be generated at any time stopped immediately.
b. it is reversible b. nuclear power will be a better energy
c. it is easy to generate alternative
d. it does not cause any pollution E c. solar energy will become popular
e. it is not costly among consumers
d. the generation of solar cell can be
3. Which of the statement is TRUE about solar done in lower attitude areas
cells? e. solar energy will remain difficult to
a. Great progress has been made in the develop
development of solar cell energy.
b. Solar cells can generate and store energy 6. The purpose of writing this text is to ….
during the night. a. persuade the readers that something in
c. Unlike solar cell energy, nuclear energy is the case
more expensive to generate. b. explain the process involved in a
d. Producing solar energy is quite a profitable phenomenon.
business nowadays. c. describe how something works
e. Generating solar cell energy greatly d. convince the readers to share the
depend on the condition of the weather. writer‘s opinion
e. show different points of view over an issue

Text 2
The following text is for questions 7 to 10.

Once a memory is created, it must be stored (no matter how briefly). Many experts think there are three
ways we store memories; first in the sensory stage; then in short term memory; and ultimately, for some
memories, in long-term memory. Because there is no need for us to maintain everything in our brain, the different
stages of human memory function as a sort of filter that helps to protect us from the flood of information that
we‘re confronted with on a daily basis.
The creation of a memory begins with its perception. The registration of information during perception
occurs in the brief sensory stage that usually lasts only a fraction of a second. It‘s your sensory memory that
allows a perception such as a visual pattern, a sound, or a touch to linger for a brief moment after the stimulation
is over.
After the first flicker, the sensation is stored in short-term memory. Short-term memory has a fairly
limited capacity; it can hold about seven items for no more than 20 or 30 seconds at a time.
Important information is gradually transferred from a short-term memory into long-term memory. The
more the information is repeated or used, the more likely it is to eventually end up in long-term memory, or to be
‗retained‘. Unlike sensory and short-term memory, which are limited and decay rapidly, long-term memory can
store unlimited amounts of information indefinitely.
People tend to more easily store material on subjects that they are already know something about,
since the information has more meaning to them and can be mentally connected to related information that is
already stored in their long-term memory. That‘s why someone who has an average memory may be able to
remember a greater depth of information about one particular subject.

SMA Alfa Centauri Bandung Halaman 37


MODUL PERSIAPAN UN & SBMPTN BAHASA INGGRIS 2017/ 2018

7. The text is about ….


a. how human brain works
b. how the long-term memory occurs
c. how to protect our memory from being lost
d. how to store information in our memory
e. how to maintain everything in our brain

8. According to the text, ….


a. not all information in short-term memory is stored in long-term memory
b. the process of transfer from short-term to long-term memory is very fast
c. the process of transfer from short-term memory is no more than 20 seconds
d. short-term memory have unlimited capacity
e. the perception of information happens after the creation of memory

9. We know from the text that ….


a. we can store all information we get in our long-term memory
b. the process of storing information is done in three stages
c. the work of our memory depends on the capacity of it
d. short-term memory is the most important memory
e. information just stays for a while in our memory

10. ―… that helps to protect us from the flood of information that we‘re confronted with on a daily basis‖.
(Paragraph 1)
The underlined word is closest in meaning to ….
a. encountered
b. compared
c. united
d. expressed
e. interpreted

SMA Alfa Centauri Bandung Halaman 38


MODUL PERSIAPAN UN & SBMPTN BAHASA INGGRIS 2017/ 2018

ANALYTICAL EXPOSITION
A. Ringkasan Materi

Analytical exposition text is a type of spoken or written text that is intended to persuade the listeners or
readers that something is the case. To make the persuasion stronger, the speaker or the writer gives arguments
as the fundamental reasons why something is the case. That‘s why analytical exposition sometimes is also
called a persuasive text or argumentative text. This type of text can be found in scientific books, journals,
magazines, newspaper articles, academic speech or lectures, etc. analytical expositions are popular among
science, academic community, and educated people.

The analytical expositions are organized in three stages:


1. Statement of position (the thesis)
This stage usually includes a ―preview of arguments‖. It introduces topics and indicates the writer‘s position; outlines
the arguments to be presented.

2. Arguments
This stage consists of a ―point of elaboration‖ sequence. The number of points may vary, but each must be
supported by discussions and evidence (which is the elaboration of the point). Points are ordered according to the
writer‘s choice, but it is usual to discuss the strongest point first. Point states the main argument and elaboration
develops and supports each point of argument.

3. Reinforcement (restatement) or conclusion


This last stage restates the position more forcefully.

Language features of analytical exposition text:


1. An analytical exposition focuses on generic human and non human participants.
2. It uses mental processes: to state what the writer / the speaker thinks or feels about something. For
examples: realize, feel, etc.
3. It often needs material processes: to state what happens, e.g. has polluted, etc.
4. It usually uses Simple Present tense and Present Perfect tense.
5. Enumeration is necessary to show the list of given arguments: the first, the second, secondly, finally, etc.
6. Analytical exposition text usually used complex and compound sentences.

Here is the example of analytical exposition text.

SMA Alfa Centauri Bandung Halaman 39


MODUL PERSIAPAN UN & SBMPTN BAHASA INGGRIS 2017/ 2018

Paragraph 1 Thesis
Paragraph 2-4  Argument
Paragraph 5  Reiteration

B. Contoh Soal dan Pembahasan

Integrated Pest Management

There is no best way to deal with pests in agriculture. Pesticides which are commonly used may cause
many problems. I think combining different management operations is the most effective way to control pests.
Firstly, the chemicals in the pesticides may build up as residues in the environment and in the soil which
absorbs the chemicals. This reduces the quality of farm product.
Secondly, pests can gradually become resistant to pesticides. This means that newer and stronger ones
have to be developed.
Lastly, some pesticides affect non target plants and animals such as fish and bees. This affects the
ecology and environment as well.
So, understanding of ecology of an area helps a lot in pest control. Pesticides should be chosen and
applied carefully so that they don't affect the ecological balance and environment.
Therefore, integrated pest management is a safe and more effective option to fight pest in agriculture
and livestock.

1. Which of the following is not directly affected by quality of farm product and affect plants,
pesticides used? animals, ecology and environment as well.
a. Plants. Jawaban: B
b. Ecology.
c. Animals. 3. One of the disadvantages of using chemical
d. Environment. pesticides is ….
e. Human beings. a. Killing fish and bees
b. Increasing crops productivity
Pembahasan: c. Creating balanced ecosystem
Jelas tertulis pada paragraph 4, some d. Causing the pests to become inactive
pesticides affect plants, animals, ecology and e. Helping reduce pollutants in the
environment. environment
Jawaban: E
Pembahasan:
2. What can you say about paragraph two and Jelas tertulis pada paragraph keempat.
four? Jawaban: A
a. The fourth paragraph supports the idea
stated in paragraph two. 4. Secondly, pests can gradually become
b. Both paragraphs tell about the resistant to pesticides. (paragraph 3)
disadvantages of using pesticides. The word resistant in the sentence above
c. Both paragraphs tell about how pesticides means ….
affect the quality of farm products. a. Weak
d. The statement in paragraph is contrary to b. Fragile
the statement in paragraph four. c. Damage
e. The second paragraph tells about the d. Unaffected
effects of using pesticides on animals e. unbalanced
mentioned in paragraph four.
Pembahasan:
Pembahasan: Kata resistant pada kalimat tersebut bermakna
Kedua paragraph tersebut membahas tentang ‗tahan/kebal‘ memiliki kesamaan arti dengan
kerugian menggunakan pestisida (reduce the ‗unaffected‘.
Jawaban: D

SMA Alfa Centauri Bandung Halaman 40


MODUL PERSIAPAN UN & SBMPTN BAHASA INGGRIS 2017/ 2018

C. Latihan Soal

The following text is for question 1 to 4.

Dust Bin

To improve comfort and cleanliness at our school, a number of dust bins should be increased.
When we look at classrooms, school corridors and school yard, there are paper mineral water cups,
straws, and napkins here and there. The condition of uncleanliness and discomfort really hinders learning and
teaching environment. Litters thrown carelessly cause disease, especially empty plastic cup or glasses. They
can be filled out with water coming from the rain. This can be placed for dengue mosquitoes to spread
out. Besides, these rubbish can deteriorate the scene. Well painted wall and green school yard do not mean
anything litters are scattered everywhere.
Anyway I notice that most of the students in our school have responsibilities for the school
environment. They put their litters on the proper places. But some are not diligent enough to find the dust
bins. The numbers of dust binds in our schools are not enough. Ore dust bins should be put beside each of
steps, outside of the classrooms, and some more also the corridors. Probably one dust bin should be in every
ten meters. So when students want to throw away their litters, they can find the dust bins easily.
When school is equipped with sufficient dust bins, we do not have problems of freak and discomfort any
more. Our school will be very clean and become a nice place to study.

1. What is the writer‘s intention? To … readers to


do something good. 3. What is the writer‘s argument on a sufficient
a. inform number of dust bins?
b. explain a. They can prevent litters.
c. describe b. They can save janitor‘s energy.
d. entertain c. Students are asked to clean them.
e. persuade d. They make school environment neat.
e. Students can throw garbage away easily.
2. According to the writer, more dust bins … in
every ten meters. 4. What is the writer‘s suggestion?
a. should be decorated a. To buy more dustbins.
b. should be painted b. To hire more gardeners.
c. should be placed c. To use dustbins efficiently.
d. are unnecessary d. To ask parents to give more dustbins.
e. are not required e. To ask students to clean the school yard

The following text is for questions 5 to 8.

The Importance of Reading

I personally think that reading is a very important activity in our life. Why do I say so? Firstly, by
reading we can get a lot of knowledge about many things in the world such as Science, technology. Sports, arts,
culture, etc written in either books, magazine, newspaper, etc.
Secondly, by reading we can get a lot of news and information about something happening in any parts
of the world which can we see directly.
Another reason, reading can give us pleasure too. When we are tired, we read books, newspaper or
magazine on the entertainment column such as comedy, short story, quiz, etc. To make us relaxed.
The last, reading can also take us to other parts of the world. By reading a book about Irian Jaya we
may feel we‘re really sitting in the jungles not at home in our rooms.
From the facts above, it‘s obvious that everyone needs to read to get knowledge, information and also
entertainment. Or in summary we can say reading is truly important in our life.

SMA Alfa Centauri Bandung Halaman 41


MODUL PERSIAPAN UN & SBMPTN BAHASA INGGRIS 2017/ 2018

5. Why is reading very important in our life? Because ….


a. By reading, we can get a lot of friends, relatives, experience, etc.
b. By reading, we can get little knowledge but a lot of entertainment
c. By reading, we are always relaxed
d. By reading, we are always happy
e. By reading, we can get a lot of knowledge, news, information and entertainment

6. If we want to get knowledge, what should we do?


a. Buy a lot of books.
b. Borrow a lot of books.
c. Look for newspaper and magazine.
d. Sell and buy many expensive books.
e. Read a lot of books and other printed materials.

7. What does the text tell us about?


a. The description of reading.
b. The function of reading.
c. The importance of reading.
d. The disadvantages of reading.
e. The purpose of reading.

8. What is the social function of the text?


a. To tell a story.
b. To describe the reader.
c. To entertain the reader.
d. To give information.
e. To persuade the reader.

SMA Alfa Centauri Bandung Halaman 42


MODUL PERSIAPAN UN & SBMPTN BAHASA INGGRIS 2017/ 2018

HORTATORY EXPOSITION
A. Ringkasan Materi

A hortatory exposition is a type of spoken or written text that is intended to explain the listeners or
readers that something should or should not happen or be done. In other words, the main function Hortatory
Exposition texts is to persuade the reader or listener that something should or should not be the case.
Hortatory exposition text can be found in scientific books, journals, magazines, newspaper articles,
academic speech or lectures, research report etc. Hortatory expositions are popular among science, academic
community and educated people. It means that the text that asserts something to the readers by giving
supporting statement and evidence to convince it. To make the persuasion stronger, the speaker or writer gives
some arguments as the fundamental reasons why something is the case.

The generic structure of hortatory exposition text


According to Hartono (2005), the generic structures of hortatory exposition are:

1. Thesis
Thesis is similar to tentative conclusion which needs to be proven by certain fact and argument. In the end, it can
be true or false. Thesis is introducing the topic and indicating the writer‘s position. The thesis consists of
statement or announcement of issue concern.

2. Arguments
This is the phases which try to examine and support that the thesis stated above is true. It is explaining the
arguments to support the writer‘s position. The arguments then will lead to recommendation. Example: Firstly,
smoking in a restaurant is impolite. The smell of the smoke affects all people and can turn them off their food.
People pay to taste good food and not to be put off by foul smelling smoke.

3. Recommendation
This part give suggestions which contains what should or should not be done or happen based on the given
arguments. This recommendation is differentiating from analytical exposition. It is restating the writer‘s position.
Example: It is suggested that smokers consider stopping smoking for their health. Therefore, smoking in
restaurants is impolite, harmful to others and a health risk to the smokers and should not be allowed in any
restaurants.

Language features of hortatory exposition text


1. A hortatory exposition focuses on generic human and non-human participants, except for speaker writer
referring to self.
2. It uses mental processes: to state what the speaker / writer thinks or feels about something, e.g. realize,
feel.
3. It often needs material processes: to state what happens, e.g. .... has polluted...., etc.
4. It usually uses Simple Present Tense and Present Perfect Tense.
5. Enumeration is sometimes necessary to show the list of the given arguments: Firstly, secondly, finally,
etc.
6. Modal auxiliary is usually used: can, may, should, etc.

SMA Alfa Centauri Bandung Halaman 43


MODUL PERSIAPAN UN & SBMPTN BAHASA INGGRIS 2017/ 2018

Here is the example of hortatory exposition text:

Paragraph 1  thesis
Paragraph 2  argument 1
Paragraph 3  argument 2
Paragraph 4  Recommendation (statement of what should or should not happen)

B. Contoh Soal dan Pembahasan

Keeping dogs
There are many reasons for keeping dogs as pets in the city area, but many people feel that keeping
pet dogs should not be allowed in the city. Dogs are often not taken care properly. They are kept in small
backyards and rarely taken for walks. They are left in the yard all day by themselves while the family is at work.
Very little attention is given to the dog and it is not a very good life for it. It is no wonder why these dogs bark and
disturb the neighbors and become a nuisance to the community.
On the other hand, not all people treat their dogs this way and why should the people receive a lot of
pleasure and enjoyment from dog's suffer? Dogs can make a lonely person's life happy or make wonderful
playmate. It can also teach a child responsibility as they not only get to lay with the dog, but also need to
exercise, feed, and care for the dog.
I feel that we should be allowed to keep dogs in the city, because if it is taken care of properly, dogs can
be a great source of pleasure. As a child I used to enjoy playing and taking care of my pet. There is not greater
loyalty a person can get than from a well-cared dog.
e. the dogs guard the security of the
1. Some people do not like the presence of pet environment
dogs in their surroundings because ....
a. the dogs are left in the yards Pembahasan:
b. the dogs are often neglected Pada paragraph pertama dipaparkan
c. the dogs often bark and disturb the bahwa beberapa orang tidak menyukai
neighbors kehadiran anjing peliharaan di
d. the dogs play with the owner's child lingkungannya karena mereka dibiarkan di
halaman belakang sehingga kurang

SMA Alfa Centauri Bandung Halaman 44


MODUL PERSIAPAN UN & SBMPTN BAHASA INGGRIS 2017/ 2018

diperhatikan oleh pemiliknya. Hal ini Jawaban: C


menyebabkan anjing-anjing tersebut
menggonggong dan mengganggu 3. We can infer from the text that keeping pet
penduduk sekitar. dogs in the city will not cause any problem as
Jawaban: C long as ....
a. the dogs are amusing
2. The main idea of the last paragraph is .... b. the neighbours like having pet dogs
a. pet dogs are loyal animals c. the dogs do not bark at the neighbors
b. pet dogs are more loyal than humans d. the writer has succeeded in persuading the
c. dogs can be a great source of pleasure neighbors
d. the writer has no objection to keeping dogs e. its presence does not disturb the
in the city neighbourhood
e. the writer used to play and took care of his
pet when he was a child Pembahasan:
Yang menjadi argumentasi bagi mereka yang
Pembahasan: tidak setuju adalah keberadaan anjing
Paragraph terakhir pada teks hortatory berisi peliharaan yang mengganggu penduduk
rekomendasi dari penulis terhadap isu atau sekitar sehingga bisa kita simpulkan bahwa
permasalahan yang sedang dibahas. Penulis keberadaan anjing tidak akan menjadi
merekomendasikan bahwa kita harus masalah apabila kita bisa menjaganya dengan
diperbolehkan untuk membawa anjing baik sehingga tidak mengganggu orang-orang
peliharaan ke kota karena hal tersebut bisa di sekeliling.
menjadi sumber kesenangan. Jawaban: E

C. Latihan Soal

The following passage is for questions 1 to 6.

Old Enough?

When a person old enough to be responsible for a crime? This question needs to be investigated
because the current law is not good enough.
The law at present protects children aged between ten years and four years from being punished for
committing crimes. It is believed that children under fourteen years are too young to realize the seriousness of
their crimes. Mr. Stephen Scarlett, head of the NSW Children‘s Court, describes how clever young offenders use
this defense, saying that they are too young to understand that they have broken the law. Mr. Scarlett, an expert
on this subject, states that the age should be dropped from fourteen to twelve years. I agree with this.
Teenagers these days are far more sophisticated than those in the past. The law should recognize this.
In a recent survey some people suggested that the age for being responsible for a crime should be dropped to
eight years old. This, I feel, is too young. Fourteen, however, is too old. Is there anyone who believes that a
fourteen-year-old does not know that it is against the law to steal or vandalize property? By the age of twelve,
children are aware of what is legal and what is not.
Public pressure creates change. It is now up to the public to put pressure on the government to change
the age at which a person may be held responsible for a crime from fourteen years to twelve years. Out of date
laws have no place in a modern society, especially one that needs people to be responsible for their actions.

1. What does the text above talk about?


a. When a person is old enough to be responsible for a crime.
b. When a person is being punished for committing crimes.
c. When children are too young to be responsible for a crime.
d. When a person is too old to be responsible for a crime.
e. When do teenagers realize the seriousness of their crimes.

SMA Alfa Centauri Bandung Halaman 45


MODUL PERSIAPAN UN & SBMPTN BAHASA INGGRIS 2017/ 2018

2. In what age are the children protected by the present law from being punished for committing crimes?
a. Ten years old.
b. Fourteen years old.
c. Ten to fourteen years old.
d. Eight years old.
e. Twelve years old

3. In what paragraph do we find the recommendation of the text above ….


a. In the first paragraph
b. In the second paragraph
c. In the third paragraph
d. In the fourth paragraph
e. In the fifth paragraph

4. The part of the text which states what ought or ought not to happen is called ….
a. thesis
b. argument
c. elaboration
d. reiteration
e. recommendation

5. ―Teenagers these days are far more sophisticated than those in the past.
The word ―those‖ refers to ….
a. days
b. teenagers
c. laws
d. children
e. crimes

6. ―Mr. Scarlett, an expert on this subject, states that the age should be dropped from fourteen to twelve years.
I agree with this.‖
The underlined sentence expresses ….
a. argument
b. opinion
c. certainty
d. debate
e. agreement

SMA Alfa Centauri Bandung Halaman 46


MODUL PERSIAPAN UN & SBMPTN BAHASA INGGRIS 2017/ 2018

DISCUSSION
A. Ringkasan Materi

Discussion is a text which present a problematic discourse. This problem will be discussed from different
viewpoints. Discussion is commonly found in philosophical, historic, and social text.

Generic Structure of Discussion:


 Statement of issue; stating the issue which is to discussed
 List of supporting points; presenting the point in in supporting the presented issue
 List of contrastive point; presenting other points which disagree to the supporting point
 Recommendation; stating the writer‘ recommendation of the discourse

Language Feature of Discussion:


 Introducing category or generic participant
 Using thinking verb; feel, hope, believe, etc
 Using additive, contrastive, and causal connection; similarly, on the other hand, however, etc
 Using modalities; must, should, could, may, etc
 Using adverbial of manner; deliberately, hopefully, etc

B. Contoh Soal dan Pembahasan

There are a lot of discussions as to whether children should be given homework or not.
Some people claim that children do enough work in school already. They also argue that children have
hobbies that they want to do after school, such, as sports or music. A further point they make is that a lot of
homework is pointless and doesn‘t help the child learn at all.
However, there are also strong arguments against this point of view. Parents and teachers argue that it
is important to find out whether children can work on their own without the support from the teacher. They say
that the evening is a good time for children to sit down and think about what they have learned in school.
Furthermore they claim that the school day is too short to get anything done that needs doing and it
makes sense to send home tasks like independent reading or further writing tasks that don‘t need teacher‘s
support.
On balance, I think that some homework is a good idea but that it should only given at the weekend
when children have more time.

1. How do we compare the second and the fourth Pembahasan:


paragraph? Sangat jelas pada paragraph kedua
a. Both paragraphs argue that homework is menyatakan bahwa pekerjaan rumah (PR)
necessary for students. adalah tidak perlu bagi siswa, bertentangan
b. Unlike the fourth paragraph, the second dengan paragaf keempat.
paragraph argues that homework is Jawaban: B
unnecessary.
c. The second and the fourth paragraphs do 2. Those who are pro homework think that the
not say anything about the benefit of students can … in the evening.
homework. a. prepare for the next lesson
d. The second paragraph and the fourth b. review their lessons
paragraph argue that students do not need c. enjoy their pastime
homework. d. do their hobbies
e. The second paragraph supports that e. test themselves
students need homework, but the fourth
paragraph does not. Pembahasan:
Dari paragraph ketiga kalimat ketiga yaitu,
―They say that the evening is a good time for

SMA Alfa Centauri Bandung Halaman 47


MODUL PERSIAPAN UN & SBMPTN BAHASA INGGRIS 2017/ 2018

children to sit down and think about what they Uninteresting means boring, uneventful, not
have learned in school‖, kalimat tersebut interesting.
mempunyai makna yang sama dengan review Dari definisi-definisi diatas, bisa disimpulkan
their lessons. bahwa pointless memiliki arti yang sama
Jawaban: B dengan unimportant.
Jawaban: D

3. ―A further point they make is that a lot of


homework is pointless and doesn‘t help the 4. What is the writer‘s suggestion about
child learn at all.‖ (Paragraph 2) The underlined homework?
word is synonymous with … a. Homework is pointless.
a. terrible b. Homework is badly needed.
b. careless c. Homework should be given at weekend.
c. difficult d. Student should not be given homework.
d. unimportant e. Student must frequently have homework.
e. uninteresting
Pembahasan:
Pembahasan: Paragraph terakhir berisi kesimpulan dari
Pointless means ridiculous, senseless. penulis terkait issue yang sedang dibahas. Dari
Terrible means bad, horrible. paragraph terakhir tersebut jelas dinyatakan
Careless means without sufficient attention. bahwa penulis setuju dengan adanya PR tapi
Difficult means hard on someone, hard to do. PR tersebut harus diberikan di akhir pecan
Unimportant means of no real worth, value, not disaat anak-anak punya banyak waktu luang.
important. Jawaban: C

C. Latihan Soal

Text 1
The following text is for questions 1 to 4.

The government has just published a report which suggest that television is partly responsible for
the serious increase in crime over the last ten years. The exposure of violence of pornography harmfully affects
on children.
Many people who are alive today know what it is like a world without television. Television as we know
is only about forty years old. Yet it is so much a part of our lives that it seems as if it had always existed.
Some people think that years before the invention of television were a better time. They claim that
families talked more and did more things together. More books were read. People used their imaginations more
fully. People got more outdoor exercises.
But others disagree. They claim that television is a power education tool. It informs us of what is
going on in the world, from a famine in Africa to a local politics and fashions. It helps us understand how
people live, work, and struggle.
In 1961, Newton Minow, a government official, called prime-time schedules ―a vast wasteland‖. Even
though Minow said that over thirty years ago, many feel that it is still true today. Television is credited with being
a great teacher, but it is also blamed for the poor reading and writing skills of our population. Television gets
praised for helping us understand the people of the world. But it has been accused of helping to destroy family
life. Television keeps us informed about the political issues of the day.
Expert will probably continue to argue about television‘s value. But everyone agrees that it is one of the
most significant inventions of the twentieth century.

SMA Alfa Centauri Bandung Halaman 48


MODUL PERSIAPAN UN & SBMPTN BAHASA INGGRIS 2017/ 2018

1. The writer wants to tell ….


a. the development of television 3. Which of the following is good for children in
b. the advantages of television programs watching TV?
c. how television destroys people‘s a. Children should watch the violence on TV.
knowledge b. Children may watch TV whenever they
d. how television improves people‘s like.
knowledge c. There shouldn‘t be government censorship
e. the influence of television of people‘s daily of TV program.
life d. Children should spend all of their time to
watch TV program.
2. Many people claim that television is a power e. Children should be accompanied by their
tool. From this statement we know that they … parents in watching TV.
with the existence of television.
a. love 4. ―… What is going on in the world, from a
b. agree famine in Africa….‖(paragraph 4). The
c. prefer underlined word refers to ….
d. satisfy a. food
e. choose b. water
c. nutrient
d. education
e. entertainment

Text 2
The following text is for questions 5 to 7.

There are many arguments in favor of a boarding school education. Nevertheless, the boarding school
is not always the best education institution for everyone.
At an early age interacting and communicating with people is very important for a child's personal life
and can be especially helpful for his/her future. In a boarding school, shy children can take advantage of
interaction through communal activities. The boarding school also offers a great variety of activities such as arts,
sports, and music that allow children to demonstrate and develop specialized skills in their free time.
Furthermore, the structured way of life and strict rules at the boarding school may helps students to get
used to a well ordered way of life. The manners and social skills will help them to become more responsible
and confident, and to develop their talents in leadership. Professionally trained teachers and educators in the
boarding school can offer excellent education without the parents' constant supervision.
In spite of these good points, it is not advisable to send a child to a boarding school if he/she is a
dependent leaner. A boarding school usually demands that student learn independently. In addition, the
boarding school should not be seen as a measure to solve inappropriate behavior or unsatisfactory study
performance. Instead of improving, problematic children may close off their relationship with their peers and
teachers.
In conclusion although a boarding school may provide good education to many children, it is not
recommended for those who are strongly attached to their families. They may become frustrated and socially
isolated.

5. According to the writer, children in a 6. Why do parents send their children to Boarding
boarding school can develop specialized School? Because ….
skills in .... a. it is good for shy children
a. entrepreneurship b. it gives good education for adults
b. community service c. interacting and communicating with
c. reading people is very important
d. drama d. it does not allow children demonstrate
e. musik excellence and develop their skills

SMA Alfa Centauri Bandung Halaman 49


MODUL PERSIAPAN UN & SBMPTN BAHASA INGGRIS 2017/ 2018

e. it is safe and makes children become c. the boarding school is the solution to our
responsible and develop talents in educational problems
leadership d. it's not necessary to send children to a
boarding school because the students
7. From the text, we can conclude that …. can live independently
a. the boarding school can be very e. not everyone thinks that the boarding
expensive school is the best educational institution
b. there are good and bad boarding for children
schools

Text 3
The following text is for questions 8 to 10.

Internet has been the most outstanding innovation in the history of mankind. As with every single
innovation, the internet has its own advantages and disadvantages.
The target of the internet has always been the communication. By the beginning of computer‘s internet,
our earth has reduced and has attained the form of a global village. Now we can communicate in a fraction of
second with a person who is sitting in the other parts of the world. Today, for better communication, we can avail
the facilities of e-mail and we can chat for hours with our loved ones. There are plenty messenger services in
offering. With the help of such services, it has become very easy to establish a kind of global friendship.
However, the internet also has disadvantages. One of them is the wide spread of unsuitable contents
for kids. It is a very serious issue and related to children‘s healthy mental life. There are thousands of such
inappropriate contents on the internet which can be easily found. Though, the internet can also create havoc,
destruction and its misuse can be very fatal, the advantages of it out-weigh its disadvantages.

8. What is one of the advantages of internet?


a. To create the misuse of time.
b. To easily find inappropriate person.
c. To chat with love ones only.
d. To connect people in the other part of the world.
e. To spread issues related to children healthy mental life.

9. What is the main idea of paragraph 2?


a. The internet could create havoc and destruction.
b. The internet could facilitate communication.
c. The internet has more advantages than disadvantages.
d. Many modern devices can be connected to the internet.
e. The internet has become the most noticeable innovation.

10. The writer believes that the internet is ….


a. very harmful
b. inappropriate
c. very useful
d. destructive
e. cheap

SMA Alfa Centauri Bandung Halaman 50


MODUL PERSIAPAN UN & SBMPTN BAHASA INGGRIS 2017/ 2018

NEWS ITEM
A. Ringkasan Materi

News item is a text which informs readers about events of the day. The events are considered newsworthy
or important.

Social function:
To inform readers, listeners or viewers about events of the day which are considered newsworthy or
important.

Generic structure:
 News worthy events: The event in summary form
 Background events: Elaborate what happened, to whom, in what circumstances.
 Sources: comments by participants in, witnesses to an authority‘s expert on the event.

Significant Lexicogrammatical Features:


 Short telegraphic information about story captured in headline.
 Use of material processes to retell the event (in the text below, many of the material processes are
nominalized)
 Use of projecting verbal processes in sources stage.
 Focus on circumstances (e.g. mostly within qualifiers)

Example of News Item text:


Newsworthy Moscow – A Russian journalist has uncovered evidence of another Soviet nuclear
event catastrophe, which killed 10 sailors and contaminated an entire town.
Background Yalena Vazshavskya is the first journalist to speak to people who witnessed the explosion of a
event nuclear submarine at the nayal base of shkejove – 2 near Vladisvostock.
The accident, which occurred 13 months before the Chernobyl disaster, spread radioactive fall
out over the base nearby town, but was covered up by officials of the Soviet Union. Residents
were told the explosion in the reactor of the Victor-class submarine during a refit had been a
―thermal‖ and not a nuclear explosion. And, those involved in the clean up operation to
remove more than 600 tones of contaminated material were sworn to secrecy.
Sources A board of investigators was later to describe it as the worst accident in the history of the
Soviet Navy.

B. Contoh Soal dan Pembahasan

PHILIPPINES: At least nine people were killed and dozens were injured when the Philippines security
forces clashed with dozens of slum dwellers who resisted the tearing down of their homes in the Northern
Province, a police commander said Tuesday.
Raul Gonzales, the police chief in the northern Cordillera area, said, that the soldiers and police officers
traded gunfire with dozens of people who are illegally occupying the private land in Kalinga province.
―Our team was ambushed on their way to the community to be demolished,‖ said Gonzales. He added
that the security only defended themselves after the residents dug foxholes and opened fire with automatic rifles.
―Nine people were killed and dozens were wounded, including 10 police officers during almost 10 hours
of fighting. We even had to evacuate some of our officers who needed surgery to get the bullets from their
bodies.‖ – Reuters

SMA Alfa Centauri Bandung Halaman 51


MODUL PERSIAPAN UN & SBMPTN BAHASA INGGRIS 2017/ 2018

1. What is the passage about? terjadi. Pada kalimat … when the Philippines
a. An illegal gunfire trade. security forces clashed with dozens of slum
b. A demolition of illegal slum dwellers. dwellers who resisted the tearing down of their
c. A fight between the police officers and the homes in the Northern Province, a police
soldiers. commander said Tuesday, bisa kita ketahui
d. A clash between the security forces and kenapa bentrokan tersebut bisa terjadi.
the slum dwellers. Jawaban: A
e. A clash between the police officers and the
security officers. 3. Raul Gonzales said that …
a. ten police officers had to be evacuated.
Pembahasan: b. ten police officers were killed in the
Paragraf 1 adalah ringkasan mengenai apa fighting.
yang terjadi, yaitu the Philippines security c. some of the wounded people needed
forces clashed with dozens of slum dwellers. surgery.
Jawaban: D d. ten police officers were injured during the
fighting.
2. The clash happened because … e. more than ten police officers were killed
a. illegal slum dwellers resisted the and injured in the fighting.
demolition of their homes.
b. the police officers and soldiers shot the Pembahasan:
dwellers. Pada paragraf keempat terdapat kalimat ―Nine
c. nine people were killed by the security people were killed and dozens were wounded,
forces. including 10 police officers during almost 10
d. the police officers firing the dwellers. hours of fighting. We even had to evacuate
e. the people dwelled in private land. some of our officers who needed surgery to get
the bullets from their bodies.” Dari kalimat
Pembahasan: tersebut bisa kita ketahui bahwa ten police
Paragraph 1 adalah ringkasan mengenai apa officers were injured during the fighting.
yang terjadi dan kenapa hal tersebut bisa Jawaban: D

C. Latihan Soal

Text 1
The following text is for questions 1 to 3.

SINGAPORE: A supervisor was jailed for two months for repeatedly striking his Indonesia maid on
the head and back with a television remote control, news report said on Thursday.
Muhamad Shafiq Woon Abdullah admitted in Singapore court. He physically abused the woman on
several occasions between June and October 2002, The Straits said.
The magistrate‘s court heard that Shafiq, 31: began striking Winarti, 22, about a month after she
started working for him.
He hit her on the head with the TV set‘s remote control because he was unhappy with her work.
On one occasion, he punched her on the back after accusing her of daydreaming.
S.S.Dhillon, Shafiq‘s lawyer, said his client had become mad when he saw his daughter‘s face
covered as she laid in bed.
He said his client thought the maid had put the child in danger. –DPA

SMA Alfa Centauri Bandung Halaman 52


MODUL PERSIAPAN UN & SBMPTN BAHASA INGGRIS 2017/ 2018

1. The text reported … c. On October 31st.


a. Shafiq, a Singaporean supervisor d. On June 22nd.
b. S.S.Dhillon, the lawyer in magistrate court e. Any time.
c. an Indonesia maid who was jailed for two
months 3. Why did Shafiq punch Winarti on her back?
d. an Indonesia maid who was working in She was accused of ….
Singapore a. taking much time for herself
e. a Singaporean supervisor who was jailed
because of striking his maid b. working carelessly

2. When did Shafiq physically abuse the woman? c. leaving her work
a. Between June and October 2002. d. daydreaming
b. On several occasion. e. being lazy

Text 2
The following text is for questions 4 to 6.
YONKERS, Nov 12th. A four alarm fire damaged 14 stores today in the Cross County Shopping
Center, the largest shopping center in Westchester County.
Two fire investigators said the blaze apparently started in a pile of cardboard cartons at the rear of a
shoe store and spread through a utilities duct above the 13 other stores. The fire started at 4.40 p.m. and was
declared under control at·6.14 p.m. The center is on the Cross County Parkway at the Gov. Thomas E. Dewey
Thruway.
Five fire-fighters were busy at the scene. Lieut. JOM Carey of the Yonkers Arson Squad said the
cause of the fire was under investigation.

4. The text mainly tells us about.... c. half an hour


a. the blaze at a shoe store d. forty five minutes
b. the Yonkers Arson Squad e. one and a half hours
c. a fire in the shopping center
d. a shopping center in the cross country 6. How many investigators and fire fighters were
e. the largest shopping center in Westchester involved in the scene?
County a. Two.
b. Seven
5. The fire has lasted about .... c. Twelve.
a. one hour d. Thirteen.
b. two hours e. Fourteen.

Text 3
The following text is for questions 7 to 10.

SINGAPORE: The Singapore government wants to double the size of the country‘s media, design and
arts industries to percent of the economy by 2012, a minister said on Saturday.
The target comes as officials have said they want the country to move away from its stuffy, rule-bound
image and embrace more creativity as it seeks ways to sustain economy growth.
―The creative industries are poised to play a significant role in transforming our economy and society,‖
Lee Boon Yang, minister for information, communication and the art, told a graduation ceremony for fine arts
students.
Singapore‘s government has set aside more than S$200 million (US $ 116 million) over five years to
invest in the arts sector, Lee said, without giving details. –AP.

SMA Alfa Centauri Bandung Halaman 53


MODUL PERSIAPAN UN & SBMPTN BAHASA INGGRIS 2017/ 2018

7. Who was the journalist interviewed?


a. Communication industries
b. The minister for information, communication and arts
c. Art community
d. Art student at a graduation ceremony
e. Art industries

8. Why does the government want to double the


size of the country‘s media, design, and art industries?
a. Because the government has set aside more than S$200 million.
b. To make the balance of the creative industries in playing a significant role in transforming economy
c. To move away from its stuffy
d. To move away from its stuffy and to make the balance of the creative industries in playing a significant
role in transforming economy
e. To poise the creative industries to play a significant role in transforming economy

9. Who is Lee Boon Yang?


a. A governor
b. minister
c. A student
d. A businessman
e. An artist

10. … as it seeks ways to sustain economic growth.


(paragraph 2). The word ‗sustain‘ means ….
a. improve
b. maintain
c. encourage
d. urge
e. repair

SMA Alfa Centauri Bandung Halaman 54


MODUL PERSIAPAN UN & SBMPTN BAHASA INGGRIS 2017/ 2018

PROCEDURAL TEXT
A. Ringkasan Materi
Ada tiga definisi "umum" mengenai procedure text: (1) Text that explain how something works
or how to use instruction / operation manuals e.g. how to use the video, the computer, the tape recorder, the
photocopier, the fax. (2) Texts that instruct how to do a particular activity e.g. recipes, rules for games,
science experiments, road safety rules. (3) Texts that deal with human behaviour eg how to live happily, how
to succeed.
Dari keterangan di atas, dapat kita garis bawahi bahwa procedure text adalah (1) Teks yang
menjelaskan bagaimana sesuatu bekerja atau teks yang menjelaskan cara menggunakan pedoman
instruksi / penggunaan. contoh : cara menggunakan video, komputer, mesin fotokopi, fax dll. (2) Teks yang
menunjukan cara melakukan aktifitas tertentu. contoh : resep, aturan bermain game, eksperimen ilmiah,
aturan keamanan berkendara. (3) Teks yang berhubungan dengan tingkah laku manusia. contoh : cara
hidup bahagian, cara sukses. dll..
The purpose procedural text is to tell the reader how to do or make something. The information
is presented in a logical sequence of events which is broken up into small sequenced steps. These texts are
usually written in the present tense. The most common example of a procedural text is a recipe.
Tujuan procedure text adalah memberitahu pembaca cara melakukan / membuat sesuatu.
Informasi disajikan dengan urutan peristiwa yang logis. Peristiwa tersebut biasanya dibagi menjadi beberapa
langkah-langkah terpisah. Teks ini biasanya ditulis menggunakan present tense. Contoh paling umum
procedure text adalah resep masakan.

Generic Structure:
Seperti halnya pengertian procedure text di atas, generic structure (susunan umum) procedure text juga ada
tiga :
1) Goal (Maksud atau tujuan)
2) Material Needed (Materi / alat / bahan yang dibutuhkan)
3) Methods or Steps (Metode / langkah-langkah)

Language Features:
 Menggunakan pola kalimat imperative (perintah), misalnya, cut the carrots into pieces, put the sugar on
the cup, boil the water, etc.
 Menggunakan action verbs, misalnya cut, put, boil, etc.
 Menggunakan conjunctions (kata penghubung) untuk mengurutkan kegiatan, misalnya then, while, dsb.
 Menggunakan adverbs (kata keterangan) untuk menyatakan rinci waktu, tempat, cara yang akurat,
misalnya for five minutes, 2 hours, dsb.
 Menggunakan adverbs (kata keterangan) untuk menyatakan tahapan atau urutan, misalkan the first, the
second, the third, the last, etc.
 Menggunakan simple present.

SMA Alfa Centauri Bandung Halaman 55


MODUL PERSIAPAN UN & SBMPTN BAHASA INGGRIS 2017/ 2018

B. Latihan Soal

1. Arrange the sentences below into a good order!

How to start a scrapbook


1. Organize the photos and mementos.
2. Decide on a theme for your photo album. It can be specific or general.
3. Group similar items onto pages. Sort and group picture into cohesive units.
4. Choose an album. In addition to the patterns and designs, think about the size.
5. Decorate each page. Stickers, ribbon, chipboard, pieces, and stamps are useful for accenting photos.
a. 2–3–4–1–5
b. 2–3–4–5–1
c. 3–4–5–2–1
d. 1–2–3–4–5
e. 3–2–1–5–4

2. Arrange the sentences into a meaningful paragraph.


1) If you are using one of the allocated car parks, make sure you don‘t leave your special car pass and ID
behind.
2) Carry everything you need with you. It is a good idea to make a list of what is required so you don‘t
forget anything.
3) First, give yourself plenty of time. Leave early to avoid traffic jams or long queues at the car parking
areas.
4) Carry your ID and ticket as these must be shown when you enter the gates.
5) Or, if you are travelling by public transport, leave early and have your route mapped out before you
leave.

The best arrangement for the sentences is ….


A. 3 – 2 – 5 – 4 – 1
B. 3 – 2 – 5 – 1 – 4
C. 3 – 2 – 4 – 5 – 1
D. 3 – 2 – 1 – 5 – 4
E. 3 – 2 – 4 – 1 – 5

SMA Alfa Centauri Bandung Halaman 56


MODUL PERSIAPAN UN & SBMPTN BAHASA INGGRIS 2017/ 2018

REVIEW TEXT
A. Ringkasan Materi

Review text is a text that presents a critical evaluation of a text, performance, or production (for
example, a book, movie, concert, or video game). This text is to criticize an art work, event for a public
audience.

Generic Structure:
 Orientation : places the work in its general and particular context, often by comparing it with others of
its kind or through an analog with a non–art object or event. (menempatkan karya yang ditinjau pada
konteks umum ataupun khusus, biasanya dengan membandingkan dengan karya lain yang sejenis atau
melalui analogi obyek yang bukan karya seni.)
 Interpretive Recount : summarize the plot and/or providers an account of how the reviewed rendition
of the work came into being.(meringkas alur cerita "jika mereview buku" bagaimana cara penyampaian
karya tersebut)
 Evaluation : provides an evaluation of the work and/or its performance or production; is usually
recursive.(memberikan sebuah evaluasi karya ataupun penampilan, produksi; evaluasi ini biasanya
berulang-ulang)

B. Contoh Soal dan Pembahasan

Laskar Pelangi: The Audacity of Hope

This is a movie adapted from a best-selling Indonesian novel. It took 40 days of filming on Belitung
Island, Bangka-Belitung province. Involving 12 local actors, it reportedly cost 8 billion rupiahs. With all the
efforts of transforming Laskar Pelangi (Rainbow Warrior) into a moving picture, will it satisfy readers‘
imaginations?
Laskar Pelangi, the novel, was written by Andre Hirata in 2005, based on his own experiences. It is
about an inspiring teacher and her 10 students in the poverty-stricken Kampung Gantong in Belitung. The
poor condition of their school building does not dampen their high spirits and hopes for a better future.
Two years later, the novel became a phenomenon in Indonesian literature. With its humanistic touch,
Laskar Pelangi has sold more than 500,000 copies and has won the position of Must-Read Novel in every
corner of the nation‘s bookshops and media review pages. It has finally overcome the domination of teen-lit,
chick-lit and even religious novels, the popular theme of today.
Andrea entrusted the filming of the story to the respected figures in the film industry, Mira Lesmana and
Riza, as producer and film director. In July 2007, Mira and Riri started the pre-production, which took a year
to finish. Together with the scriptwriter Salman Aristo they decided to create a different scenario for the film.

1. What is the main idea of the second Jawaban: D


paragraph?
a. Laskar Pelangi is the story of poor people 2. From the text above, we know that ….
at the Bangka Belitung province. a. The movie was played by poor children
b. Laskar Pelangi is the story of poor school b. Andre Hirata is the producer of Laskar
children of the Belitung Island. Pelangi
c. Laskar Pelangi is about poverty in c. The novel of Laskar Pelangi was a
Kampung Gantong in Bangka Belitung. phenomenal work of literature
d. Laskar Pelangi is about an inspiring, story d. People like the movie better than the novel
towards a better life. e. People like the novel better than the movie
e. Laskar Pelangi is about Andrea Hirata. Pembahasan:
Jelas terdapat pada paragraph ketiga baris
Pembahasan: kesatu, yaitu … the novel became a
Jelas terdapat pada kalimat kedua … it is phenomenon in Indonesian literature.
about an inspiring teacher and …. Jawaban: E

SMA Alfa Centauri Bandung Halaman 57


MODUL PERSIAPAN UN & SBMPTN BAHASA INGGRIS 2017/ 2018

C. Latihan Soal

Text 1
The following text is for question 1 to 5.

The Halloween Kid


Burgeoning filmmaker Axelle Carolyn showed a lyrical, controlled sense of cinema in her 2011 short film
debut THE LAST POST. Now, the sometime actress, model and author have created another work of gentle,
frightening craft in THE HALLOWEEN KID.
The short film features a rhyming, Edward Gorey-esque poem (penned by Carolyn) as its script and
story, a tale of a poor, put – upon little boy named Henry who feels more akin to the supernatural creatures
of his imagination than the cruel, narrow minded adults and children who permeates his world. But of course,
Henry might not be as delusional as we think and, come Halloween, he found his only moments of respite
amongst other kids, who are in fact, dead!
That‘s pretty much it as far as THE HALLOWEEN KID‘s narrative goes, but like THE LAST POST, the
film is not about story, rather creating a tiny, perfectly realized universe that aims for more sensitivity than
shock, more melancholy than morbid. The music is lovely and whimsical, as in the almost Boris Karloff like
narration by legendary Shakespearean dramatic Derek Jacobi.
THE HALLOWEEN KID shows even more promise from Carolyn, who with this beautifully sculpted
piece of work is one to watch, not just as a ‗horror filmmaker‘ but as a quality director of movies, full stop.

1. The writer of script and story in the Halloween e. a talented model and author
Kid is ….
a. Axelle Carolyn 4. Now, the sometime actress, model and author
b. Edward Gorey have created another work of gentle,
c. Derek Jacoby frightening craft in THE HALLOWEEN KID. The
d. Shakespeare underlined can also be replaced by ….
e. Henry a. fear
b. happy
2. According to the reviewer, the movie was …. c. glad
a. lovely d. satisfied
b. morbid e. attractive
c. shocking
d. frightening 5. Now, the sometime actress, model and author
e. melancholy have created another work of gentle,
frightening craft in THE HALLOWEEN KID. The
3. Based on the text, the film has proven that underlined can also be replaced by ….
Axelle Carolyn is …. a. produced
a. a popular horror filmmaker b. finished
b. a beautiful sculptor c. accumulated
c. a qualified movie director d. appointed
d. a talented actress e. shared

SMA Alfa Centauri Bandung Halaman 58


MODUL PERSIAPAN UN & SBMPTN BAHASA INGGRIS 2017/ 2018

Text 2
The following text is for questions 6 to 7.

Music and Lyrics Movie Review

Alex Fletcher (Hugh Grant) is a washed-up 80‘s pop star who‘s been reduced to working the nostalgia
circuit at country fairs and amusement parks. The charismatic and talented musician gets a chance at a
comeback when reigning diva Cora Corman (Haley Bennet) invites him to write and record a duet with her, but
there‘s a problem – Alex hasn‘t written a song in years, he‘s never written lyrics, and he has to come up with a hit
in a matter of days, enter Sophie Fisher (Drew Barrymore), Alex‘s attractively quirky lady, whose flair for words
strikes a chord with the struggling songwriter on the rebound from a bad relationship.
Sophie is reluctant to collaborate with anyone, especially commitment-phobe Alex. As their chemistry
heats up at the piano and under it, Alex and Sophie will have to face their fears-and the music- if they want to
find the love and success they both deserve.
Adapted from: http://musicandlyrics.warnerbros.com

6. What does the review present?


a. The reviewer‘s assessment.
b. The reviewer‘s speculation.
c. The movie‘s prediction.
d. The movie‘s synopsis.
e. The movie‘s rating.

7. The word ―quirky‖ (line 7) has the closest meaning with ….


a. Odd
b. Pretty
c. Beautiful
d. Eye-catching
e. Good looking

SMA Alfa Centauri Bandung Halaman 59


MODUL PERSIAPAN UN & SBMPTN BAHASA INGGRIS 2017/ 2018

QUESTIONS ABOUT THE IDEAS OF THE PASSAGE


Skill 1: Answer main idea questions correctly

SMA Alfa Centauri Bandung Halaman 60


MODUL PERSIAPAN UN & SBMPTN BAHASA INGGRIS 2017/ 2018

SMA Alfa Centauri Bandung Halaman 61


MODUL PERSIAPAN UN & SBMPTN BAHASA INGGRIS 2017/ 2018

SMA Alfa Centauri Bandung Halaman 62


MODUL PERSIAPAN UN & SBMPTN BAHASA INGGRIS 2017/ 2018

Skill 2: Recognize Organization of ideas

SMA Alfa Centauri Bandung Halaman 63


MODUL PERSIAPAN UN & SBMPTN BAHASA INGGRIS 2017/ 2018

SMA Alfa Centauri Bandung Halaman 64


MODUL PERSIAPAN UN & SBMPTN BAHASA INGGRIS 2017/ 2018

SMA Alfa Centauri Bandung Halaman 65


MODUL PERSIAPAN UN & SBMPTN BAHASA INGGRIS 2017/ 2018

SMA Alfa Centauri Bandung Halaman 66


MODUL PERSIAPAN UN & SBMPTN BAHASA INGGRIS 2017/ 2018

DIRECTLY ANSWERED QUESTIONS


Skill 3: Answer stated detail questions correctly

SMA Alfa Centauri Bandung Halaman 67


MODUL PERSIAPAN UN & SBMPTN BAHASA INGGRIS 2017/ 2018

SMA Alfa Centauri Bandung Halaman 68


MODUL PERSIAPAN UN & SBMPTN BAHASA INGGRIS 2017/ 2018

Skill 4: Find ”unstated” details

SMA Alfa Centauri Bandung Halaman 69


MODUL PERSIAPAN UN & SBMPTN BAHASA INGGRIS 2017/ 2018

SMA Alfa Centauri Bandung Halaman 70


MODUL PERSIAPAN UN & SBMPTN BAHASA INGGRIS 2017/ 2018

Skill 5: Find pronoun refferents

SMA Alfa Centauri Bandung Halaman 71


MODUL PERSIAPAN UN & SBMPTN BAHASA INGGRIS 2017/ 2018

SMA Alfa Centauri Bandung Halaman 72


MODUL PERSIAPAN UN & SBMPTN BAHASA INGGRIS 2017/ 2018

SMA Alfa Centauri Bandung Halaman 73


MODUL PERSIAPAN UN & SBMPTN BAHASA INGGRIS 2017/ 2018

6. Which of the following would be the 7. According to the passage, which of


best title for the passage? the following helps to explain why the
term “hay fever” is somewhat of a
A. The relationship between Season misnomer?
and Allergies.
B. Misconceptions and Facts about A. A strong fever occurs after an
Hay Fever. attack.
C. Hay Fever in the Eastern United B. The amount of pollen in the air
States. depends on geographical location.
D. How Ragweed Causes Hay Fever. C. Hay fever is often caused by
ragweed pollen.

SMA Alfa Centauri Bandung Halaman 74


MODUL PERSIAPAN UN & SBMPTN BAHASA INGGRIS 2017/ 2018

D. Grass pollen is prevalent in June 10. Choose a picture that represents a


and July. potential cause of hay fever!

8. Which of the following is NOT true


about hay fever in the Eastern United
States?

A. Suffering from hay fever is equally


severe year-round.
B. Pollen from trees causes hay fever
suffering in the spring.
C. Grass pollen fills the air earlier in
the year than ragweed pollen.
D. Ragweed pollen is most prevalent
at the end of the summer.

9. Which of the following is NOT a


11. Which of the following is NOT
symptom of hay fever?
discussed in the passage as a
A. A high fever. determining factor of the amount of
B. A runny nose. pollen in the air?
C. Red eyes.
D. Persistent sneezing. A. Place.
B. Climate.
C. Time of year.
D. Altitude.

SMA Alfa Centauri Bandung Halaman 75


MODUL PERSIAPAN UN & SBMPTN BAHASA INGGRIS 2017/ 2018

INDIRECTLY ANSWERED QUESTIONS


Skill 6: Answer implied detail question correctly

SMA Alfa Centauri Bandung Halaman 76


MODUL PERSIAPAN UN & SBMPTN BAHASA INGGRIS 2017/ 2018

SMA Alfa Centauri Bandung Halaman 77


MODUL PERSIAPAN UN & SBMPTN BAHASA INGGRIS 2017/ 2018

Skill 7: Answer transition questions correctly

SMA Alfa Centauri Bandung Halaman 78


MODUL PERSIAPAN UN & SBMPTN BAHASA INGGRIS 2017/ 2018

SMA Alfa Centauri Bandung Halaman 79


MODUL PERSIAPAN UN & SBMPTN BAHASA INGGRIS 2017/ 2018

SMA Alfa Centauri Bandung Halaman 80


MODUL PERSIAPAN UN & SBMPTN BAHASA INGGRIS 2017/ 2018

SMA Alfa Centauri Bandung Halaman 81


MODUL PERSIAPAN UN & SBMPTN BAHASA INGGRIS 2017/ 2018

SMA Alfa Centauri Bandung Halaman 82


MODUL PERSIAPAN UN & SBMPTN BAHASA INGGRIS 2017/ 2018

VOCABULARY QUESTIONS
Skill 8: Find definition from structural clues, determine meanings from word parts, use context to
determine meanings of difficult words

SMA Alfa Centauri Bandung Halaman 83


MODUL PERSIAPAN UN & SBMPTN BAHASA INGGRIS 2017/ 2018

SMA Alfa Centauri Bandung Halaman 84


MODUL PERSIAPAN UN & SBMPTN BAHASA INGGRIS 2017/ 2018

SMA Alfa Centauri Bandung Halaman 85


MODUL PERSIAPAN UN & SBMPTN BAHASA INGGRIS 2017/ 2018

Skill 9: Use context to determine meanings of simple words

SMA Alfa Centauri Bandung Halaman 86


MODUL PERSIAPAN UN & SBMPTN BAHASA INGGRIS 2017/ 2018

TOEFL EXERCISE (Skills 8-9)


Study each of the passages and choose the best answers to the questions that follow.

SMA Alfa Centauri Bandung Halaman 87


MODUL PERSIAPAN UN & SBMPTN BAHASA INGGRIS 2017/ 2018

TOEFL REVIEW EXERCISE (Skills 1-9)


Study each of the passages and choose the best answer to the questions that follow.

SMA Alfa Centauri Bandung Halaman 88


MODUL PERSIAPAN UN & SBMPTN BAHASA INGGRIS 2017/ 2018

SMA Alfa Centauri Bandung Halaman 89


MODUL PERSIAPAN UN & SBMPTN BAHASA INGGRIS 2017/ 2018

OVERALL REVIEW QUESTIONS


Skill 10: Determine where specific information is found

SMA Alfa Centauri Bandung Halaman 90


MODUL PERSIAPAN UN & SBMPTN BAHASA INGGRIS 2017/ 2018

Skill 11: Determine the tone, purpose, or course

SMA Alfa Centauri Bandung Halaman 91


MODUL PERSIAPAN UN & SBMPTN BAHASA INGGRIS 2017/ 2018

SMA Alfa Centauri Bandung Halaman 92


MODUL PERSIAPAN UN & SBMPTN BAHASA INGGRIS 2017/ 2018

TOEFL EXERCISE (SKILLS 10 - 11):


Study each of the passafes and choose the best answers to the questions that follow

SMA Alfa Centauri Bandung Halaman 93


MODUL PERSIAPAN UN & SBMPTN BAHASA INGGRIS 2017/ 2018

TOEFL REVIEW EXERCISE (Skills 1 - 11):


Study each of the passages and choose the best answers to the questions that follow

SMA Alfa Centauri Bandung Halaman 94


MODUL PERSIAPAN UN & SBMPTN BAHASA INGGRIS 2017/ 2018

SMA Alfa Centauri Bandung Halaman 95


MODUL PERSIAPAN UN & SBMPTN BAHASA INGGRIS 2017/ 2018

SMA Alfa Centauri Bandung Halaman 96


MODUL PERSIAPAN UN & SBMPTN BAHASA INGGRIS 2017/ 2018

PART OF SPEECH

1. NOUN
This part of a speech refers to words that are used to name persons, things, animals, places, ideas, or
events.
Examples:
 Tom Hanks is very versatile.
The italicized noun refers to a name of a person.
 Dogs can be extremely cute.
In this example, the italicized word is considered a noun because it names an animal.
 It is my birthday.
The word ―birthday‖ is a noun which refers to an event.

There are different types of nouns namely:


 Proper
Proper nouns always start with a capital letter and refer to specific names of persons, places, or things.
Examples: Volkswagen Beetle, Shakey‘s Pizza, Game of Thrones
 Common
Common nouns are the opposite of proper nouns. These are just generic names of persons, things, or
places.
Examples: car, pizza parlor, TV series

SMA Alfa Centauri Bandung Halaman 97


MODUL PERSIAPAN UN & SBMPTN BAHASA INGGRIS 2017/ 2018

 Concrete
This kind refers to nouns which you can perceive through your five senses.
Examples: folder, sand, board
 Abstract
Unlike concrete nouns, abstract nouns are those which you can‘t perceive through your five senses.
Examples: happiness, grudge, bravery
 Count
It refers to anything that is countable, and has a singular and plural form.
Examples: kitten, video, ball
 Mass
This is the opposite of count nouns. Mass nouns are also called non-countable nouns, and they need to
have ―counters‖ to quantify them.
Examples of Counters: kilo, cup, meter
Examples of Mass Nouns: rice, flour, garter
 Collective
It refers to a group of persons, animals, or things.
Example: faculty (group of teachers), class (group of students), pride (group of lions)

2. PRONOUN
A pronoun is a part of a speech which functions as a replacement for a noun. Some examples of pronouns
are: I, it, he, she, mine, his, hers, we, they, theirs, and ours.

Sample Sentences:
 Janice is a very stubborn child. She just stared at me when I told her to stop.
 The largest slice is mine.
 We are number one.

The italicized words in the sentences above are the pronouns in the sentence.

3. ADJECTIVE
This part of a speech is used to describe a noun or a pronoun. Adjectives can specify the quality, the size,
and the number of nouns or pronouns.
Sample Sentences:
 The carvings are intricate.
The italicized word describes the appearance of the noun ―carvings.‖
 I have two hamsters.
The italicized word ―two,‖ is an adjective which describes the number of the noun ―hamsters.‖
 Wow! That doughnut is huge!
The italicized word is an adjective which describes the size of the noun ―doughnut.‖

SMA Alfa Centauri Bandung Halaman 98


MODUL PERSIAPAN UN & SBMPTN BAHASA INGGRIS 2017/ 2018

When a number of adjectives are used together, the order depends on the function of the adjective. The
usual order is:
Opinion, Size, Age, Shape, Colour, Origin, Material, purpose
An opinion adjective explains what you think about something (other people may not agree with
you).
Opinion
For example: silly, beautiful, horrible, difficult
A size adjective, of course, tells you how big or small something is.
Size
For example: large, tiny, enormous, little
An age adjective tells you how young or old something or someone is.
Age
For example: ancient, new, young, old
A shape adjective describes the shape of something.
Shape
For example: square, round, flat, rectangular
A colour adjective, of course, describes the colour of something.
Colour
For example: blue, pink, reddish, grey
An origin adjective describes where something comes from.
Origin
For example: French, lunar, American, eastern, Greek
A material adjective describes what something is made from.
Material
For example: wooden, metal, cotton, paper
A purpose adjective describes what something is used for. These adjectives often end with ―-ing‖.
Purpose
For example: sleeping (as in ―sleeping bag‖), roasting (as in ―roasting tin‖)

Before adjectives + noun we normally have a determiner: a, an, the, my, your, her, four, this, those, some,
etc.
Some examples of adjective order
Opinion Size Age Shape Colour Origin Material Purpose
A Silly young English man
A Huge round Metal bowl
A Small red sleeping bag

4. VERB
This is the most important part of a speech, for without a verb, a sentence would not exist. Simply put, this is
a word that shows an action (physical or mental) or state of being of the subject in a sentence.
Examples of ―State of Being Verbs‖ : am, is, was, are, and were
Sample Sentences:
 As usual, the Stormtroopers missed their shot.
 The italicized word expresses the action of the subject ―Stormtroopers.‖
 They are always prepared in emergencies.
 The verb ―are‖ refers to the state of being of the pronoun ―they,‖ which is the subject in the sentence.

Types of verbs
 Action verbs
Action verbs are words that express action (give, eat, walk, etc.) or possession (have, own, etc.).
Action verbs can be either transitive or intransitive.

Transitive verbs
A transitive verb always has a noun that receives the action of the verb, called the direct object.
Example:
Laurissa raises her hand.

SMA Alfa Centauri Bandung Halaman 99


MODUL PERSIAPAN UN & SBMPTN BAHASA INGGRIS 2017/ 2018

The verb is raises. Her hand is the object receiving the verb‘s action. Therefore, raises is a
transitive verb.

Transitive verbs sometimes have indirect objects, which name the object to whom or for whom the
action was done.
Example:
Abdullah gave Becky the pencil.
The verb is gave. The direct object is the pencil. (What did he give? The pencil.)
The indirect object is Becky. (To whom did he give it? To Becky.)

Intransitive verbs
An intransitive verb never has a direct or indirect object. Although an intransitive verb may be
followed by an adverb or adverbial phrase, there is no object to receive its action.
Example:
Laurissa rises slowly from her seat.
The verb is rises. The phrase, slowly from her seat, modifies the verb, but no object receives the
action.

 Linking verbs
A linking verb connects the subject of a sentence to a noun or adjective that renames or describes
the subject. This noun or adjective is called the subject complement.
Example:
Jason became a business major.
The verb (became) links the subject (Jason) to its complement (a business major).

Lisa is in love with Jason.


The verb (is) links the subject (Lisa) to the subject complement (in love with Jason) describing
Lisa.

The most common linking verb is the verb to be in all of its forms (am, are, is, was, were, etc.). This
verb may also be used as a helping verb. To become and to seem are always linking verbs.

Other verbs may be linking verbs in some cases and action verbs in others:
to appear to feel to look to remain to stay to taste
to continue to grow to prove to sound to smell to turn

Linking : Libby appeared happy. (Appeared links Libby to the subject complement, happy).
Action : Deon suddenly appeared. (Here, appeared is an intransitive action verb).

 Helping verbs
Helping verbs are used before action or linking verbs to convey additional information regarding
aspects of possibility (can, could, etc.) or time (was, did, has, etc.). The main verb with its
accompanying helping verb is called a verb phrase.

Examples:
Teja is (helping verb) going (main verb) to Florida.
The trip might (helping verb) be (main verb) dangerous.

The following words, called modals, always function as helping verbs:


can May must shall will
could might ought to should would

Examples:
Tanya could learn to fly helicopters. (Could helps the main verb, learn).
Jasmine will drive to Idaho tomorrow. (Will helps the main verb, drive).

SMA Alfa Centauri Bandung Halaman 100


MODUL PERSIAPAN UN & SBMPTN BAHASA INGGRIS 2017/ 2018

In addition, the following forms of the verbs to be, to do, and to have sometimes serve as helping
verbs. (Note: in other cases, they may serve as action or linking verbs).
am Be Being do had have was
are Been Did does has is were

Helping : Jana is moving to a new house.


Linking : Jana is ready to go.

Helping : Dustin did eat his vegetables.


Linking : Dustin did his homework last night. (transitive verb)

5. ADVERB
Just like adjectives, adverbs are also used to describe words, but the difference is that adverbs describe
adjectives, verbs, or another adverb.

The different types of adverbs are:


 Adverb of Manner– this refers to how something happens or how an action is done.
Example: Annie danced gracefully.
The word ―gracefully‖ tells how Annie danced.
 Adverb of Time- this states ―when‖ something happens or ―when‖ it is done.
Example: She came yesterday.
The italicized word tells when she ―came.‖
 Adverb of Place– this tells something about ―where‖ something happens or ‖where‖ something is done.
Example: Of course, I looked everywhere!
The adverb ―everywhere‖ tells where I ―looked.‖
 Adverb of Degree– this states the intensity or the degree to which a specific thing happens or is done.
Example: The child is very talented.
The italicized adverb answers the question, ―To what degree is the child talented?‖

6. PREPOSITION
This part of a speech basically refers to words that specify location or a location in time.
Examples of Prepositions: above, below, throughout, outside, before, near, and since

Sample Sentences:
 Micah is hiding under the bed.
 The italicized preposition introduces the prepositional phrase ―under the bed,‖ and tells where Micah is
hiding.
 During the game, the audience never stopped cheering for their team.
 The italicized preposition introduces the prepositional phrase ―during the game,‖ and tells when the
audience cheered.

SMA Alfa Centauri Bandung Halaman 101


MODUL PERSIAPAN UN & SBMPTN BAHASA INGGRIS 2017/ 2018

Types of preposition:

7. CONJUNCTION
The conjunction is a part of a speech which joins words, phrases, or clauses together.
Examples of Conjunctions: and, yet, but, for, nor, or, and so
Sample Sentences:
 This cup of tea is delicious and very soothing.
 Kiyoko has to start all over again because she didn‘t follow the professor‘s instructions.
 Homer always wanted to join the play, but he didn‘t have the guts to audition.
The italicized words in the sentences above are some examples of conjunctions.

SMA Alfa Centauri Bandung Halaman 102


MODUL PERSIAPAN UN & SBMPTN BAHASA INGGRIS 2017/ 2018

8. INTERJECTION
This part of a speech refers to words which express emotions. Since interjections are commonly used to
convey strong emotions, they are usually followed by an exclamation point.
Examples of Interjections:

Sample Sentences:
 Ouch! That must have hurt.
 Hurray, we won!
 Hey! I said enough!
The bold words attached to the main sentences above are some examples of interjections.

SMA Alfa Centauri Bandung Halaman 103


MODUL PERSIAPAN UN & SBMPTN BAHASA INGGRIS 2017/ 2018

EXERCISE:

A. Look at the word in bold. Which part of speech is it?

1. Remember to drive safely. Part of speech:


9. New York is a big city. Part of speech:
10. I want the blue ball. Part of speech:
2. Don't talk like that. Part of speech:
3. Sally and Tom are coming back soon. Part of 11. We finished it yesterday. Part of speech:
speech:
4. This is a pretty vase. Part of speech: 12. What am I going to do without you? Part of
speech:
5. Let's do it. Part of speech: 13. He will come. Part of speech:
6. The cat is on the table. Part of speech: 14. Wow, those are great news! Part of speech:

7. Oh, I didn't know that. Part of speech: 15. We have pens, pencil, notebooks and markers.
Part of speech:
8. Do you want to go out or stay home? Part of 16. They went into the hall. Part of speech:
speech:

B. Choose the correct answer!

1. She lived in Surabaya during her .... 6. She has made an … discovery in science.
a. childish d. childhood a. important d. import
b. childlike e. child b. importance e. unimportant
c. chilldhoodness c. importancy

2. It is … that he doesn‘t know anything about 7. His explanation was quite ….


Maths. a. clear d. cleared
a. evident d. eviden b. clearly e. clearing
b. evidently e. evidential c. clearness
c. evidence
8. She is a professional ….
3. The … driver caused the accident. a. beautiful d. beautician
a. careful d. carefully b. beautifully e. beauty
b. careless e. carelessly c. beautify
c. carelessness
9. She gets … to the party.
4. The … of his family makes him proud. a. onvite
a. popular d. popularly b. invited
b. popularity e. popularitazion c. invitation
c. popularition d. invited
e. invitational
5. This machine will measure the … of the lake.
a. deep d. deeply 10. If you want to … in final examination, you
b. deepest e. deeper should study hard from now.
c. depth a. successful
b. success
c. succeed
d. successfully
e. succeeded

SMA Alfa Centauri Bandung Halaman 104


MODUL PERSIAPAN UN & SBMPTN BAHASA INGGRIS 2017/ 2018

TENSES

EXERCISE:

1. When you reach the top of the hill, you … a 4. By the end of this week, he will have finished
beautiful panorama. reading the novel. It means ….
A. are seeing A. He has finished reading the novel
B. is seeing B. He may have finished reading the novel
C. seeing C. He was reading the novel
D. will see D. He will not finish reading the novel
E. will be seeing E. He has not finished reading the novel and
is going to finish reading it this week.
2. My mother … buy me shoes next Sunday.
A. will 5. Every night the watchman turns on all the lights
B. will be and … around the building every half an hour.
C. will have A. walks
D. have B. is walking
E. is buying C. to be walking
D. walking
3. I wonder what I … this time next year. E. to walk
A. will be doing
B. am doing
C. have done
D. will have done
E. do

SMA Alfa Centauri Bandung Halaman 105


MODUL PERSIAPAN UN & SBMPTN BAHASA INGGRIS 2017/ 2018

6. ―I had never seen a panda bear until I visited A. had learned


the London zoo‖. This means ….
A. I didn‘t go to the London zoo after I saw a B. was learning
panda bear C. will have learning
B. I visited the London zoo after I saw a D. would learn
panda bear E. has learned
C. I saw panda bear for the first time in the
London zoo 9. Suryo had already been working in that
D. I didn‘t see a panda bear when I visited the company for two years when I started working
London zoo there. It means ….
E. When I entered London zoo, I immediately A. Suryo wasn‘t working in that company
saw a panda bear anymore
B. Suryo would be working for two more
7. I didn‘t hear the thunder during the storm last years
night because I … soundly. C. Suryo stopped working in that company
A. have been sleeping D. Suryo worked in that company only for two
B. had slept years
C. am sleeping E. Suryo was still working in that company
D. was sleeping
E. have slept 10. We couldn‘t get a room at the beach Hotel
because we … reservation.
8. Harry started learning to play chess when he A. would not make
was only five years old and won his first B. had not make
national chess competition when he was ten. C. do not make
This means that Harry … to play chess very D. were not making
well by the time he was 10 years old. E. have not make

SMA Alfa Centauri Bandung Halaman 106


MODUL PERSIAPAN UN & SBMPTN BAHASA INGGRIS 2017/ 2018

PASSIVE
Passive voice is used when the focus is on the action. It is not important or not known, however, who or what is
performing the action.
Example: My bike was stolen.

In the example above, the focus is on the fact that my bike was stolen. I do not know, however, who did it.
Sometimes a statement in passive is more polite than active voice, as the following example shows:
Example: A mistake was made.
In this case, I focus on the fact that a mistake was made, but I do not blame anyone (e.g. You have made a
mistake.).

Form of Passive
Subject + finite form of to be + Past Participle (3rd column of irregular verbs)
Example: A letter was written.
When rewriting active sentences in passive voice, note the following:
 the object of the active sentence becomes the subject of the passive sentence
 the finite form of the verb is changed (to be + past participle)
 the subject of the active sentence becomes the object of the passive sentence (or is dropped)

Active/ Passive Overview

Tense Active Passive

Simple Present Once a week, Tom cleans the house. Once a week, the house is cleaned by Tom.

Present
Right now, Sarah is writing the letter. Right now, the letter is being written by Sarah.
Continuous

Simple Past Sam repaired the car. The car was repaired by Sam.

The salesman was helping the customer The customer was being helped by the
Past Continuous
when the thief came into the store. salesman when the thief came into the store.

Present Perfect Many tourists have visited that castle. That castle has been visited by many tourists.

Present Perfect Recently, the work has been being done by


Recently, John has been doing the work.
Continuous John.

George had repaired many cars before he Many cars had been repaired by George
Past Perfect
received his mechanic's license. before he received his mechanic's license.

Chef Jones had been preparing the The restaurant's fantastic dinners had been
Past Perfect
restaurant's fantastic dinners for two years being prepared by Chef Jones for two years
Continuous
before he moved to Paris. before he moved to Paris.

Simple Future
Someone will finish the work by 5:00 PM. The work will be finished by 5:00 PM.
will

SMA Alfa Centauri Bandung Halaman 107


MODUL PERSIAPAN UN & SBMPTN BAHASA INGGRIS 2017/ 2018

Simple Future Sally is going to make a beautiful dinner A beautiful dinner is going to be made by Sally
be going to tonight. tonight.

Future
At 8:00 PM tonight, John will be washing At 8:00 PM tonight, the dishes will be being
Continuous
the dishes. washed by John.
will

Future
At 8:00 PM tonight, John is going to be At 8:00 PM tonight, the dishes are going to be
Continuous
washing the dishes. being washed by John.
be going to

Future Perfect They will have completed the project before The project will have been completed before
will the deadline. the deadline.

Future Perfect They are going to have completed the The project is going to have been completed
be going to project before the deadline. before the deadline.

Future Perfect The famous artist will have been painting The mural will have been being painted by the
Continuous the mural for over six months by the time it is famous artist for over six months by the time it is
will finished. finished.

Future Perfect The famous artist is going to have been The mural is going to have been being
Continuous painting the mural for over six months by painted by the famous artist for over six months
be going to the time it is finished. by the time it is finished.

Used to Jerry used to pay the bills. The bills used to be paid by Jerry.

Would Always My mother would always make the pies. The pies would always be made by my mother.

Future in the
I knew John would finish the work by 5:00
Past I knew the work would be finished by 5:00 PM.
PM.
Would

Future in the
I thought Sally was going to make a I thought a beautiful dinner was going to be
Past
beautiful dinner tonight. made by Sally tonight.
Was Going to

SMA Alfa Centauri Bandung Halaman 108


MODUL PERSIAPAN UN & SBMPTN BAHASA INGGRIS 2017/ 2018

Passive Sentences with Two Objects


Rewriting an active sentence with two objects in passive voice means that one of the two objects becomes the
subject, the other one remains an object. Which object to transform into a subject depends on what you want to
put the focus on.

Subject Verb Object 1 Object 2

Active: Rita Wrote a letter to me.

Passive: A letter was written to me by Rita.

Passive: I was written a letter by Rita.

As you can see in the examples, adding by Rita does not sound very elegant. That‘s why it is usually dropped.

EXERCISE:

I. Change these sentences into passive voice.

No. Active Passive

1 The boy writes poems every evening.

2 The girl drove a blue car.

3 They have collected enough money.

4 They will open a new restaurant.

5 The little boy can draw pictures.

6 Her friend gave her a book.

7 The man showed us the house.

8 They offered him a job.

9 She called me when I was doing my


homework.

10 I have to submit the work tomorrow.

SMA Alfa Centauri Bandung Halaman 109


MODUL PERSIAPAN UN & SBMPTN BAHASA INGGRIS 2017/ 2018

MODAL VERBS

SMA Alfa Centauri Bandung Halaman 110


MODUL PERSIAPAN UN & SBMPTN BAHASA INGGRIS 2017/ 2018

EXERCISE:

I. Complete the following sentences using can/may/must/should. Sometimes you need the negatives.

1. My father is very smart. He __________ speak five languages.


2. Look! The girl is falling into the river. We __________rescue her now.
3. Tom gets failed in his exams. I think he __________ study harder.
4. There is no boy in the room now. They __________ go out to buy something.
5. If you don‘t bring your dictionary, you __________ borrow mine now.
6. You __________ touch the wall. My father has just painted it.
7. You __________ care about her since she loves you very much.
8. John is absent from school today. He __________be sick.
9. The bus no 1 has just left. We ____________ wait another later.
10. Sorry, I am very busy now. I ____________ help you.

II. Give your responses or opinions to the following situations with the suitable modal auxiliaries. Use
your own words to express your ideas.

1. You want to go or study abroad. What must you do?

2. Your father gives you a lot of money. What can you do?

3. Your mother is sick today. What should you do?

4. You come late and don‘t see any friends in the class. Tell the possibilities about it.

5. You‘ll have the exams next week. What you must you do?

SMA Alfa Centauri Bandung Halaman 111


MODUL PERSIAPAN UN & SBMPTN BAHASA INGGRIS 2017/ 2018

MODAL PERFECT

EXERCISE:

I. Complete the following sentences with Modal perfect.

1. The ground was wet this morning. It ________________ (rain) last night.
2. Tom wake up late this morning. He ________________ (sleep) earlier last night.
3. Susan ______________ (win) the contest but she got an accident on the way to the stage last month.
4. John got lost to my house last. He ____________________ (take) the wrong bus.
5. Jane got failed in her interview because of being late last week. She _____________________ (come)
earlier to the interview.

SMA Alfa Centauri Bandung Halaman 112


MODUL PERSIAPAN UN & SBMPTN BAHASA INGGRIS 2017/ 2018

II. Use Modal Perfect to give your responses or opinions about the following situations.

1. I didn‘t see my jacket in the class this morning. I had left it there last night. (might)

2. Tom got accident last month. He drove his car 100 kilometers an hour. (should)

3. Tom was absent from school yesterday. Nobody knew where Tom was. (might)

4. I planned to take some pictures when I climbed the mountain. But I lost my camera on the way to get
the top of the mountain. (could)

5. Everyone laughed when the film started. They watched a comedy show in the theatre last night. (must)

SMA Alfa Centauri Bandung Halaman 113


MODUL PERSIAPAN UN & SBMPTN BAHASA INGGRIS 2017/ 2018

CONDITIONAL SENTENCES
Conditional Sentences are also known as Conditional Clauses or If Clauses. They are used to express that the
action in the main clause (without if) can only take place if a certain condition (in the clause with if) is fulfilled.
There are three types of Conditional Sentences:

Conditional Functions Form Fact


Sentences if clause Main clause
Type 1 To express real or  If + Simple  Will + infinitive
very probable present  Can/may/might + infinitive
situations in the  Unless + simple  Imperative
present or future. present
 If you study  You will pass
 Unless you study  You won‘t pass
 If you‘re happy  Clap your hands
Type 2  To express If + past simple Would/could/might/ + bare Present tense
imaginary infinitive
situations which  If I had money  I would buy a new car.  I don‘t have
are contrary to money, so I
facts in the present don‘t buy a
and , therefore, new car.
are unlikely to
happen in the  If I were you  I wouldn‘t buy that car.  I am not you.
present or future.
 To give advice.
Type 3  To express  If + past perfect  Would/could/might/have + Past tense
situations which past participle
are contrary to  If I had had  I would have bought a  I didn‘t have
facts in the past. money new car money.
 To express regrets  I wouldn‘t have made such  I wasn‘t
or criticism.  If I had been a big mistake. careful, so I
more careful made a big
mistake.

Exercise:
Fill in the blanks with the correct verb!
1. If you have enough time, please … (clean) your room before you go to school.
2. She would give you the money if she … (have) it.
3. Had my mother arrived at the sale earlier, she … (find) a better dress.
4. She would have understood her English teacher if she … (speak) slowly.
5. If today … (be) Sunday, we could go to the beach.

SMA Alfa Centauri Bandung Halaman 114


MODUL PERSIAPAN UN & SBMPTN BAHASA INGGRIS 2017/ 2018

Choose the correct answer!


1. If she had spare time, she would join us. It means ….
A. She didn‘t have spare time
B. She has spare time
C. She join us
D. She does not have spare time
E. She did not join us

2. If I hadn‘t quarreled with John, I could have gone to the party with him.
It can be concluded that …
A. Perhaps I will go with John
B. John promised to go with me
C. I didn‘t go with John
D. John would like to go with me
E. I went with John

3. When the little boy saw the dog, he ran away and was bitten. We can conclude that ….
A. If he hadn‘t run away he wouldn‘t have been bitten
B. If he doesn‘t run away, he won‘t be bitten
C. If he hasn‘t run away, he wouldn‘t be bitten
D. If he hasn‘t run away, he won‘t be bitten
E. Unless he run away, he wouldn‘t be bitten

4. Had I known that Agus was a liar, I … my secret.


A. would not tell him
B. did not tell him
C. will not tell him
D. had not told him
E. would not have told him

5. ―If all students pass their final examination, the teacher will give a party for them at this bouse‖.
It means that ….
a. there will possibly be a party
b. there has been a party
c. there is no party at all
d. there was a party
e. there has to be a party

SMA Alfa Centauri Bandung Halaman 115


MODUL PERSIAPAN UN & SBMPTN BAHASA INGGRIS 2017/ 2018

SUBJUNCTIVE

The subjunctive is used to indicate conditions that aren‘t true. It also appears in commands, wishes, and
requests.

Types of Subjunctive

 Present Subjunctive
Tevye, the main character in the musical Fiddler on the Roof, sings ―If I Were a Rich Man‖ with the sadness
of a man who knows that he‘ll never be anything but poor. Tevye‘s song is about a condition contrary to fact
— something that is not true. Take note of the verb in the title: were. Normally (that is to say, in an indicative
sentence) the subject–verb pair would be I was. But Tevye sings If I were because he isn‘t a rich man. The
verb were is in subjunctive mood.

Here are some examples of present and future tense subjunctives:


Subjunctive: If Roger were an honorable spy, he would not reveal the atomic secret hidden in the bean
burrito.
Why it’s subjunctive: Roger is not an honorable spy, and he‘s going to blab the secret.

To sum up, in subjunctive sentences, were is usually all you need. Here are a few details about subjunctive
for present or future statements of conditions contrary to fact:
 Use were for all subjects in the part of the sentence that expresses what is not true. (If she were
entranced by Max‘s explanation.)
 For the other part of the sentence, use the helping verb would. (Lola would stare at him in silence.)
 Never use the helping verb would in the untrue part of the sentence. For example:
Wrong: If I would have been president, I would ask the Martian colony to secede.

Right: If I were president, I would ask the Martian colony to secede.

Here are some more examples:


If only I could turn back the time, I would stay for a while.
(The fact is I cannot turn back the time).

I wish I were a queen.


(The fact is I am not a queen).

SMA Alfa Centauri Bandung Halaman 116


MODUL PERSIAPAN UN & SBMPTN BAHASA INGGRIS 2017/ 2018

She walks as if she were a supermodel.


(The fact is she is not a supermodel).

I would rather he picked me up now.


(The fact is he doesn‘t pick me up now).

 Past Subjunctive
For past tense sentences, the had belongs in the part of the sentence that is contrary to fact. The contrary-
to-fact (that is, the lie) part of the sentence may begin with if, or the if may be understood.

Here are a few examples of the past subjunctive:


Subjunctive with the word if:
If Lola had known about the atomic secret, she would not have eaten that burrito.

Subjunctive without the word if:


Had Lola known about the atomic secret, she would not have eaten that burrito.

Why it’s subjunctive:


Lola knew nothing about the atomic secret; Roger told her that the crunch in the burrito came from an
undercooked bean.

Here are some more examples:


I wish I had brought that dress yesterday.
(The fact is I didn‘t buy that dress).

She acted as if he had known the secret.


(The fact is she didn‘t know the secret).

I would rather he had called me last night.


(The fact is he didn‘t call me).

SMA Alfa Centauri Bandung Halaman 117


MODUL PERSIAPAN UN & SBMPTN BAHASA INGGRIS 2017/ 2018

EXERCISE:

1. Husband : My car‘s battery is very weak.


Wife : Why don‘t we get a new one?
Husband : I think we should, but I wish I could recharge the battery.
The underlined expression means that he … recharge the battery.
A. can D. does not
B. could E. could not
C. cannot

2. ―Is Dina still out of town?‖


―Yes. I wish she … at home now so that I can take her to the movie.
A. is D. had been
B. will be E. would be
C. were

3. ―If only his son had studied harder‖. It means ….


A. His son didn‘t study harder
B. His son had studied harder
C. His son has studied harder
D. His son will not study harder
E. His son never studies harder

4. She is always busy with her work. I wish I … her more often.
A. visit D. have visited
B. would visit E. could visit
C. am visiting

5. ―Maybe Paula will bring us good news about our test‖.


―I wish she ….‖
A. does D. would
B. will E. brought
C. brings

SMA Alfa Centauri Bandung Halaman 118


MODUL PERSIAPAN UN & SBMPTN BAHASA INGGRIS 2017/ 2018

LATIHAN USBN
This text is for questions 1 to 3.

ANNOUNCEMENT
Pay attention!
. Our School will have Debate Competition

. Participants: All students in our school


. It will be held from 22nd – 26th May 2018
Prizes : I. Rp. 2.000.000
II. Rp. 1.500.000
III. Rp. 1.000.000
Please join us!

1. What does the announcement tell you about ?


A. The school activity 3. Participants: All students in our school
B. The school competition The underlined word has the closest meaning
C. The school debate competition with………
D. The payment of debate register A. member of the school who take part in the
E. The competition held in the school competition
B. group in the school that are involved in the
2. How many days will the competition be held ? competition
A. Four days C. people in the school who take part in the
B. Five days competition
C. Six days D. students who take part in the competition
D. Seven days E. students who are at school
E. Eight days

This text is for questions 4 to 6.

Dear Mr. Blake,


It‘s with great disappointment that I‘m writing this letter. We have bought sea food from your company
for the last five years. We have always been really pleased with the quality and service. That is until now.
Last Friday, we were scheduled to receive our order of 100 pounds of shrimp and 150 pounds of crab at
9.00 A.M. Finally, at 11.30 A.M. your delivery truck arrived, giving us only 30 minutes to prepare for our lunch
time crowd. Unfortunately, that wasn‘t the only problem. The order contained half of what we ordered, and most
of the shrimp was bad. We had to change our menu at the last minute, and as you can imagine, we had many
unhappy customers.
We will not pay for this order and are seriously considering another vendor. It is only because we have
been buying from you for so long then we want to hear explanation. You can contact me at the restaurant.

Sincerely,

Joanne Brown
(Manager)

SMA Alfa Centauri Bandung Halaman 119


MODUL PERSIAPAN UN & SBMPTN BAHASA INGGRIS 2017/ 2018

4. What does the letter tell us about? 5. What time did the writer receive the order?
A. Joanne Brown has been the consumer of A. at 8.30 A.M.
Mr. Blake‘s company for five years. B. at 9.00 A.M.
B. Joanne Brown‘s pleasure after receiving C. at 9.30 A.M.
the sea food she ‗s bought from Mr.Blake‘s D. at 11.00 A.M.
company. E. at 11.30 A.M.
C. Joanne Brown‘s feeling satisfaction about
the quality and service of the sea food she 6. We will not pay for this order and are seriously
‗s bought from Mr.Blake‘s company. considering another vendor.
D. Joanne Brown‘s complaining about the The underlined word has the closest meaning
quality and service of the sea food she ‗s to?
bought from Mr.Blake‘s company. A. Customer
E. Mr. Blake‘s complaining about the quality B. Consumer
and service of the sea food she ‗s bought C. Merchant
from Jonne Brown‘s company. D. Server
E. Client

This text is for questions 7 to 8.


Sylla Pizza
Authentic Italian Pizza by True-blue Neapolitan Chef
A cozy little restaurant with creamy sand-colored walls, deep-toned wood counters and furnishings and natural
grey stone.
Food and Beverage :
Italian Cuisine
Italian Pizza and Pasta
Various Kinds of juice
Best Italian Wine

Business Hour Details :


Sunday to Thursday : 11.30 AM to 2.30 PM
Friday to Saturday : 11.30 AM to 5.30 PM

Address :
30 Robertson Quay Riverside View
#01-14 Singapore 277798
Tel : +659484938

7. Which atmosphere can you find in the restaurant?


A. Inconvenient
B. Festive
C. Convenient
D. Cheerful
E. Childlike

8. When are you not suggested to dine at Sylla Pizza?


A. at the weekend
B. on weekdays
C. Monday and Tuesday
D. at noon
E. late evenings

SMA Alfa Centauri Bandung Halaman 120


MODUL PERSIAPAN UN & SBMPTN BAHASA INGGRIS 2017/ 2018

This text is for questions 9 to 12.

There was once a poor old widow her only obedient son. with son. One day mother said, ‖Son, go to
the outside cellar and bring in the last of the flour, so I can make bread.‖.
The boy did as his mother asked and took a bowl to carry the flour from the cellar into the house. As he
left the cellar, the North Wind blew the bowl. Flour went all over the boy‘s face and across the countryside.
The boy thought the North Wind would correct the bad deed. He walked and walked until he found the
house of the North Wind .‖ Good day‖. Said the boy. ― I know you were only playing a trick, but my mother and I
are poor. You blew away the last of our flour and we are hungry.‖
The North Wind apologised to the boy and gave him a special cloth. ― Take this. Put it over a table and
say the words, ―Spread, cloth, spread. ‗ You will never be without food again,‖.
The boy thanked the North Wind and carried the cloth under his arm. He had walked a long way and
stopped at an inn for the night.‖ How will you pay?‖ asked the innkeeper, ‖You say you have no money for food
or a bed!‖.
The boy placed the cloth over a table and said, ―Spread, cloth spread!‖ The table was suddenly covered
with delicious food and drinks.
Then, the innkeeper gave the boy a room of his own. During the night, he crept into the boy‘s room and
traded a table cloth that looked exactly like the boy‘s magic cloth. The next morning, the boy went home to his
mother. He told her about their good fortune and laid the cloth over the table. ―Spread, cloth, spread,‖ said the
boy. However, nothing happened and he decided to return to the North Wind.
Arriving at the North Wind‘s house, the boy told it about what had happened. Then, the North Wind
replied,‖ I will give you an item much better. This stick will solve your problem. When you say the words, Beat,
stick, beat !‘ the stick will do what needs to be done. Say ‗ stop, stick, stop, ‘when you want it to quit.‖
The boy thanked the North Wind and travelled towards home. He again stopped at the inn. The
innkeeper fed the boy and gave him a room of his own. Late that night, he came into the boy‘s room with a stick,
to trade it for the one belonging to the boy. This time the by woke up and yelled, ―Beat, stick, beat !‖.
The stick flew into the air and chased the innkeeper. The innkeeper ran around the room to avoid the
stick until he apologised and promised to give the boy‘s cloth back. Then, the boy said, ‖Stop, stick, stop‖, and
the stick stopped.
(adopted from: http://www.mikelockett.Com/stories.php?action=view&id=118(4th November 2016)

9. How did the North Wind play a trick on the 11. What can we learn from the story?
boy? A. Telling a lie will ruin oneself
A. it asked a magic stick to chase him B. Strangers are good to be friends.
B. it covered food with a magic cloth. C. Bad deeds will receive bad rewards.
C. it exchanged a magic cloth with a false D. Playing tricks will avoid one‘s fortune.
one E. Clarification is needed for
D. it exchange a magic stick with a false one misunderstandings.
E. it blew the boy‘s flour to all directions.
12. There was once a poor old widow with her only
10. What is the innkeeper like? He was …. obedient son, …
A. spoiled What is the antonym of the underlined word ?
B. evil A. Kind
C. tricky B. Pious
D. stingy C. Friendly
E. arrogant D. Naughty
E. Rebellious

SMA Alfa Centauri Bandung Halaman 121


MODUL PERSIAPAN UN & SBMPTN BAHASA INGGRIS 2017/ 2018

This text is for questions 13 to 15.

JAKARTA: Jakarta residents have been advised to be on the alert for flood-borne diseases, including
leptospirosis, which is transmitted through rat urine.
Aside from diarrhea, residents must be wary of leptospirosis since the floodwater could contain the
leptospirosis bacteria. A senior official with the Central Jakarta Health Office, Maryana, was quoted as saying by
beritajakarta.com.
Maryana said resident could avoid contacting the disease by ensuring good hygiene in their
environments.‖They should wear boots when walking through water that might be contaminated by rat urine,‖ she
said.
Those infected by the disease display various symptoms, including a prolonged fever, yellowish eyes and
pallor, and require immediate medical treatment.

13. What is the topic of the text ? B. Diarrhea may cause prolonged fever,
A. The dangerous of rat urine yellowfish eyes, and parlor.
B. Leptospirosis disease C. Leptospirosis is spread through rat urine.
C. The health office‘s advice for Jakarta D. People who are infected by leptospirosis
Resident on how to prevent flood borne need to get medical treatment.
diseases, including Leptospirosis E. The citizen should not walking barefooted
D. Immediate treatment were required for through contaminated water.
Leptospirosis
E. Ensuring good hygene in Jakarta 15. ―… , including a prolonged fever,…‖
environtment. What is similar word of prolonged?
A. Protracted
14. Which of the following statements is not true B. Shortened
about the text? C. Widened
A. the government has been warned the D. Enriched
people about leptospirosis. E. Gained

This text is for questions 16 to 18.


ANNOUNCEMENT

TO ALL 4TH BEACH VOLLEYBALL TOURNAMENT PARTICIPANTS:

PLEASE BE INFORMED THAT THE TOURNAMENT THIS COMING AUGUST 7


( SUNDAY) IS CANCELLED DUE TO THE CURRENT INCLEMENT WEATHER.

IT IS TENTATIVELY RECHEDULED TO AUGUST 14 (SUNDAY) IF THE WEATHER WILL PERMIT, PLEASE WAIT
FOR UPDATES.

THANKS YOU FOR YOUR KIND UNDERSTANDING

16. What can we infer about the announcement above?


A. The current inclement weather
B. The 4th Beach Volleyball Tournament
C. The Volleyball tournament participants
D. The cancelation of the 4th Beach Volleyball Tournament
E. The rescheduling of the tournament regardless the weather

SMA Alfa Centauri Bandung Halaman 122


MODUL PERSIAPAN UN & SBMPTN BAHASA INGGRIS 2017/ 2018

17. How long will the tournament be postponed? 18. ―Please be informed that the tournament this
A. 4 Days coming August 7 (Sunday) is cancelled due to
B. 7 Days the current inclement weather‖.
C. 10 Days What does the underlined word mean?
D. 11 Days A. Sunny
E. 14 Days B. Bright
C. Terrible
D. cloudless
E. Incredible

This text is for questions 19 to 21.

25 Brighton Boulevard
Leasington NSW 2066
The Manager
Steelwood homes
12 Scott Street
Leasington NSW 2066

Dear Sir or Madam,


I rent a house from your agency in Brington Boulevard and there is a problem with the stove. Two elements are
not working and it is very difficult for me to cook a meal.
This problem is urgent and I would appreciate if you could arrange to have the stove fixed immediately.

Your faithfully,
Sarah Johnson

19. What is the letter about?


A. Appreciating
B. Complaining
C. Suggesting
D. Offering
E. Inviting

20. Who is Sarah Johnson?


A. Tenant
B. Owner
C. Manager
D. Landlord
E. Secretary

21. …. if you could arrange to have the stove fixed immediately.


The opposite of the underlined word is….
A. Rapidly
B. Directly
C. Recently
D. Promptly
E. Eventually

SMA Alfa Centauri Bandung Halaman 123


MODUL PERSIAPAN UN & SBMPTN BAHASA INGGRIS 2017/ 2018

This text is for questions 22 to 23.


BALI‘S NEWEST APARTMENT DEVELOPMENT
A DREAM BECOMES REALITY

Sunwel Beach Residences Ketewel-Bali


Prime property – Prime location – Prime investment.
Only 5 minutes away from Sanur, directly at the beach
All apartments with view to the sea,
Full hotel amenities
Large pools, spa, fitness, Sauna &
Steam, parking, restaurant
For complete information, please, visit our website
Invitation – stimulation – temptation
Where: Hotel Mulia Senayan, Jakarta
(Leatris Room)
When: 5th and 6th of May 2018
(15.00-21.00 WIB)
For reservation: PT. Umadamai
Phone: +62361759 958 or Email
For every purchase prior to 31.12.2016
We will invite the buyer for Ground-Breaking Ceremony
Flight from Jakarta to Bali and a night in a luxury villa FREE

22. The mainly passage tells us about...... 23. It can be inferred from the text that……
A. to promote about Apartement in Bali A. it is near from Sanur Beach.
B. to tell about the newest apartemen in Bali. B. offered a free apartmenent and flight from
C. to inform the Bali‘ newest Sunwel Beach Jakarta to Bali for prior purchase in a in a
Besidences Ketewel period time
D. to explain about the falicities of Ketewel C. it is offered Large pools, spa, fitness,
Bali Residences. sauna, and free charge for one night in a
E. to give information about the newest luxury Hotel.
apartement in Bali. D. Bali‘s newest apartment is good hotel to
have a view to the sea.
E. Bali‘s newest apartment offered free
charge for all the customer.

This text is for questions 24 to 27.

Blue-Tongue Lizard and his wife camped near a swamp long ago. One day Blue-Tongue
Lizard went to get some food, and while he was down at the swamp, he left his wife sitting under a
shady tree. He had not been gone very long when Taipan the Snake passed by Blue-Tongue Lizard‘s
camp. Taipan saw Blue-Tongue Lizard‘s wife sitting under the tree, and he decided he would steal her
away from Blue-Tongue Lizard. He made her come with him and together they ran a long way away.
Taipan the Snake did not know that Black Bird had been watching him, and as soon as he ran
away with Blue-Tongue Lizard‘s wife, Black Bird began singing out to Blue-Tongue Lizard. ‗Your wife is
gone, Taipan has taken her away‘, he cried. Blue-Tongue Lizard was still at the swamp getting food
when he heard Black Bird‘s call. He went back to the shady tree where he had left his wife and saw she
was gone. He put down his bag which was full of food and made a fire. He cooked his food and after he
had finished eating it, he went to get his spears.
He found they had all been broken by Taipan. Then he found the tracks of his wife and Taipan,
and he followed them. Blue-Tongue Lizard followed their tracks until he came to a tree in which a freshly
killed emu had been hung. It had been killed by Taipan and left there to be eaten that night. Blue-
Tongue Lizard knew that Taipan and his wife must be nearby and he soon found them near a river.

SMA Alfa Centauri Bandung Halaman 124


MODUL PERSIAPAN UN & SBMPTN BAHASA INGGRIS 2017/ 2018

When Taipan saw Blue-Tongue Lizard he ran to get his spears but Blue-Tongue Lizard had already
broken them.
‗We can fight with our teeth‘, said Blue-Tongue Lizard. Taipan agreed and the two of them
fought wildly, each of them trying to get a hold of the other. Until finally Blue-Tongue Lizard caught hold
of Taipan‘s body in his powerful jaws, and bit him in half. With Taipan the Snake dead, Blue-Tongue
took back his wife and together they returned to the swamp.

24. Which of the statements below is Not True E. Strategy to win the battle is the most
according to the text above? important thing.
A. The Blue Tongue Lizard was a smart
lizard. 26. Blue Tongue Lizard followed their tracks untill
B. The Black bird was the Taipan snake‘s he came to a tree ... (paragraph 3). The
friend. underlined word refers to ...
C. Taipan snake was the Blue Tongue A. Lizard and Taipan.
Lizard‘s friend. B. Lizard and his wife.
D. Taipan snake would kill the Blue Tongue C. Taipan and his wife.
Lizard‘s wife. D. Blue Tongue Lizards.
E. The Blue Tongue Lizard‘s wife wanted to E. Lizard and Taipan‘s wife.
go with Taipan snake.
27. He went back to a shady tree. The underlined
25. What is the moral value of the story? word has the similar meaning with ...
A. Hardwork can defeat anything. A. calm
B. Sneaking is the bad way in stealing other B. quiet
heart. C. leafy
C. Loyalty is important to maintain the D. moist
relationship. E. decayed
D. Powerful is the most important in winning
the battle.

This text is for questions 28 to 30.

BERLIN: European countries are stating to break down the barriers between intelligence and police agencies,
sweeping aside concerns about civil rights abuses in hopes of combating terrorism more effectively.
The agencies are starting to cooperative in what are being called ―war rooms‖, state – of – the – art
communication centers, that allow police and intelligence agencies to meet and share information, such as live
wiretaps, video or satellite pictures.
The moves have been in the works for more than a year but have taken on a new urgency sine terrorist
attacks in Spain in March and the killing of Dutch film director, Theo Van Gogh.
Next month, Germany will open its ―terrorism Information and Analysis Center‖, according to Jorg
Ziercke,the president of the German Federal Police.

28. What is the first paragraph about? 29. The moves have been in the works for more
A. The statement on breaking down of the than a year but have taken on a new urgency
barriers sine terrorist attacks in Spain ( Paragraph 3)....
B. The starting of cooperation on The underlined word has the closest meaning
communication centers to...
C. The effort to increase concern about civil A. exigency
right B. necessity
D. The meeting and sharing of information C. rapidness
E. The opening of terrorism information and D. spontanity
analysis center E. unimportance

SMA Alfa Centauri Bandung Halaman 125


MODUL PERSIAPAN UN & SBMPTN BAHASA INGGRIS 2017/ 2018

30. Why the agencies are being called ―war rooms‖. Because ...
A. the agencies are just communication
B. the agencies are very urgency
C. the agencies are communication center and share information
D. the agencies attacks terrorist
E. the agencies are cooperative

This text is for questions 31 to 33.

A Cold Crisp Day

BLINK... BLINK... I lifted my wet woolen gloves up to my damp eyelashes and wiped the delicate snowflakes from
my eyes. I could not feel my nose and my breath was like icicles forming in the air. I was cold. I was fed up!
I wanted to go home. Mum and dad had forced me and my 2 sisters –Jenny and Mary to go for a Sunday afternoon walk in
the park. I could see my footprints in the snowy path. Like a trail of breadcrumbs in the forest showing me the way home.
I stopped and listened to the magic around me. I could hear the wind howling like a stray dog hungry for a yummy
bone. The branches of the trees that used to whisper and rustle had become bare, brown bones and the grass was covered
with a white sparkling blanket. Then I saw a filmy web shimmer and glimmer beneath a pale sun. My breath was like silky
soft smoke wafting up into the freezing cold air.
Suddenly there was mum with a big cup of liquid heaven. She handed it to me with an enormous smile on her
face.‗ Here you are Jo, this will keep you warm.‖ I slowly sipped the delicious liquid and felt it dripping and slipping down into
my tummy like lava flowing down a mountainside. My wet woolen gloves were no longer frozen but warm. My breath had
thawed and I was no longer fed up or cold. I was happy and contented.

http://www.primaryresources.co.uk/english/englishD6.htm
accessed on 16 Feb 2017, 3.02 am

31. The text is mainly telling you about .... D. terrifying


A. going to the rainy forest E. momentous
B. walking on the winter park
C. drinking a big cup of coffee 33. Which sentence is true based on the text?
D. watching a magical nature A. The writer‘s parents have three sons.
E. enjoying a snowy day B. The writer was happy walking in the park.
C. The writer went to the park with her family.
32. ....it to me with an enormous smile on her face. The D. The writer heard a dog barking on her way to
underlined word has the closest meaning with... the park.
A. nice E. The writer slipped down her drink into her
B. good woolen gloves.
C. attractive

This text is for questions 34 to 36.

FRESHWATER ENVIRONMENT

From earliest times, man has been finding new uses for freshwater and making new demands upon
the world‘s freshwater resources. Today, the whole of modern society depends upon a vast supply to serve its
agricultural, industrial, domestic, and other needs of freshwater.
Of all major activities that depend on freshwater, agriculture is by far the World‘s largest costumer. In
much of Europe and Latin America, rainfall is usually not enough to water the corps. In the other parts of the
world, the climate simply does not produce enough rainfall. Therefore, farmers use irrigation. This technique is an
essential element of agriculture.
Freshwater is not only an integral part of agriculture. It also plays an important role in industrial sector.
As a source of power, it has been used since the early days of civilization in water wheels that is one of man‘s
first industrial inventions. Today, these simple devices are rarely seen in industrial societies, but water power is
more important than ever before.
Freshwater system has also developed for recreation and sport. Lakes are stoked with fish, and
waterways are dredged to provide sailing and swimming facilities. People have been developing freshwater
system for leisure pursuits.

SMA Alfa Centauri Bandung Halaman 126


MODUL PERSIAPAN UN & SBMPTN BAHASA INGGRIS 2017/ 2018

34. The text above mainly tells about .... D. People do not need to create the expand
A. New uses for freshwater use of fresh water such as for recreation,
B. New demands upon the freshwater this can make people lake of water.
C. Major activities depend on fresh water E. Let‘s use freshwater in balance to our
D. Fresh water system has also developed for need, although we know that the
recreation and sport. development could not be avoided
E. New uses and demands upon the world‘s
freshwater resources 36. What is TRUE about the text?
A. Modern society does not depend on a
35. What is the social function of the text? great supply of water.
A. People should use the water as much as B. In much of Europe and Latin America, the
they want because the earth water supply supply cannot sufficiently water the crops.
is abundant C. Today, water wheels are still commonly
B. All creature needs water; human needs found in industrial society.
based on their activities; animals and D. Irrigation does not take important part in
plants need water to live on agriculture.
C. People need water only for their on life E. People never see water at the mean of
without caring for other creatures such as recreation
plants and animal

This text is for questions 37 to 39.

THE UNIVERSITY OF OXFORD


The University of Oxford (informally Oxford University, or simply Oxford) is a university in Oxford,
England. It is the second oldest surviving university in the world and the oldest in the English-speaking world.
Although the exact date of its foundation remains unclear, there is evidence of teaching there as far back as the
11th century. The University grew rapidly from 1167 when Henry II banned English students from attending the
University of Paris. In post-nominals, the University of Oxford was historically abbreviated as Oxon. (from the
Latin Oxoniensis), although Oxf is nowadays used in official university publications.
After disputes between students and Oxford townsfolk in 1209, some academics fled north-east to
Cambridge, where they established what became the University of Cambridge. The two ancient English
universities have many common features and are often jointly referred to as Oxbridge. In addition to their cultural
and practical associations, as a historic part of British society, they have a long history of rivalry with each other.
Most undergraduate teaching at Oxford is organizedt around weekly essay-based tutorials at self-
governing colleges and halls, supported by lectures and laboratory classes organized by University faculties and
departments. League tables consistently list Oxford as one of the UK‘s best universities; the university regularly
contends with Cambridge for first place in the tables. Oxford consistently ranks in the world‘s top 10. For more
than a century, it has served as the home of the Rhodes Scholarship, which brings students from a number of
countries to study at Oxford as postgraduates or for a second bachelor‘s degree.
Oxford is a member of the Russell Group of research-led British universities, the Coimbra Group, the G5, the
League of European Research Universities, and the International Alliance of Research Universities. It is also a
core member of the Europaeum and forms part of the ‗Golden Triangle‘ of British universities.

37. What is main idea of paragraph 3?


A. League tables are very important organizations.
B. Oxford University as one of the world‘s top 10 in the world.
C. The reward of the Rhodes scholarship for the Oxford University.
D. The graduates from the Oxford University are considerable by the people who want to study.
E. The teaching strategies that makes Oxford University one of the most reputable university in the world.

SMA Alfa Centauri Bandung Halaman 127


MODUL PERSIAPAN UN & SBMPTN BAHASA INGGRIS 2017/ 2018

38. What does the word ―they‖ in ―they have a 39. How many years is the difference between the
long history of rivalry with each other.‖ in the establishments of Oxford University and
end of paragraph 2 refers to? Cambrige University?
A. Oxford University and Cambridge A. 43 years
University. B. 42 years
B. Cultural and practical associations. C. 44 years
C. Cambridge University. D. 142 years
D. Some academics. E. 143 years
E. Oxford University.

This text is for questions 40 to 42.

Men Sana In Corpore Sano


The term ―Men sana in corpore sano‖ means that a healthy body makes a sound mind. More and more
people nowadays are aware of the truth behind the maxim.
As a result, they make sports an integral part of their lives. Some people participate in sports to release
energy and tension, while others do it to make friends. Sports, such as soccer, basketball or boxing, have
become an acceptable way to release energy and aggression.
Other sport like golf, mountain climbing, dancing or bowling can be means of starting or consolidating
friendships. Most sports offer a constructive escape from the pressures of the everyday life. In fact, you can
change your life if you take up and keep up an activity suited to your character, abilities and lifestyle.
It is important to realize that no matter what kind of exercise you do, you can be sure that it‘s always
good for your health and that it‘s fun.

40. What is the communicative purpose of the C. A healthy body can change our lifestyle
text? D. A healty body makes us strong and
A. To tell the reader about the sports friendly
B. To explain about the kinds of sport E. Doing exercises can make our body
C. To show the reader the important of the energetic
sports
D. To persuade the reader to do exercises 42. Which of the following statement is NOT
regularly TRUE according to the text that people decide
E. To entertain the reader with the to do the exercises?
information about sports A. To make friends
B. To change your health
41. What does the term of ― Men sana in corpore C. To escape from the pressures
sano‖ mean? D. To release energy and tension
A. Doing sports can make our body fat E. To integrate part of their lives
B. A healthy body makes a sound mind

This text is for questions 43 to 45.

Walter Elias Disney (december 5, 1901- December 15, 1966) was an American animated film producer
and animator. He was also the creator of an American-based theme park called Disneyland, and the founder of
the highly profitable corporation, now known as The Walt Disney Company.
Disney was born in Chicago to Elias Disney and Flora Call. He was named after his father‘s close friend
walter Parr, the minister at St. Paul Congregational Church. In 1906, his family moved to a farm near Marceline,
Missouri. The family sold the farm in 1909 and lived in a rented house untill 1910, when they moved to Kansas
City, Disney was nine years old.
According to the Kansas City, Missouri, Public School District records, Disney began attending the Benton
Grammar School in 1911, and continued his formal education there untill he graduated on June 8, 1917. During
this time, Disney also enrolled in classes at the Kansas City Art Institute. In the fall of 1917, Disney rejoined his
family. He left school at the age sixteen and became a volunteer ambulance driver in World War I, after he
changed his birth certificate to show his year of birth as 1900 in order to be able to enlist in the service. He
served as a member of the American Red Cross ambulance Force in France till 1919.

SMA Alfa Centauri Bandung Halaman 128


MODUL PERSIAPAN UN & SBMPTN BAHASA INGGRIS 2017/ 2018

43. The text is about ... B. abandoned


A. Disney‘s life C. accompanied
B. Disney‘s Birth D. followed
C. Disney‘s History E. escorted
D. Disney‘s Country
E. Disney‘s education 45. From the text, we conclude that Disney ... to
become an ambulance driver.
44. He left school at the age sixteen and became A. rejoined his family
a volunteer ambulance driver in World War I. B. took a driving license
(Last paragraph) C. showed his birth certificate
The underlined word can be replaced by …. D. reduced the year of his birth
A. accomplished E. left Benton Grammar School

This text is for questions number 46 – 47.

1. My friend and his group drove on through mountains.


2. My friend ten-day-stay ended very quickly beside his two tours, all his day was spent on
the beach.
3. My friend stayed in Kuta on arrival.
4. The second tour to Ubud was a very different tour.
5. He went sailing or surfboarding every day.
6. Then they return very late in the evening to Kuta.
7. He was quite satisfied
8. It was not to see the scenery but to see the art and the craft of the island
9. There were so many places to see in Bali that my friend decided to join the tours to see as
much as possible.
11. On the first day of the tour, he was ready.

46. The best arrangement is…. 47. Where does my friend stay?
A. 1-8-3-5-6-7-10-4-2-9 A. Denpasar
B. 9-3-10-5-1-2-5-8-4-7 B. Ubud
C. 9-3-7-10-1-6-8-4-2-5 C. Bali
D. 9-3-10-1-6-4-8-2-5-7 D. Kuta
E. 1-8-3-5-10-2-9-4-7-6 E. Anyer

This text is for questions number 48 – 50.

1. Prepare clean carrots and cucumbers then cut them into long thin strips
2. Then cut the sliced salmon into little strips and make them fairlly similar in length
3. After the cooked rice is ready, put it in a bowl and add a pinch of sugar and salt. Then stir it
until it dissolves.
4. Place the mixture onto the rice and mix thoroughly by slicing the rice
5. Put the seaweed layer on the bamboo mat and then spread rice onto the seaweed
6. Spread out the bamboo in a long roll, then fold and roll it up.
7. Cut the roll in the middle. Take each half and place them parallel to each other.
8. Serve them and enjoy your sushi

48. What kind of food will the writer probably want to make?
A. Nasi Kebuli
B. Fried Rice
C. Lumpia
D. Bread
E. Sushi

SMA Alfa Centauri Bandung Halaman 129


MODUL PERSIAPAN UN & SBMPTN BAHASA INGGRIS 2017/ 2018

49. To make the food tasty, we should add seasoning such as ...
A. bamboo
B. fried rice
C. salt and sugar
D. seaweed and vinegar
E. cucumber and carrot

50. "Put the seaweed layer on a bamboo mat ... "


The underlined word (seaweed) can be replaced by...
A. rice
B. algae
C. aloe vera
D. bamboo
E. vinegar

National Monument

The National Monument or usually called as Monas is located in Central Jakarta, Indonesia. This
obelisk was built in 1961 with the purpose to commemorate the struggle and fight of the people in Indonesia to
achieve their independence. It was opened to the public in 1975.
The National Monument is a rectangular tower with the height of 132 meters. The typical part of the building
that became a special characteristic of it is the flame shape covered with gold foil located on the top of the tower.
There is a museum at the base part of the tower with the size of 80 x 80 meters. Everyone can visit the museum
to learn the history of Indonesia. There is also an amphitheater in this building called Ruang Kemerdekaan, it is
located in the "cup" part of Monas and it can be reached by using spiral stairs at the north and south doors. If you
go to the southern side of the building, you will find an elevator that can be used to access the top platform
where we will find the observation deck and also the flame of independence.

51. Draw about National Monument briefly?


52. Why does towering monument symbolize of Lingga and Yoni?

SMA Alfa Centauri Bandung Halaman 130


MODUL PERSIAPAN UN & SBMPTN BAHASA INGGRIS 2017/ 2018

Ujian Nasional SMA/MA


Tahun 2012
Listening section 2. ….
A. In the morning
In this section of the test, you will have the chance B. In the afternoon
to show how well you understand spoken English. C. In the evening
There are four parts to this section with special D. At noon
direction for each part. E. At midnight

Part I 3. ….
Questions 1 to 4 A. Expressing hope
B. Expressing pride
Directions: C. Expressing satisfaction
ln this part of the test, you will hear some dialogues D. Showing happiness
and questions spoken in English. The questions E. Asking for an apology
and the dialogues will be spoken twice. They will
not be printed in your test book, so you must listen 4. ….
carefully to understand what the speakers are A. Buying a book
saying. B. Celebrating a birthday
After you listen to the dialogue and the question C. Getting a beautiful book
about it, read the five possible answers, and decide D. Giving a birthday present
which one would be the best answer to the question E. Thanking the man for his gift
you have heard. Now listen to a sample question.
Part II
You will hear: Questions 5 to 7

Man : We are almost out of cereal. Directions:


Woman : I know. I will go to the grocery store to ln this part of the test, you will hear some
buy some more. I will also get some milk, incomplete dialogues spoken in English, followed
bread and fruit. by four responses, also spoken in English. The
Man : Would you get me some instant dialogues and the responses will be spoken twice.
noodles? They will not be printed in your test book, so you
Woman: Sure. must listen carefully to understand what the
speakers are saying. You have to choose the best
Narrator: What does the man want the woman response to each question.
to buy?
A. Fruit. Now listen to a sample question.
B. Milk. Man : May I taste the caramel pudding you
C. Bread. just made?
D. Noodles. Woman : Of course you may. How does it
E. Cereal. taste?
Man : ....
The best answer to this question is "Noodles".
Therefore you should choose answer (D). Narrator: What is the most appropriate response for
the man to reply?
1. …. A. It is very expensive, I think.
A. Shopping at a grocery store B. It‘s very delicious. I really enjoy the taste.
B. Going to a grocery store C. I think I will cook it myself.
C. Following directions D. I don‘t think I‘ll come with you.
D. Giving directions to the man Narrator : The best answer to the question "lt
E. Walking down Maple street provides excellent service." is choice B. Therefore,
you should choose answer B.

SMA Alfa Centauri Bandung Halaman 131


MODUL PERSIAPAN UN & SBMPTN BAHASA INGGRIS 2017/ 2018

5. Mark your answer on your answer sheet. 10.


6. Mark your answer on your answer sheet.
7. Mark your answer on your answer sheet.

Part III
Questions 8 to 11.

Directions:
In this part of the test, you will hear some dialogues
or monologues spoken in English. The dialogues or
monologues will be spoken twice. They will not be
printed in your test book, so you must listen
carefully to understand what the speakers saying.
After you listen to the dialogue or monologue, look
at the five pictures provided in your test book, and
decide which one would be the most suitable with
the dialogue or monologue you have heard.

8.

11.

9.

Part IV
Questions 12 to 15.

Directions:
In this part of the test, you will hear some
monologues. Each monologue will be spoken twice.
They will not be printed in your test book, so you
must listen carefully to understand what the
speakers are saying.
After you hear the monologue and the question
about it, read the five possible answers and decide

SMA Alfa Centauri Bandung Halaman 132


MODUL PERSIAPAN UN & SBMPTN BAHASA INGGRIS 2017/ 2018

which one would be the best answer to the question


you have heard.
14. ….
12. …. A. Snakes
A. Rail services B. Anacondas
B. Christmas holiday C. Kinds of snakes
C. Airports D. What an anaconda looks like
D. Snowfalls E. Another name for an anaconda
E. Snow stroms
15. ….
13. …. A. 10 – 20
A. Two B. 10 – 30
B. Three. C. 20 – 30
C. Four. D. 20 – 40
D. Five. E. 30 – 40
E. Six.

This is the end of the listening section.

This text is for question 16.

Dear Mr. Anwar,


We regret to inform you that we are unable to lend you the sum of $500 that you have requested, but it is
possible to grant you part of sum.
If you are still interested, please contact our main office to arrange an appointment with the assistant manager.
He will be happy to discuss the matter further.

Your sincerely,

(Lending Officer)

16. Instead of a loan of $500, what alternative is offered by the officer?


A. An appointment to discuss the offer.
B. A cash of $500.
C. An appointment with the lending officer.
D. A loan of less than $500.
E. An appointment with the assistant manager.

The following text is for questions 17 and 18.

ASEAN private companies and its dialogue partner countries are expected to be more actively engaged
in the region‘s economic integration process, says ASEAN deputy secretary-general Pushpanathan Sundram.
―We have provided several channels for ASEAN public officials and private sectors, such as regular consultations
and business dialogues with industry associations and business councils from ASEAN and partner countries‖, he
said during his presentation at the ASEAN Trade Facilitation Forum in Manado, North Sulawesi, on Saturday. He
explained that it was important for ASEAN to pay close attention to responses and feedback from the private
sector on the implementation of measures stipulated in the ASEAN Economic Community (AEC) blueprint.
The forum was jointly held by the Indonesian Trade Ministry, the US Mission to ASEAN and the US
Agency for International Development (USAID) on the sidelines of the 43rd ASEAN Economic Minister‘s Meeting
(AEMM). Sundram revealed that despite positive progress of the AEC blueprint measure implementation, several

SMA Alfa Centauri Bandung Halaman 133


MODUL PERSIAPAN UN & SBMPTN BAHASA INGGRIS 2017/ 2018

challenges remained in place, such as how to ensure a timely implementation of cutting issues such as
coordination at national and regional levels‖, he said.

17. How can private companies and public officials


from ASEAN countries participate in the 18. Which ministers of ASEAN countries most
region‘s economic integration process? probably attended the ASEAN Trade
A. By participating in ASEAN Trade Facilitation Forum?
Facilitation Forum. A. The Ministers of Foreign Affairs.
B. Through consultations and dialogues. B. The Finance Ministers.
C. By paying close attention to responses C. The Tourism Ministers.
and feedback from private sectors. D. The Trade Ministers.
D. Through intensive partnership with partner E. The Information Ministers.
countries.
E. By conducting meetings and conferences.

The following text is for questions 19 and 20.

CALL FOR PROPOSALS


(Funded by the European Commision)
The ongoing outbreaks of H5N1 Avian influenza in Indonesia have had a significant impact. Since
August 2003, more than 10 million livestock have been affected and the H5N1 virus has crossed the species
barrier resulting in at least 135 confirmed human cases with 110 deaths. Moreover, the potential exists for the
H5N1 Avian influenza virus to trigger a human pandemic. Research on clinical disease, virology and
epidemiology of H5N1 In Indonesia is needed to understand the ecology and variability in transmission to
human. To foster essential research, WHO requests for proposals that will then be evaluated through a review
process. The awards will then be made to competent researchers and institutions focused on research of local
scientist and public health professionals.
Areas for research include:
1. Studies of disease ecology, transmission and epidemiology.
2. Studies of clinical spectrum and management of disease.
Detailed information of call for proposal includes: application and assessment process, assessment criteria and
deadline or proposal submission, is attached and available at WHO Indonesia Country Office Web-site.

19. Who are specifically invited to send research 20. "... for the H5N1 Avian influenza virus to
proposals? trigger a human pandemic." (lines 4 - 5)
A. Scientist and professionals from around The underlined word is closest in meaning to
the world. ....
B. Indonesian scientist and public health A. to impact
professionals. B. to share
C. Professional researchers in health C. to increase
scientist. D. to immune
D. Health institutions in Indonesia. E. to cause
E. Researchers specializing in H5N1 virus.

SMA Alfa Centauri Bandung Halaman 134


MODUL PERSIAPAN UN & SBMPTN BAHASA INGGRIS 2017/ 2018

The following text is for questions 21 and 22.

Once, a cap seller was passing through a jungle. He was dead tired and needed to rest. Then, he
stopped and spread a cloth under a tree. He placed his bag full of caps near him and lay down with his cap on
his head.
The cap seller had a sound sleep for one hour. When he got up, the first thing he did was to look into his
bag. He was started when he found all his caps were not there. He was wondering where they could have gone.
Indeed, he was greatly puzzled.
When he looked up the sky, he was very surprised to see monkeys sitting on the branches of a tree,
each wearing a cap on his head. They had evidently done it to imitate him.
He decided to get his caps back by making a humble request to the monkeys. In return, the monkeys
only made faces of him. When he began to make gestures, even when he raised his first towards them to
threaten them, they also imitated him.
At last he hit upon a clever idea. "Monkeys are a great imitator," he thought. So he took off his own cap
and threw it down on the ground. As he had expected, all the monkeys took off the caps and threw them down
on the ground. Quickly he stood up and collected the caps, put them back into his bag and went away.

21. Why was each monkey wearing a cap on their 22. What is the moral value of the text?
head? A. Monkeys are clever animals.
A. They liked them. B. Put your belongings in a safe place.
B. They liked wearing caps. C. Kindness must be possessed by every-
C. They were imitating the cap seller. one.
D. They were teasing the cap seller. D. Foolishness is a source of faulty.
E. They had stolen the caps. E. You have to know you are talking to.

The following text is for questions 23 to 25.

Kapoposang is one of the largest of the Spermonde Islands, around 70 km northwest of Makassar,
South Sulawesi. Kapoposang covers an area of arounf 50,000 hectares and is inhabited by around 100 families.
Several other islands, both inhabited and uninhabited, lie near Kapoposang. The larger ones include
Papandangan, Kondongbali, Suranti and Tambakulu.
Kapoposang and the nearby islands were declared a Nature Tourism Park by the Department of
Forestry in 1999. Diving has been an important activity at Kapoposang. There were so few local divers at the
time, and because of the distance from other well-known diving areas in Indonesia, Kapoposang has basically
remained a secret.
The best time to visit Kapoposang is between April and November, when the seas are calm, the
weather favorable, and the wind less fierce than in the other months. The trip takes two hours by speedboat; the
ocean breezes help to cool your skin in the hot morning sun. As you approach Pulau Kapoposang, you see the
deep blue water, which suddenly meets an expanse of coral reefs between extensive shallows with the white
sands in the seabed. Sea grass also grows lushly near the shore. The sea bed at Kapoposang has a unique
composition; near the island are some quite shallow countours, and then suddenly sleep underwater cliffs
leading to seemingly endless depths. The water is very clear; you can expect underwater visibility of over 15
meters – ideal condition for divers.

SMA Alfa Centauri Bandung Halaman 135


MODUL PERSIAPAN UN & SBMPTN BAHASA INGGRIS 2017/ 2018

23. The text is about …. D. Among the big islands near Makassar.
A. Kapoposang and the surrounding islands E. Very close, to the north of Spermonde.
B. Kapoposang as a diving spot
C. Kapoposang as a very popular tourist spot 25. We know from the text that ….
D. Kapoposang as a secret area A. there are very few divers in the island
E. Kapoposang and the weather B. hundreds of families live in the island
C. Kapoposang has a large area of coral
24. Where is Kapoposang located? reefs
A. In the Nature Tourism Park of Makassar. D. Kapoposang always has good weather
B. Next to Makassar, South Sulawesi. E. Kapoposang is surrounded by deep blue
C. In the Spermonde Islands. waters

The following text is for questions 26 and 27.

PT. EUREUKA
Jl. Jendral Sudirman 112-116
Palembang

May 21, 2011

PT. Internusa
Jl. Gatot Subroto
Jakarta

Dear Sir or Madam,


We are interested in becoming a distributor for your
software products in Palembang. Would you please send us
your latest catalogs, brochures, and terms of agreement?
We are a hardware company and eager to add
software products to our sales offering. Our annual report is
enclosed.
We look forward to hearing from you soon.

Sincerely yours,
Imam Subagio
Imam Subagio
Chairman

26. Why did PT. Eureka have a letter to PT. 27. We know from the letter that ….
Internusa? A. PT. Eureka is a distributor of PT. Internusa
A. They are in the sales offering. B. PT. Eureka is a successful hardware
B. The distributors like to add them in their company
offering. C. PT. Eureka has not sold software before
C. There are latest catalogue and brochure to D. PT. Eureka will get the products soon
be added to the product. E. PT. Eureka will get the answer soon
D. PT. Eureka proposed to become a
distributor for PT. Internusa products.
E. The distributors like to the product send
and combine it with other products.

SMA Alfa Centauri Bandung Halaman 136


MODUL PERSIAPAN UN & SBMPTN BAHASA INGGRIS 2017/ 2018

The following text is for questions 28 to 30.

Remote sensing is the acquisition of information about an object or phenomenon, without making
physical contact with the object. In modern usage, the term generally refers to the use of aerial sensor
technology to detect and classify objects on Earth (both on the surface, and in the atmosphere and oceans) by
means of propagated signals (e.g. electromagnetic radiation emitted from aircraft or satellites).
There are two main types or remote sensing: passive remote sensing and active remote sensing.
Passive sensors detect natural radiation that is emitted or reflected by the object or surrounding area being
observed. Reflected sunlight is the most common source of radiation measured by passive censors. Examples of
passive remote sensors include film photography, infra-red, charge-coupled devices, and radiometers. Active
collection, on the other hand, emits energy in order to scan objects and areas whereupon a sensor then detects
and measures the radiation that is reflected or backscattered from the target. RADAR and LIDAR are examples
of active remote sensing where the time delay between emission and return is measured, stabilizing the location,
height, speed, and direction of an object.

28. From the text we know that remote sensing …. D. The source of energy radiated for sensing
A. is a way to obtain information about an determines whether it is the active or
object or phenomenon passive type.
B. does not difficult to do E. Active remote sensing and passive remote
C. makes physical contact with the object sensing are equally useful for the
D. does not make use of censors development of knowledge and
E. does not make use propagated signals technology.

29. What is the main idea of the second 30. Based on the text we can say that ….
paragraph? A. remote sensing is expensive technology
A. Active sensing is better than passive B. remote sensing is useful for transportation
sensing. C. we can apply the technology to study
B. The energy in sunlight is important for animals
remote sensing. D. medical science can be improved through
C. Passive sensing is not as strong as active remote sensing
sensing. E. remote sensing make it possible to collect
information of an object in a dangerous
area

The following text is for question 31 and 32.

THE WORLD AT YOUR FEET


GALAXY TOURS
THE SPECIALIST FOR SOUTH AND
NORTH ASIA, AUSTRALIA,
NEW ZEALAND, AND BALl OFFERS
UNBEATABLE BARGAIN !!!
TRAVEL TO EXCITING NEW ZEALAND
HURRY BOOK NOW! LIMITED SEATS
ONLY
Travel highlights
a) Visit a native Maori village and observe their
b) unique dances and customs
c) Stay at peaceful sheep farms and dairy farms.
d) Ski on Mount Cook, the highest peak in
e) New Zealand.
f) Asian meals provided.
g) Five-star accommodation.

All flights on Air New Zealand

SMA Alfa Centauri Bandung Halaman 137


MODUL PERSIAPAN UN & SBMPTN BAHASA INGGRIS 2017/ 2018

Departing 10 Dec, 16 Dec and 22 Dec


Prices : $2500 $1800
$2200 $1600
(Adults) (Children below 12)

BOOK NOW TO AVOID HOLIDAY CROWDS

31. According to the text, what is excluded from the


travel? 32. What is being promoted in the package?
A. Flower gardens. A. Tours to the New Zealand.
B. Sheep farms. B. Tours to the North Asia.
C. Maori dances. C. Tours to the South Asia.
D. Skiing. D. Tours to the Australia.
E. Dairy farms. E. Tours to the Bali.

The following text is for questions 33 to 35.

Being on time is a beautiful social ethic and one of great importance, as it creates efficiency is systems
and implies respect for one another. However, it is one the many values that is not easy for our students to learn.
Inner discipline, one that comes from an understanding of the set rules and regulations, is the highest
form of behavior. Most excellent schools try to instill this with a loving environment. Why, even adults arrive late
to meetings, work, etc. Here, we do not agree that late comers should be shut out. They can be given warnings,
most of which are enough to make them try their best to reach the school on time later. If this fails, invite their
parents to school.
By closing the gates, the school is behaving cruelly, to which we prefer not to expose our children, every
school has a responsibility to implant good educational principle, but it should be in appropriate ways. Good
schools create competitive students who can organize themselves effectively in society, so that everyone gets a
quality life as a result of ethics and values learned for as long as 12 years.
Children are precious and dependent on the adults for guidance. Understanding them is the key, and to
this end, both parents and schools must work hand in hand without playing the blame game.

33. What should the school do if a student can‘t D. Understanding the rules.
stop his/ her habit to come late to school? E. Inner discipline.
A. Punish him/her.
B. Shut him/her out. 35. ―Children are precious and dependent on the
C. Fail him/her. adults for guidance‖.
D. Talk to his/her parents. The underlined word is closest in meaning to
E. Try to understand him/her. ….
A. valuable
34. What does writer think to be ―the highest form B. excellent
of behavior‖ (in the second paragraph)? C. clever
A. Respect for one another. D. innocent
B. Not being late to school. E. naïve
C. Understanding the regulations.

The following text is for questions 36 to 38.

Will solar energy ever be a main source of energy for industrial societies? The solar energy is cheaper
than any fossil fuel because we can get the abundant source of energy from the sun.
In sunny deserts, the sun‘s radiation that reaches the ground can be used to produce electricity for
industries and homes. In one of experiment, solar ponds can produce hot water to drive generators.
Unfortunately, we can‘t power our homes entirely on sunlight. Solar energy can only be exploited in
bright light. Its great potential, therefore, is in countries that have clear skies for most of the year. Most houses,
however, are not in the sunniest part of the world. In addition to convert sunlight directly into electricity, solar cells

SMA Alfa Centauri Bandung Halaman 138


MODUL PERSIAPAN UN & SBMPTN BAHASA INGGRIS 2017/ 2018

are needed. Although they are very cheap to run, relatively they are expensive to buy and many people can‘t
afford them.

36. Which of the following can produce hot water to D. Solar ponds produce hot water to drive
drive generators? generators.
A. Solar car. E. The main important device to harness the
B. Solar cells. solar energy cheap.
C. Solar ponds.
D. Solar heating. 38. From the text, we know that ….
E. Solar batteries. A. converting solar energy into electricity is
expensive
37. Which of the following statements does not B. solar energy generators can be found in
agree with the text? most houses
A. Solar energy can onlu be harnessed in C. it is easy to convert solar energy into
bright sunlight. electricity
B. The solar energy is cheaper than any D. the potential source of solar energy can
other fossil fuel. only be found in desert areas
C. Solar energy is non-polluted. E. most of people in bright sunny areas use
solar energy

The following text is for questions 39 to 41.

A natural disaster is a terrible accident, e.g. a great flood, a big fire or an earthquake. It usually causes
great suffering and loss of a large sum of money. The casualties are injured or died. Some people are homeless
and need medical care. Floods occur when the water of rivers, lakes, or streams overflow their banks and pour
out into the surrounding land.
Floods are caused by many different things.
Often heavy rainstorms that last for a brief time can cause a flood. But not all heavy storms are followed by
flooding. If the surrounding land is flat and can absorb the water, no flooding will occur. If, however, the land is
hard and rocky, heavy rain cannot be absorbed. Where the banks are low, a river may overflow and flood
adjacent to lowland.
In many parts of the world floods are caused by tropical storms called hurricanes or typhoons. They
bring destructive wind of high speed, torrents of rain, and flooding. When a flood occurs, the destruction to the
surrounding land can be severe. Whole villages and towns are sometimes swept away by water pouring swiftly
over the land. Railroad tracks and buckles are uprooted from their beds. Highways are washed away.
When a building caught fire, the firemen pitched in to help battle the blaze. Before the pumps were
invented, people formed bucket brigades to fight fires. Standing side by side, they formed a human chain from
the fire to nearby well or river. They passed buckets of water from hand to hand to be poured on the flames.
The damage of fire did depend a great deal on where it happened. In the country or a small village, only
a single house might burn down. But in crowded cities, fire often destroyed whole locks and neighborhoods
before being controlled.

39. What can possibly prevent rivers and lakes from D. Water flood is absorbed by land directly
overflowing? E. Typhoons caused heavy flood.
A. An absorbent bed.
B. A rocky surrounding. 41. We know from the text that ...
C. A low land. A. The pump is the only tool used by
D. A high bank. firefighters now
E. A high road. B. The pump helps people to fight fires more
efficiently
40. We know from the text that ... C. Fires in big cities are always very big
A. River can sweep heavy flood. D. People no longer use buckets to control
B. People can make money from flood. fire
C. The destruction by flood is always less E. Only firemen can control fires in crowded
severe. cities

SMA Alfa Centauri Bandung Halaman 139


MODUL PERSIAPAN UN & SBMPTN BAHASA INGGRIS 2017/ 2018

The following text is for questions 42 to 44.

I, Milton Friedman was born on July 31, 1921, in Brooklyn, N.Y., the fourth and last child and first son of
Sarah Ethel (Landau) and Jeno Saul Friedman. My parents were born in Carpatho-Ruthenia of the Soviet Union.
They emigrated to the U.S. in their teens, meeting in New York. When I was a year old, my parents moved to
Rahway, N.J., a small town about 20 miles from New York City.
I was awarded a competitive scholarship to Rutgers University. I graduated from Rutgers in 1932. I
financed the rest of my college expenses by the usual mixture of waiting at tables, clerking in a retail store,
occasional entrepreneurial ventures, and summer earnings. Shortly, however, I became interested in economics.
In economics, I had the good fortune to be exposed to two remarkable men: Arthur F. Burns and Homer
Jones. Arthur Burns shaped my understanding of economic research, introduced me to the highest scientific
standards, and became a guiding influence on my subsequent career. Homer Jones introduced me to rigorous
economic theory, made economics exciting and relevant, and encouraged me to go on to graduate work. On his
recommendation, the Chicago Economics Department offered me a tuition scholarship. As it happened, I was
also offered a scholarship by Brown University in Applied Mathematics, but, by that time, I had definitely
transferred my primary allegiance to economics. In 1976 I won the Nobel Memorial Prize in Economics for my
achievements in the fields of consumption analysis, monetary history and theory and for his demonstration of the
complexity of stabilization policy. In 1977, at age 65, I retired from the University of Chicago after teaching there
for 30 years.

42. Who influenced Milton Friedman on economic D. Brown University offered Milton Friedman
research? a tuition scholarship.
A. Sarah Ethel. E. Milton Friedman was offered a scholarship
B. Jeno Friedman. by the Chicago Economics Department in
C. Arthur F. Burns. Applied Mathematics.
D. Homer Jones.
E. Alfred Nobel. 44. What did Milton Friedman do in financing his
college expenses?
43. What is the main idea of paragraph 3? A. He was a teacher during summer.
A. Milton Friedman was very committed in his B. Ha had a job in entrepreneur company.
interest and talent in economics. C. He worked as a clerk in a retail store.
B. Homer Jones shaped Milton Friedman D. He sold tables to people.
understanding of economic research. E. He taught economics in the university.
C. Arthur Burns introduced Milton Friedman
to rigorous economic theory.

The following text is for questions 45 and 46.

"Laskar Pelangi" : The Audacity of


Hope
This is a movie adapted from o best selling Indonesian novel. It took 40 days of filming on Belitung
Island, Bangka-Belitung province, involving 12 local actors, it reportedly costs Rp.8 Billion, With all the efforts of
transforming Laskar Pelangi (Rainbow Warrior) into a moving picture, will it satisfy readers' imaginations?
Laskar Pelangi, the novel was writen by Andrea Hirata in 2005, based on his own experiences. It is
about an inspiring teacher and her 10 students in the poverty-stricken Kampung Gantong in belitung, the poor
condition of their school building does not dampen their high spirits and hopes for a better future.
TWO years later, the novel became a phenomenon in Indonesian literature. With its humanistic touch,
Laskar Pelangi has sold more than 500.000 copies and has won the position of Must-Read novel in every corner
of nation's bookshops and media review pages. It has finally overcome the domination of teen-lit, chick-lit and
even religious novels, the popular theme of today.
Andrea entrusted the filming of the story to the respected figures in the film industry, Mira Lesmana and
Riri Riza as producer and film director. In July 2007, Mira and Riri started the pre-production, which took a year
to finish. Together with the scriptwriter Salman Aristo they decided to create a different scenario for the film.

SMA Alfa Centauri Bandung Halaman 140


MODUL PERSIAPAN UN & SBMPTN BAHASA INGGRIS 2017/ 2018

45. What is the main idea of the second E. 'Laskar Pelangi' is about Andrea Hirata.
paragraph?
A. 'Laskar Pelangi' is the story of poor people 46. From the text above, we know that ....
at the Bangka Belitung province. A. The movie was played by poor children
B. 'Laskar Pelangi' is the story of poor school B. Andrea Hirata is the procedur of Laskar
children of the Belitung Island. Pelangi
C. ‗Laskar Pelangi' is about poverty in C. The novel of Laskar Pelangi was a
Kampung Gantong in Bangka Belitung. phenomenal work of literature
D. 'Laskar Pelangi' is about an inspiring story D. People like the movie better than the novel
towards a better life. E. People like the novel better than the movie

Questions 47 to 49 based on the following cloze text.


I recently took a trip to New Hampshire and had what I
would call my first genuine (47) … hiking up a mountain.
I‘ve gone up hiking before but this was without a doubt
the best experience of my life. My friend, Sean and
Madeline (48) … some pictures during the trip. We need
2 hours straight before we reached the summit. The
storm clouds were heading right toward us and you
could actually see the rain falling from the cloud. It was
like a wall of water moving at an alarmingly (49) … pace
ready to engulf us.

47. …. 50.
A. experience 1. You can make instant noodles with the
B. influence helo of a microwave.
C. experiment 2. Finally, add any items that you want and
D. expedition serve the noodles.
E. independence 3. First, fill the bowl with 2 cups of water and
the noodle.
48. …. 4. Microwave it for about three to five
A. broke minutes.
B. bought 5. Then, take the noodles out of the
C. brought microwave and pour the seasonings.
D. took 6. After the microwave beeps, wait for three
E. put minutes in the closed microwave.
7. Next, stir the noodles until the seasonings
49. …. dissolve.
A. wide
B. strong The best arrangement of the sentences above
C. long is ….
D. bright A. 1-7-5-6-2-4-3
E. fast B. 1-3-4-6-5-7-2
C. 1-2-7-3-6-5-4
D. 1-4-7-5-2-6-3
E. 1-4-6-5-2-3-7

SMA Alfa Centauri Bandung Halaman 141


MODUL PERSIAPAN UN & SBMPTN BAHASA INGGRIS 2017/ 2018

UjianNasional SMA/MA
Tahun 2013
Listening Section

In this section of the test, you will have the chance to show how well you understand spoken English. There are
four parts to this section with special direction for each part.

Part 1
Questions 1 to 4

Directions:
In this part of the test, you will hear some dialogues and questions spoken in English. The dialogues and the
questions will be spoken twice. They will not be printed in your test book, so you must listen carefully to
understand what the speakers are saying.
After you listen to the dialogue and the question about it, read the five possible answers, and decide which one
would be the best answer to the question you have heard. Now listen to a sample question.

You will hear:


Man : I give up! I‘ll stop learning French.
Woman : Why do you say that? You are making a lot of progress.
Man : No, I‘m not. I‘ve tried hard but still I cannot speak it well.
Woman : How come? You can speak Arabic, Hindi, Japanese and even Russian well, can‘t
you?

Narrator: What language is difficult for the man to learn?

A. Hindi
B. Arabic
C. French
D. Russian
E. Japanese

The best answer to the question is ―French‖. Therefore you should choose answer (C).

1. ….
A. Job vacancy 3. ….
B. Job interview A. The exam is easy
C. Printing company B. This time will not fly
D. Getting high salary C. The exam is in three weeks
E. Finding a job D. He knows many things about the exam
E. He still has three days for the exam
2. …. preparation
A. A novel
B. A comic 4. ….
C. A magazine A. She came to Nisa‘s party
D. A science book B. She couldn‘t make a party
E. An English Grammar Book C. She had to go to a doctor
D. She invited the man to a party
E. She didn‘t go to Nisa‘s party

SMA Alfa Centauri Bandung Halaman 142


MODUL PERSIAPAN UN & SBMPTN BAHASA INGGRIS 2017/ 2018

Part II
Questions: 5 to 7

Directions:
In this part of the test, you will hear some incomplete dialogues spoken in English, followed by four responses,
also spoken in English. The dialogues and responses will be spoken twice. They will not be printed in your test
book, so you must listen carefully to understand what the speakers are saying. You have to choose the best
response to each question.

Now listen to a sample question.

Man : hey, Cindy could you do me a favor?


Woman : What can I do for you?
Man : Would you mind going to the post office to send my letter?
Woman : ….
Narrator: What is the woman‘s possible response?
A. I am all out. Sorry.
B. I love to write letters
C. Sure, by all means
D. It‘s next to the police station
Narrator: The best answer to the question is: ―Sure,by all means‖. Therefore you should choose answer (C).

5. Mark your answer on your answer sheet.


6. Mark your answer on your answer sheet.
7. Mark your answer on your answer sheet.

Part III
Questions 8 to 11

Directions:
In this part of the test, you will hear some dialogues or monologues spoken in English. The dialogues or
monologues will be spoken twice. They will not be printed in your test book, so you must listen carefully to
understand what the speakers are saying. After you listen to the dialogue or monologue, look at the five pictures
provided in your test book, and decide which would be the most suitable one with the dialogue or monologue you
have heard.
9.
8.

SMA Alfa Centauri Bandung Halaman 143


MODUL PERSIAPAN UN & SBMPTN BAHASA INGGRIS 2017/ 2018

10. 11.

Part IV
Questions 12 to 15.

Directions:
In this part of the test, you will hear several monologues. Each monologue will be spoken twice. They will not be
printed in your test book, so you must listen carefully to understand what the speakers are saying.
After you hear the monologue and the question about it, read the five possible answers and decide which one
would be the best answer to the question you have heard.

Questions 12 and 13 are based on the following monologue.

12. …. 13. ….
A. Job Paris A. Job vacancy
B. The Louvre B. Job interview
C. Notre Dame C. Printing company
D. The crowded city D. Getting high salary
E. The City of Light E. Finding a job

Questions 14 and 15 are based on the following monologue.

14. …. 15. ….
A. Other beasts A. They ate an animal
B. The water B. They lived in a lake
C. The frogs C. They saw wild horses
D. A lake D. They went somewhere
E. A horse E. They jumped into water

This is the end of the listening section.

SMA Alfa Centauri Bandung Halaman 144


MODUL PERSIAPAN UN & SBMPTN BAHASA INGGRIS 2017/ 2018

Reading section

The following text is for questions 16 and 17.

Dear Mr. Budi Santoso


Sales Manager
Sindang Mahu Ltd.

I am writing to inform you of my resignation from the position of sales executive. My last day of work will be
March 10, 2013. I have signed an employment contract with another company. I would like to express my
appreciation for your excellent leadership in the company. I have learned a lot about direct selling. These skills
will be valuable in my future career. I have been fortunate to be employed by Sindang Maju. I have learned that
putting customer first is important in today‘s competitive environment.
I will do my best to finish my project before my leaving.

Sincerely,

Nadia Utami
Sales Executive
16. Nadia will quit working at Sindang Maju 17. From Nadia‘s letter, we know that she ….
because she …. A. promises to finish her project before
A. has been offered a position in another leaving Sindang Maju
company B. is promoted to another position in the
B. has signed a contract with another same company
company C. is fired from her current position and
C. is disappointed with Sindang Maju seeks another job
D. wants to have a better career D. quits the job at Sindang Maju to pursue
E. want a better payment higher salary
E. continues working at Sindang Maju as an
assistant

The following text is for questions 18 and 19.


Hotel Work:
One Month Training Course
Suitable young men and women are invited to apply for places on the one month training course on hotel work
organized by the Hong Kong Institute of Hotel Management. Tuition is free of charge and students who
successfully complete the course will be offered employment in the Colony‘s leading hotels.

The Training Course will take place from Monday 21st July to Friday 22nd August, from 9 a.m. to 4.30 p.m. daily,
except Saturdays and Sundays.

Applications for places on the course are welcome for students now in their third year at secondary school, who
have good knowledge of English and have interest in hotel work.
Application forms may be obtained from:
The Hong Kong Institute of Management,
PO Box 948,
The South China Times.

The closing date for application is April 29th

SMA Alfa Centauri Bandung Halaman 145


MODUL PERSIAPAN UN & SBMPTN BAHASA INGGRIS 2017/ 2018

18. Those who successfully complete the course 19. What requirement is needed by an applicant to
will be given …. apply for the hotel work training?
A. free tuition A. Able to speak Mandarin.
B. a further training course B. Secondary school graduate.
C. jobs in big hotels in the colony C. Good knowledge of English.
D. a chance to stay in the colony‘s leading D. Reputable university graduate.
hotels for one month E. 2-year experiences in hotel work.
E. membership of the Hong Kong Institute of
Hotel Management

The following text is for questions 20 to 22.

Rafael Nadal pull out of London Olympics

Associated Press|Thu, 07/19/2012 10:00 PM|Sports

Rafael Nadal has pulled out of the London Olympics, saying he is not in condition to compete. Nadal
said in a statement Thursday it is one of the saddest moments of his career and that he will not travel with the
Spanish delegation to London. He was sent to be the flag bearer for Spain during the opening ceremony.
Nadal won gold in Beijing in 2008. He has not played since he was eliminated in the second round of
Wimbledon by 100th-ranked Lukas Rosol.

20. Rafael Nadal has pulled out of London D. defeated the world‘s 100th ranked Lukas
Olympics because he …. Rosol
A. is not fit enough to compete E. was eliminated by the world‘s 100th ranked
B. will not be with the Spanish delegation Lukas Rosol
C. has just been from the Beijing Olympics
D. did not want to be the flag bearer for Spain 22. Paragraph 1 tells us that Rafael Nadal decides
E. was eliminated in the 2nd round of to ….
Wimbledon A. leave London soon
B. leave the Olympics
21. In Wimbledon, Rafael Nadal …. C. compete in the Olympics
A. was to be the flag bearer for Spain D. play in the London Olympics
B. was injured during the Wimbledon match E. cancel his participation in the Olympics
C. beat the world‘s 100th ranked Lukas Rosol

The following text is for questions 23 to 25.

BRITAIN: they have a show. They have a social networking. Now comes, Wikileaks – the album. Wikileaks is
branching out into the entertainment industry, announcing a ―Beat the Blockade‖ CD intended to raise money to
keep the online transparency advocates afloat.
The group said in a statement late Thursday that the CD would feature 12 songs including ―Where
There Are No Secret‖, ―The Ballad of Julian Assange‖, and ―B Manning‖.
It is the latest attack outside the secret-busting business for Wikileaks, which has spawned a TV show,
called ―The World Tomorrow‖ and started its own social network, called Friend of Wikileaks.
Wikileaks chief Julian Assange says his website, which once featured an anonymous electronic dropbox
for secret document, has been effectively mothballed by a US financial blockade. - AP

SMA Alfa Centauri Bandung Halaman 146


MODUL PERSIAPAN UN & SBMPTN BAHASA INGGRIS 2017/ 2018

23. What is the text about? D. The music CD called ―Beat the Blockade‖.
A. A website name Wikileaks. E. The electronic box for secret documents.
B. A TV show called ―The World Tomorrow‖.
C. Wikileaks‘ CD entitled ―Beat the Blockade‖. 25. ― … once featured an anonymous electronic
D. Wikileaks‘ anonymous electronic drop box. drop box ….‖ (paragraph 4).
E. The secret-busting business for Wikileaks. The underlined word is closest in meaning to
….
24. What is meant by the latest attack outside the
secret-busting business for Wikileaks? A. identified
A. The website business of Wikileaks. B. recognized
B. The TCV show called ―The World C. nameless
Tomorrow‖. D. mysterious
C. The social network called Friends of E. strange
Wikileaks.

The following text is for questions 26 to 28.


The Near-Death Story of Mellen-Thomas Benedict

In 1982, I died from terminal cancer. My condition was non-operable. I chose not to have chemotherapy.
I was given six to eight months to live. Before this time, I had become increasingly despondent over the nuclear
crisis, the ecology crisis, and so forth. I came to believe that nature had made a mistake – that we were probably
a cancerous organism on the planet. And that is what eventually killed me.
Before my near-death experience, I tried all sort of alternative healing methods. None helped. So I
determined that this was between me and God. I had never really considered God. Neither was I into any kind of
spirituality and alternative healing. I read various religions and philosophies. They gave hope that there was
something on the other side.
I had no medical insurance, so my life savings went overnight on tests. Unwilling to drag my family into
this, I determined to handle this myself. I ended up in hospice care and was blessed with an angel for my hospice
caretaker, whom I call ―Anne‖. She stayed with me through all that was to follow.

26. According to Mellen-Thomas Benedict, what is the reason for many people to die for cancer?
A. The nuclear and ecology crisis.
B. All sorts of alternative healing methods.
C. They do not believe in the existence of God.
D. Chemotherapy is too expensive for most people.
E. The nature‘s mistake to make the human being a cancerous organism.

27. We know from the story that Mellen-Thomas Benedict believed that he eventually died ….
A. because God decided so
B. because he was not operated
C. because he did not have chemotherapy
D. because of the alternative healing methods
E. because he did not have any medical insurance

28. Why did he choose to stay in a hospice care?


A. They will take care of his health problem.
B. He had to do the medical checks there.
C. Anne asked him to stay there.
D. His family told him so.
E. He was broke.

SMA Alfa Centauri Bandung Halaman 147


MODUL PERSIAPAN UN & SBMPTN BAHASA INGGRIS 2017/ 2018

The following text is for questions 29 to 32.

Tornados occur when the conditions that cause thunderstorms are unusually violent. Winds blow in
opposite directions around a strong updraft start a narrow, violent whirl. Centrifugal force effectively throws the
air away from the center, leaving a core of very low pressure. This is much like stirring water in a cup, thus
forming a vortex-like dip in the surface.
This low-pressure core acts as a partial vacuum, sometimes helping to lift the roofs off houses. Most of
the damage, though, results from the force of the wind itself. Around the edges of the whirl, wind speeds may
reach 300 miles (480 kilometers) per hour. At first, the tornado‘s funnel is whitish-gray because it is composed of
minute water droplets formed as the air in the funnel expands and cools. After touching down, the funnel
becomes dark because of all of the debris it has picked up. This debris can include soil, tree limbs, and parts of
buildings; tornados have been known to pick up automobiles, horse, and whole trees.
A tornado usually moves toward the east (or often northeast in the Northern Hemisphere and southeast
Hemisphere) at 25 to 40 miles (40 to 65 kilometers) per hour. Fortunately, most tornados are less than half a mile
(800 meters) wide; the edge of one may destroy all of the houses on one side of a street while leaving those on
the other side completely undamaged.
Deaths from tornados in the United States averaged roughly 100 per year over the last century.
However, they have dropped somewhat in recent decades as better forecasting and warning systems have been
implemented.

29. Why have deaths from Tornado in the United E. It has picked up debris.
States decreased in the recent decades?
A. Most tornados are less than half a mile 31. What are the impacts of Tornado?
wide. A. Water dropping.
B. There are no more tornados in the United B. Heavy disasters.
States. C. Whirl of winds.
C. There are better forecasting and warning D. Wind blowing.
systems. E. Grey and dark sky.
D. The tornado left the United States
completely undamaged. 32. ―While leaving those on the other side
E. The people are evacuated by the United completely undamaged‖. (Paragraph 3)
States government. The underlined word has similar meaning with
….
30. Why is tornado‘s funnel dark as soon as it
A. damaged
touches the ground? B. vanish
A. It contains water droplets. C. wounded
B. The temperature is changed. D. safe
C. It contains building materials. E. gone
D. It whirls in high speed.

The following text is for questions 33 to 35.

Faster planes and cheaper flights are making it easier than ever before for people to travel. In the most
‗developed‘ societies, visiting exotic places is a sought-after status symbol. The tourism industries of both
developed and developing countries have recognized this fact and are learning to take advantage of it.
There are, however, some problems associated with this new industry. Firstly, there is the increasing
crime rate. Some locals see tourists as an easy prey because, not only are they in unfamiliar territory and
therefore less able to take care of themselves, but also they carry visible items of wealth, such as cameras and
jewelry which can disposed of quickly for profit.

SMA Alfa Centauri Bandung Halaman 148


MODUL PERSIAPAN UN & SBMPTN BAHASA INGGRIS 2017/ 2018

Another major problem is health. With greater mobility comes greater danger of spreading contagious
diseases around the world. One carrier returning home could easily start an epidemic before their illness was
diagnoses. Moreover, the emergence of many diseases which resist antibiotics is causing scientists to be
increasingly concerned about this issue.
Also to be considered is the natural environment, which can be seriously threatened by too many
visitors. Australia‘s Great Barrier Reef, for example, is in danger of being destroyed by tourists and there are
plans to restrict visitors to some of the more delicate coral cays.
These are just three of the reasons why any country should be wary of committing itself to an extensive
tourism development program.

33. Why does the natural environment become one C. The government is planning to restrict
of the serious problems associated with the visitors to some coral cays.
faster planes and cheaper flight? Because …. D. The new industry makes the number of
unemployment increase.
A. It easily started an epidemic before their
E. The tourists come by plane.
illness was diagnosed
B. It can be destroyed by too many visitors
C. The health problem will increase 35. ―One carrier returning home could easily start
D. They carry visible items of wealth an epidemic before their illness was
E. It can increase crime rate diagnosed‖. (Paragraph 3). The underlined
word means ….
34. Why is the crime rate increasing due to the A. found
new industry? B. observed
A. Some tourists carry visible items of wealth. C. identified
B. The tourists are spreading contagious D. examined
diseases. E. healed

The following text is for questions 36 to 38.

In the sequel of ―Sherlock Holmes‘ this time, Professor James Moriarty (Jared Harris) is the most
dangerous enemy. Moriarty is not the kind of person who is hesitant to remove the lives of many people in order
to achieve his goals. Explosions in Strassbourg are one of the results of Prof. Moriarty‘s creation.
Holmes was nosy when his friend, Dr. Watson (Jude Law) helped Holmes the adventure this time.
There was also the Holmes brother, Mycroft Holmes (Stephen Fry) who came to be an accomplice of the
detective this time.
Arguably, the visual effect of ―A Game of Shadow‘ is more stable than its predecessor. A few slow-
movement scenes later find in the cease-fire. Colors are displayed also feels right.
Farce as those made by Holmes and several other characters in the film guaranteed to invite laughter.
The chemistry between Robert Downey Jr. and Jude Law is not be doubted anymore. Since in the first film, the
two men were able to captivate the audience and not be missed.
Chess compete between Holmes and Prof. Moriarty became the ultimate point. They both describe the
mind wanders respectively. Do you intrigued with the acting of the detective? The film ‗Sherlock Holmes: A game
of Shadow‘ can be an alternative entertainment at this weekend.

36. The chemistry between Robert Downey Jr. and Jude Law isn‘t be doubted because ….
A. the two men have captivated the audience in the first film
B. the chemistry is formed through their friendship in real life
C. farce made by the two men guaranteed to invite laughter
D. the two men are partner in investigating some cases
E. the two men described the mind wanders

SMA Alfa Centauri Bandung Halaman 149


MODUL PERSIAPAN UN & SBMPTN BAHASA INGGRIS 2017/ 2018

37. What did the movie director do to invite 38. What can we conclude of ―A Game of
laughter in his film? Shadow‖?
A. He hired the clowns. A. The chess competition showed mind
B. He showed the cartoons. wanders.
C. He acted like comedians. B. The visual effect of the movie is testable.
D. He made jokes within the scene. C. The movie is terribly out of standard.
E. He forced the actors to be funny. D. The actors did not perform well.
E. The movie is worth watching.

The following text is for questions 39 to 41.

Internet has been the most outstanding in the history of mankind. As with every single innovation, the
internet has its own advantages and disadvantages.
The target of the internet has always been the communication. By the beginning of computer‘s internet,
our earth has reduced and has attained the form of a global village. Now we can communicate in a fraction of
second with a person who is sitting in the other parts of the world. Today, for better communication, we can avail
the facilities of e-mail and we can chat for hours with our loved ones. There are plenty messenger services in
offering. With the help of such services, it has become very easy to establish a kind of global friendship.
However, the internet also has disadvantages. One of them is the wide spread of unsuitable contents
for kids. It is a very serious issue and related to children‘s healthy mental life. There are thousands of such
inappropriate contents on the internet which can be easily found. Though, the internet can also create havoc,
destruction and its misuse can be very fatal, the advantages of it out-weigh its disadvantages.

39. What is one of the advantages of internet? C. The internet has more advantages than
A. To create the misuse of time. disadvantages.
B. To easily find inappropriate person. D. Many modern devices can be connected
C. To chat with love ones only. to the internet.
D. To connect people in the other part of the E. The internet has become the most
world. noticeable innovation.
E. To spread issues related to children
healthy mental life. 41. All in all, the writer believes that the internet is
….
40. What is the main idea of paragraph 2? A. very harmful
A. The internet could create havoc and B. inappropriate
destruction. C. very useful
B. The internet could facilitate D. destructive
communication. E. cheap

The following text is for questions 42 to 44.

Many people imagine the sea-horse to be real horse living in the sea. This is not true at all. The sea-
horse is so called because it is a tiny creature with a horse-liked head which lives in the sea. Sea-horse are
found mostly in warm seas.
They have a long, flexible tail which is used for wrapping around weeds for support. They swim upright,
position, their movements being helped by a rapidly oscillating fin on their back. The male sea-horse lays her
eggs in this pouch.
The most remarkable thing about the sea-horse is that part of the process of reproduction is taken over
by the male creature. Once the female sea-horse has laid her eggs in the male creature‘s pouch, she leaves.

SMA Alfa Centauri Bandung Halaman 150


MODUL PERSIAPAN UN & SBMPTN BAHASA INGGRIS 2017/ 2018

The hatching of the egg is done by the male fish. Mating begins with the male and the female sea-horses doing
courtship dance. The male and female creatures swim around each other in circles.
At the end of the dance, the male appears to bow to its partner, but actually, the male is banding to
pump water from his pouch. Next, the female fish lays eggs into the male‘s pouch. About two hundred eggs are
laid. The pouch, then close up. The female sea-horse now swims away, leaving the male sea-horse to do
hatching.
The eggs are hatched about a month later. The young sea-horses are ejected from the male creature‘s pouch by
means of muscular contractions.

42. How does the sea-horse swing its fin? E. Its rapid movement.
A. With its back.
B. With its position. 44. We know from the text that ….
C. With its flexible tail. A. sea-horse eat weeds for their food
D. With a slow manner. B. the male sea-horse do the hatching
E. With a fast movement. C. sea-horse is called because it is a tiny
horse
43. What makes sea-horse remarkable? D. the female sea-horse keeps her eggs in
A. Its size. her pouch
B. Its reproduction process. E. the female sea-horse doesn‘t eat fish
C. Its shape. during pregnancy
D. Its respiratory process.

The following text is for questions 45 and 46.

PT SUMI INDO KABEL Tbk.


(“The Company”)
ANNOUNCEMENT
TO ALL THE SHAREHOLDERS

This is to inform the shareholders of PT. SUMI INDO KABEL Tbk. (―Company‖) that the Annual General meeting
for the shareholders (―meeting‖) shall be held on Monday September 10, 2012.

The invitation for the meeting shall be announced on Thursday, August 9, 2012 in (two) newspapers.

Those who are entitled to attend or wish to be represented at the meeting by proxies and eligible to vote at the
meeting are shareholders whose names are recorded in the Company‘s register of shareholders on Wednesday,
August 8, 2012 at 4 p.m.

Shareholders who will submit their proposal to the agenda of meeting in accordance with the Company‘s Articles
of Association must send it to the Directors at the latest 7 (seven) days before invitation of meeting.

Tangerang, July 25, 2012.

PT. SUMI INDO KABEL Tbk.


Director

45. The invitation will be announced on ….


A. July 25, 2012
B. August 2, 2012
C. August 9, 2012
D. August 8, 2012
E. September 10, 2012

SMA Alfa Centauri Bandung Halaman 151


MODUL PERSIAPAN UN & SBMPTN BAHASA INGGRIS 2017/ 2018

46. According to the text, the shareholders should ….


A. Send their proposal a week before the invitation of the meeting
B. Announce the invitation for the meeting on Thursday
C. Make a proposal during the meeting and represent it
D. Receive an invitation and a proposal from the Directors
E. Inform the invitation to the other shareholders

47. Arrange these sentences below into a good paragraph.

HOW TO TIE A NECK TIE


1. First of all, start with the wide end on your right.
2. Secondly, cross the wide end over the narrow, and back underneath.
3. Then bring the wide end up through the loop.
4. Finally, pull the wide end through the knot in front. Tighten and draw up to collar.
5. Then, bring the wide end up, and turn down through the loop.
6. After that, pass the wide end around to the front from your left to right.

A. 1-2-6-5-3-4
B. 1-2-3-4-5-6
C. 1-2-3-5-4-6
D. 1-2-5-3-6-4
E. 1-2-5-6-3-4

Questions 48 to 50: complete the following text with the words provided.

The Man and the Monster

Once upon a time there lived a young man, named Nazar. He lived a very poor life. One day, Nazar went to town
to look for a job. He had to climb up the hill and (48) … a thick jungle to get to a nearest town. Suddenly, he
heard (49) … from a bush. When he approached, he found some people gathering a big tree. Nazar realized that
the people were doing against God order. Then he took out a big (50) … axe and planned to cut down the tree.

48. ….
A. checked
B. examined
C. inspected
D. went through
E. hunted through

49. ….
A. trembles
B. shadows
C. movement
D. gestures
E. sounds

50. ….
A. sharp
B. edge
C. dull
D. rough
E. thick

SMA Alfa Centauri Bandung Halaman 152


MODUL PERSIAPAN UN & SBMPTN BAHASA INGGRIS 2017/ 2018

Ujian Nasional SMA/MA


Tahun 2014
Listening Section

In this section of the test, you will have the chance to show how well you understand spoken English. There are
four parts to this section with special directions for each part.

Part I
Questions 1 to 4.

Directions:
In this part of the test, you will hear some dialogues and questions spoken in English. The dialogues and the
questions will be spoken twice. They will not be printed in your test book, so you must listen carefully to
understand what the speakers are saying.
After you listen to the dialogue and the question about it, read the five possible answers, and decide which one
would be the best answer to the question you have heard. Now listen to a sample question.

You will hear


Man : How can I help you, Mum?
Woman : Please buy a kilo of rice, two kilos of sugar, a half kilo of eggs and a pack of tea.
Man : Do you need some chicken nuggets?
Woman : No, we still have some in the refrigerator.

Narrator: What shouldn't the man buy?


A. Eggs.
B. Tea.
C. Rice.
D. Sugar.
E. Nuggets.

Narator : The best answer to the question "What shouldn't the man buy?" is nuggets. Therefore you should
answer (E).

1. ….
A. Warm 3. ….
B. Drier A. Take a guava juice
C. Hotter B. Order an orange juice
D. Cooler C. Try an ice tea
E. Very hot D. Order a cola float
E. Drink mineral bottled water
2. ….
A. A sport competition 4. ….
B. Nation's youth education A. He was spy
C. The country's college students B. He was a bullfighter
D. Oil and gas company's science competition C. He wrote famous stories
E. Means of educating the nation's youth D. He wrote about bullfighting
E. He wrote an adventure novel

SMA Alfa Centauri Bandung Halaman 153


MODUL PERSIAPAN UN & SBMPTN BAHASA INGGRIS 2017/ 2018

Part II
Questions 5 to 7.

Directions:
In this part of the test, you will hear some incomplete dialogues spoken in English, followed by four responses,
also spoken in English. The dialogues and the responses will be spoken twice.
They will not be printed in your test book, so you must listen carefully to understand what the speakers are
saying. You have to choose the best response to each question.

Now listen to a sample question.

Woman : Do you have any plans for next weekend?


Man : I am thinking of going mountain climbing.
Woman : That's interesting. Can I go with you?
Man : Sure. Do you have any suggestions for activities there?
Woman : ….

Narrator: What does the woman probably respond?

A. Sorry, but I don't know much about that.


B. Yes, we could have a barbeque there.
C. Yes, I think that mountain is too high.
D. Yes, I really love mountain climbing.

Narrator: The best answer to the question "What does the woman probably respond?" is "Yes, we could have a
barbeque there." Therefore you should choose answer (B).

5. Mark your answer on your answer sheet.


6. Mark your answer on your answer sheet.
7. Mark your answer on your answer sheet.

Part III
Questions: 8 to 11.

Directions:
In this part of the test, you will hear some dialogues or monologues spoken in English. The dialogues or
monologues will be spoken twice. They will not be printed in your test book, so you must listen carefully to
understand what the speakers are saying. After you listen to the dialogue or monologue, look at the five pictures
provided in your test book, and decide which would be the most suitable one for the dialogue or monologue you
have heard.

8.
9.

SMA Alfa Centauri Bandung Halaman 154


MODUL PERSIAPAN UN & SBMPTN BAHASA INGGRIS 2017/ 2018

11.
10.

Part IV
Questions 12 to 15.

Directions:
In this part of the test, you will hear several monologues. Each monologue will be spoken twice.
They will not be printed in your test book, so you must listen carefully to understand what the speakers are
saying.
After you hear the monologue and the question about it, read the five possible answers and decide which one
would be the best answer to the question you have heard.

Question 12 and 13 are based on the following monologue.

12. …
A. Gallery complex. 13. …
B. Buddhist mythology. A. Old Empire.
C. Cambodia. B. Yosadapura Empire.
D. Kluner Empire. C. Meru Empire.
E. Angkor Wat. D. The King Empire.
E. Khmer Empire.

Questions 14 and 15 are based on the following monologue.

14. … 15. …
A. The poor mouse. A. By using his beak.
B. The prisoners. B. By using his talons.
C. The frog. C. By shouting loudly.
D. The mouse. D. By flying in the sky.
E. The hawk. E. By observing a mouse.

This is the end of the listening section

Reading section

The following text is for question 16 and 17.

Dear Big Meal's representative,

I'm writing to inform you that I had a negative bad experience at your location in Columbus, New Jersey
on August 4. My receipt number is 512, and the person who handled my order was Alex.
First of all, I recognize that you, as the reader of this letter, are not responsible for my bad experience,
but I am still upset about the situation.
I went to the drive through and ordered seven meals with no pickles. When I received my order, I
checked that all of the sandwiches and fries were in the bag paid and drove away. When I got home, I realized
my number seven had pickles on it. I'm allergic to pickles, and I didn't want to waste the sandwich, so I drove
back to the drive through to explain the situation and get it fixed.
I feel very disappointed with this interaction, as I usually enjoy my experiences at your restaurant. To fix
this situation, I would like a coupon for a free meal of my choice. I think an apology from Alex is also appropriate.

SMA Alfa Centauri Bandung Halaman 155


MODUL PERSIAPAN UN & SBMPTN BAHASA INGGRIS 2017/ 2018

Please contact me at 555.555.5555 or email me back at jkorkell@email.com. I would like this situation to be
resolved so I can continue to be a loyal Big Meal's patron.

Best,

Jim Korkell

16. What is the letter about? 17. Who is in charge to take the order of "the drive
A. Applying for a job. through service"?
B. Complaining bad service. A. The cashier.
C. Ordering a certain item. B. Alex.
D. Inquiring Mr Jim Korkell. C. Jim Korkel.
E. Reserve for a meal. D. The manager.
E. The customer.

The following text is for question 18 and 19.

18. What is this advertisement about? 19. The good point about the office suits is ….
A. A new office building. a. furnished
B. Office suites to rent. b. available in one type size
C. The unusual building downtown. c. far from public Places
D. Office suites to sell. d. several blocks away from subway stop
E. A building in the business district. e. easy to reach for commuters

The following text is for question 20 to 22.

Damaged Roads
The number of damaged roads linking Jakarta with West Java has certainly provoked complaints from locals and
road users. Take for an example, the roads in Bekasi and Tangerang, which have been the focus of the media.
Head of traffic officials, Sutraman said "The road should be repaired using the right sort of asphalt, stones, and
sand. Repairs should be done carefully so that the roads can be prevented from constant damage. Otherwise,
the repairs would be questioned".
(Jakarta Daily)
20. The text reports ….
A. the damaged roads linking Jakarta and West Java
B. the damaged roads in Bekasi and Tangerang
C. the complaints from locals and road users
D. the poor quality of the damaged roads
E. the poorly constructed roads in Jakarta and West Java

SMA Alfa Centauri Bandung Halaman 156


MODUL PERSIAPAN UN & SBMPTN BAHASA INGGRIS 2017/ 2018

21. Sutraman's suggestion is related to his concern


that .... 22. "The number of damaged roads linking Jakarta
A. many roads in Jakarta and West Java are with West Java ..."
badly damaged The underlined word is closest in meaning to
B. many complaints are heard every day in ….
the mass media A. bridging
C. the traffic is badly affected by the bad B. adjusting
road conditions C. connecting
D. road repairs may not be conducted D. cooperating
appropriately E. relating
E. the media continually report the damaged
roads

The following text is for questions 23 to 25.

Neymar da Silva Santos Junior born 5 February 1992, commonly known as Neymar, is a Brazilian
footballer who plays for La Liga club FC Barcelona and the Brazilian National Team, as a forward or winger.
At the age of 19, Neymar Jr won the 2011 South American Footballer of the Year award, after coming
third in 2010. He followed this up by winning it again in 2012. In 2012 Neymar received nominations for the FIFA
Ballon d'or, where he came 10th, and the FIFA Puskas Award, which he won. He is known for his acceleration,
speed, dribbling, finishing and ability with both feet. His playing style has earned him critical acclaim, with fans,
media and former players drawing comparison to former Brazil
player Pele, who has called Neymar "an excellent player".
Neymar joined Santos in 2003 and, after through the ranks, he was promoted to the their first team
squad. He made his debut for Santos in 2009, and was voted the Best Young Player of the 2009 Campeonato
Paulista. Further honours followed, with Neymar being voted best player as Santos won the 2010 Campeonato
Paulista, and also being top score in the 2010 Copa de Brasil with 11 goals. He finished the 2010 season with
42 goals in 60 games as his club achieved the Double. Neymar was again voted best player of the year in 2011
as his side retained the state title and Santos also winning the 2011 Copa Libertadores in which Neymar scored
6 goals in 13 appearances. He also piayed a key role in securing a Continental Double for his team, Santos' first
since 1963. He received the Bronze Ball in the 2011 FIFA Club World Crp, with Santos making it to the final,
where they were defeated 4-0 by Barcelona.

23. What happened to Neymar in 2011?


A. He made his first debut.
B. He moved to FC Barcelona.
C. He received the FIFA Puskas Award.
D. He scored 6 goals in 13 appearances.
E. He helped Brazil to win their 11th title.

24. The main idea of paragraph 3 is . . .


A. Neymar played for Santos.
B. Neymar joined Santos.
C. Neymar played the best.
D. Neymar received award.
E. Neymar scored many goals.

25. ". . . . who has called Neymar " an excellent player " (Paragraph 2)
The underlined word is closest in meaning to ….
A. confident
B. fantastic
C. magnificent
D. skillful
E. generous

SMA Alfa Centauri Bandung Halaman 157


MODUL PERSIAPAN UN & SBMPTN BAHASA INGGRIS 2017/ 2018

The following text is for questions 26 to 29.

Solar energy evaporates exposed water from seas, lakes, rivers and wet soil, the majority of this
evaporation takes place over the seas. Water is also released into the atmosphere by the plants through
photosynthesis. During this process, known as
evapotranspiration, water vapour rises into the atmosphere.
Clouds are formed when air becomes saturated with water vapor. The two major types of cloud
formation are stratified or layered grey clouds called surutus and following white or dark grey cloud called
cumulus clouds.
Precipitation as rain or hail ensures the heated water returns to Earth's surface in a fresh form. Some of
this rain, however, falls into the seas and is not accessible to humans. When rain falls, it either washes downhill
slopes or seeps underground, when snow and hail melt, this water may also shrink into the ground.
Rain fall also replenishes river water supplies so does underground water. Snow fall may consolidate
into glaciers and ice sheet which, when they melt, release their water into the ground, into streams or into the
seas.

26. The text tells about …. 28. Evapotranspiration is a process when water is
A. types of cloud released into the atmosphere by ….
B. how clouds form A. the seas
C. cloud evaporation B. the streams
D. how water vapor rises C. the rivers
E. how water cycle works D. the wet soils
E. the vegetations
27. Why does water return to earth's surface in a
fresh form? 29."Clouds are formed when air becomes saturated
A. It has been evaporated. with water vapor." (Paragraph2)
B. It is full of water vapour. The word "saturated" has the same meaning as
C. It is kept in the atmosphere. ….
D. It is the form of precipitation. A. full
E. It is kept in the form of clouds. B. dry
C. dirty
D. heavy
E. overload

The following text is for questions 30 to 32.

Boarding School Education


While many parents send their children to public schools, there are some other people choose boarding
school education. They prefer their children are educated there for some reasons.
At an early age interacting and communicating with people is very important for a child's personal life
and can be especially helpful for his/her future. In a boarding school, shy children can take advantage of
interaction through communal activities. The boarding school also offers a great variety of activities such as arts,
sports, and music that allow the children to demonstrate and develop specialized skills in their free time.
Furthermore, the structured way of life and strict rules at the boarding school may help students to get
used to a well ordered way of life. The manners and social skills will help them to become more responsible and
confident, and to develop their talents in leadership. Professionally trained teachers and educators in the
boarding school can offer excellent education without the parents' constant supervision.
In conclusion, boarding schools allow students to interact communally and to do some positive
activities. Besides they also build students characters.

30. What is the text about? C. Why people don't send their children to
A. What makes boarding school differ from public schools.
public school. D. The comparison between boarding
B. Some good points of boarding schools. schools and public schools.
E. Boarding school education is the best.

SMA Alfa Centauri Bandung Halaman 158


MODUL PERSIAPAN UN & SBMPTN BAHASA INGGRIS 2017/ 2018

31. According to the text, the weakness of boarding 32. What is the main idea of the third paragraph?
school is …. A. Boarding school teaches student to
A. it provides excellent education interact with people.
B. it is inappropriate for dependent student B. Boarding school prepares student's future.
C. it can help the learners develop their C. Boarding school teaches student social
talents skills.
D. it makes the learners more responsible D. Boarding school has some professional
and confident teachers.
E. it helps the students develop their talent E. Boarding school teaches students to be
in leadership discipline.

The following text is for questions 33 to 35.

Detective Sherlock Holmes and his friend Dr. Watson back on the cinema through the 'A Game of
Shadows'. After the success of his first film in 2009, Guy Ritchie re-directed sequel of 'Sherlock Flolmes'.
A bomb exploded in Stassbourg, England soon became headlines in several newspapers. Holmes
(Robert Downey Jr.) who disguised as a beggar was busy stalking Irene Adler (Rachel McAdams), his enemy
which also his idol that brings an important package.
In the sequel of 'Sherlock Holmes this time, Professor, James Moriarty (Jared Harris) to be the most
dangerous enemy. Moriarty is not the kind of people who are hesitant to remove the lives of many people in
order to achieve the goal. Explosions in
Strassbourg in one of the result of Prof. Moriarty's creation.
Holmes is busy when his fiiend, Dr. Watson (Jude Law) is getting married, because marriage would
automatically make Holmes lost his partner in investigating the case.
A gypsys woman, Madam Sinza Heron (Noomi Rapace), helped Holmes get the adventure this time.
There was also the Holmes brother, Mycroft Holmes (Stephen Fry) who came to be an accomplice of the
detective this time.
Farce as those made by Holmes and several other characters in the film guaranteed to invite laughter.
The chemistry between Robert Downey Jr. and Jude Law is not doubted anymore. Since in the first film, the two
men are able to captivate the audience and not be missed.
Chess competition between Holmes and Prof. Moriarty became the ultimate point. They both described
the mind wanders respectively. Do you intrigue with the acting of the detective? The film 'Sherlock Holmes: 'A
Game of Shadows' can be an alternative entertainment at this weekend.

33. The text is about ….


A. A review of "A Game Of Shadows" movie
B. The description of Holmes' life as a Detective
C. An introduction of "Sherlock Holmes" movie
D. A discussion about Robert Downey Jr, and Jude Law's lives
E. A report of the adventure of a gypsy woman

34. According to the text, the good thing of the film is that ….
A. it shows the readers how to do the investigation
B. the first film was a sign that the further film was also interesting to see
C. the farce of characters is guaranteed
D. the characters feel sure of their performance to persuade to see
E. it is still arguable that the visual effect can influence the readers

35. Who created explosions in Strassbourg?


A. Robert Downey Jr.
B. Sherlock Holmes.
C. Prof. Moriarty.
D. Dr. Watson.
E. Jude Law.

SMA Alfa Centauri Bandung Halaman 159


MODUL PERSIAPAN UN & SBMPTN BAHASA INGGRIS 2017/ 2018

The following text is for questions 36 to 38.

When author Nicholas Sparks sat down to write The Notebook, a tender love story inspired by the
enduring relationship of his wife Cathy's grandparents, he wanted his readers to walk away with a renewed spirit
of hope.
"I'll never forget watching those two people flirt," he recalls. "I mean, you don't see that very often.
They'd been married 67 years, and yet they still loved each other. I wanted to write a book about that kind of
love. I wanted people to know that unconditional love does exist.
So Sparks created The Notebook, the simple story of Noah Calhoun, a soft spoken North Carolina
outdoorsman who carried his love for the willowy Allie Nelson with him along after their youthful romance had
ended. He paralleled Noah's silent passions with Allie's haunting thoughts-feelings she could not escape even
after she became engaged to another man. He asked his readers to consider what it might mean if these
relatively huppy, middle-aged people found their destinies once again overlapped.
He presented questions all but universal in appeal: what would happen if two people were given a
second chance at the love of a lifetime.
Sparks deftly answers that question. But it's the inspiration drawn from his real life grandparents that
makes The Notebook more than just a novel of flames reignited. The novel opens and closes with an elderly
Noah Calhoun reading aloud from his personal journals and "notebook". And as he shares the delicate details,
the good with the bad, it's clear he is as enchanted with Allie in old age as he was on the day they met.
"And that's the legacy of The Notebook," according to Nicholas Sparks. "When love is real, it doesn't
matter what turns the road takes. When love is real, the joys and possibilities are endless.

36. Who has inspired the writer? D. hatred


A. An outdoorsman. E. nifty
B. Nicholas Sparks.
C. Cathy's grandparents. 38. "He presented questions all but universal in
D. Allie Nelson. appeal: ..." (Paragraph 4)
E. Noah Calhoun. The underlined word has the same meaning
with ....
37. According to the novel reviewed, the A. asked
characteristic of Noah is …. B. conveyed
A. meticulous C. thought
B. loving D. shared
C. fragile E. gave

The following text is for question 39.

The impact of globalization visibly and largely affects the politics and the economy of the country but its effect on
the mindset and the culture is noticeable gradually in the way people think and react. Some of these good and
bad points of this worldwide phenomenon are obvious from some points. People favoring globalization state that
there is a worldwide market for the companies and for the customers there is a better access to products from
different countries. Also, politics is merging and decisions that are being taken are actually beneficial to people
all over the world. Cultural intermingling increases and every nation tries to know more about the other nations'
cultural preferences. In this process, we are actually coming across things that we like and in the course of time
adopt it. However, here are also opponents of globalization. There is immense pressure on the employed people
of developed countries who are always under the threat of their jobs being outsourced. Globalization may lead to
loss of cultural identity as western ideas are always imposed upon the eastern thoughts.

39. How can globalization be beneficial culturally?


A. Cultural adaption is more open and inevitable.
B. Developed countries realize the importance of education.
C. Political ideas inspire people for a more democratic unity.
D. Exchanges of western and eastern cultures become intensive.
E. People of different cultures are aware of others likes and dislikes.

SMA Alfa Centauri Bandung Halaman 160


MODUL PERSIAPAN UN & SBMPTN BAHASA INGGRIS 2017/ 2018

This text is for questions 40 and 41.

The Graduation Standard


The implementation of the new graduation standard for High-School Students has evoked people to
bring different responses. Some people's ideas are for some others against the rule.
Some people agree with the rule because it indicates students' mastery of the whole material or the
competence they have managed to have over the past three years. The result of the final exam is also significant
for students' future study in the higher level. In addition, we are far left behind by our neighboring countries that
have implemented a higher graduation grade standard. If the standard is omitted, then student will get left
behind even further by those students from neighboring countries students.
However, some other people believe that imposing a high standard will not be effective. The results of
the exams have proven to be unreliable. How can you assess the students' competence only in a couple of days.
A smart student for example, may get a low grade just because he is sick during the exam. On the other hand,
an idle student may achieve the standard due to luck. There should then be an alternative way to the national
exam.
Despite working on different points of view, the government's relating party are intensely working on this
matter in the hope of getting a better solution upon improving the High-School graduates' quality.

40. What is the writer's position in the controversy 41. "The implementation of the new graduation
of graduation standardization? standard for High School has evoked people to
A. The writer does not take side. bring different responses" (Paragraph 1)
B. The writer follows the government's policy. The underlined word is closest in meaning to
C. The writer is strongly against it. ….
D. The writer supports it. A. created
E. The writer opposes it. B. reminded
C. examined
D. gained
E. got

This text is for question 42.

What is the water pollution? Water pollution is what happens when factories, wastewater treatment plants,
construction sites, and people put things in the water that make it dirty. At one time, factories dumped untreated
waste into the water. This is an example of water pollution. Not only does water pollution come from what is
dumped into the water, but what is dumped on the land. When you put trash in a storm drain, toilet or in a body
of water (rivers, lakes, streams, oceans) it causes the water to be polluted. Just think, if someone were changing
their oil on their car and they dumped the oil on the ground, what would happen? Would you want to eat the fish
that came from a waterway where you knew that oil went? Even today, accidents on ships and off-shore drilling
rigs spill oil into the oceans. Animal waste run off from livestock feedlots seeps into groundwater, lakes, rivers
and streams that eventually make it to the ocean. Fertilizers and pesticides wash off from fields and forests and
soak into lakes, rivers, streams and groundwater. Wastes from mines drain into water. Garbage disposals and
toilets are also a large waste contributor.

42. The main idea of the text is ….


A. sources of water pollution are varied
B. some materials are potential pollutants
C. water pollution is caused by many factors
D. human activities are main causes of pollution
E. water pollution is a body of water affected by impurities in it

SMA Alfa Centauri Bandung Halaman 161


MODUL PERSIAPAN UN & SBMPTN BAHASA INGGRIS 2017/ 2018

This text is for questions 43 and 44.

The Red Bird of Paradise


An Indonesian endangered species, the Red Bird of Paradise is distributed to lowland rainforests of Waigeo and
Batanta islands of West Papua. This species share its home with another bird of paradise, the Wilson's Bird of
Paradise. Hybridization between these two species are expected but not recorded yet.
The Bird of Paradise, Paradisaea rubra, is large, up to 33cm long, brown and yellow bird with a dark brown iris,
grey legs and yellow bill. The male has an emerald green face, a pair of elongated black corkscrew-shaped tail
wires, dark green feather pompoms above each eye and a train of glossy crimson red plumes with whitish tips at
either side of the breast.
The male measures up to 72 cm long, including the ornamental red plumes that require at least six years to fully
attain. The female resembles the male but is smaller in size, with a dark brown face and has no ornamental red
plumes. The diet consists mainly of fruits, berries and arthropons.

43. We know from the text that .... 44. "... with a dark brown iris, grey legs and yellow
A. the female Red Bird of Paradise is bigger bill ..." (Paragraph2)
than the male The underlined word is closest in meaning to
B. the Red Bird of Paradise is rare ….
nowadays. A. wing
C. the Red Bird of Paradise cannot live alone B. tail
D. the Red Bird is as large as the female C. breast
E. the Red Bird of Paradise lives in highland D. eyes
of West Papua E. beak

This text is for questions 45 and 46.

We are announcing today that we are bringing the Milestone and Ever Green brands even closer together.
Effective as of 5th December 2013, our official name will be:
GREEN MILES WEST
The substitution of "West" in our name---replacing "Cianjur"---is the result of an agreement we reached with
Cianjur Gardening Association, following a protest over the original use of "Cianjur" in our name.

We hope this does not create any confusion among our loyal consumers. While this represents a change from
our initial name introduction, it does not change the quality of products we offer to our consumers.

45. Who issues the announcement? 46. "The substitution of "West" in our name …."
A. Green Miles West Company. The word "substitution" is closest in meaning to
B. Green Miles West Customers. ….
C. The co-worker of Green Miles West. A. merger
D. The partnership of Green Miles West. B. insertion
E. The loyal customers of Green Miles West. C. development
D. replacement
E. improvement

SMA Alfa Centauri Bandung Halaman 162


MODUL PERSIAPAN UN & SBMPTN BAHASA INGGRIS 2017/ 2018

47. Arrange the following jumbled sentences into a meaningful paragraph!


1. Finally, one of the frogs listened to what the other frogs were saying and gave up. He fell down and
died
2. When the other frogs saw how deep the pit was, they told the two frogs that they were as good as dead.
The two frogs ignored the comments and tried to jump up out of the pit with all their might.
3. The other frogs continued to jump as hard as he could. Once again the crowd of frogs yelled at him to
stop the pain and just die.
4. The other frogs kept telling them to stop, that they were as good as dead.
5. He jumped even harder and finally made it out. When he went out, the other frogs said, "Didn't you hear
us: the frogs explained to them that he was deaf. He thought they were encouraging him the entire time.
6. A group of frogs were travelling through the woods, and some of them fell into a deep pit.

The good arrangement is ….


A. 6-2-4-3-1-5
B. 6-2-4-1-5-3
C. 6-2-4-1-3-5
D. 6-2-1-4-3-5
E. 6-2-1-3-4-5

Questions 48 to 50 based on the following cloze test.


Octopuses are mollusks, a kind of animal with a soft body. Unlike other mollusks, such as clams and oysters,
octopuses and squid have no hard .... (48) to protect them. An octopus is an animal without any bones.
Surrounding the main portion of its body is a fleshy covering, called a mantle. Most of the internal organs of an
octopus are inside the mantle. An octopus has two big ... (49), so it has very good vision. Seals, eels, and other
sea animals prey on octopuses. An octopus' main method of defense is to shoot a cloud of dark ink into the
water. The ink cloud confuses the attacker, and the octopus jets away. Octopuses can also … (50) color rapidly
when they are in danger. They change color to fit in with their surroundings. This helps them to hide from prey.

48. ….
A. skin
B. scales
C. seals
D. shells
E. skull

49. ….
A. eyes
B. bones
C. fingers
D. thumbs
E. shoulders

50. ….
A. complain
B. wonder
C. deliver
D. change
E. breathe

SMA Alfa Centauri Bandung Halaman 163


MODUL PERSIAPAN UN & SBMPTN BAHASA INGGRIS 2017/ 2018

UJIAN NASIONAL SMA/MA


TAHUN 2015
Listening Section
In this section of the test, you will have the chance to show how well you understand spoken English. There are
four parts to this section with special directions for each part.

PART 1
Questions 1 to 4.

Directions:
In this part of the test, you will hear some dialogues and the questions will be spoken twice. They will not be
printed in your test book, so you must listen carefully to understand what the speakers are saying.
After you listen to the dialogue and the question about it, read the five possible answers, and decide which one
would be the best answer to the question you have heard. Now listen to a sample question.

You will hear:


Man : I don‘t know what to order. I could drink everything on the menu.
Woman : Why don‘t you try guava juice, orange juice or ice tea?
Man : Guava juice sounds good. I‘ll take it.
Woman : I think I‘ll have a big glass of cola float.

Narrator : What will the woman do?


A. Take a guava juice.
B. Order an orange juice.
C. Try an ice tea.
D. Order a cola float.
E. Drink mineral bottled water.

Narrator : The best answer to the question ‖What will the woman do?‖ is order a cola float. Therefore you
should answer (D).

1. …. 3. ….
A. The woman‘s dress A. One week
B. The elegant woman B. Two weeks
C. Woman‘s black dress C. Four days
D. The woman‘s appearance D. Three days
E. The man‘s complaint about the woman E. Seven days

2. …. 4. ….
A. Sick A. She is discussing the cause of heart
B. Tired attacks
C. Sleepy B. She is finishing her report on heart attacks
D. Thirsty C. She is finishing her school project
E. Hungry D. She is presenting her assignment
E. She is looking for a scholarship

PART II
Questions 5 to 7.

Directions:
In this part of the test, you will hear some incomplete dialogue spoken in English, followed by four responses,
also spoken in English. The dialogues and the responses will be spoken twice. They will not be printed in your

SMA Alfa Centauri Bandung Halaman 164


MODUL PERSIAPAN UN & SBMPTN BAHASA INGGRIS 2017/ 2018

test book, so you must listen carefully to understand what the speakers are saying. You have to choose the best
response to each question.

Now listen to a sample question.


Woman : You arrived late again.
Boy : I‘m sorry, Ma‘am. The bus had already left when I arrived at the shelter.
Woman : But you promised that you wouldn‘t come late again.
Boy : ….

Narrator : What does the boy probably respond?


A. I‘m sorry to hear that.
B. I really appreciate it.
C. I do apologize Ma‘am.
D. No problem, Ma‘am.

Narrator : The best answer to the question ―What does the boy probably respond?‖ is ―I do apologize
Ma‘am‖. Therefore you should choose answer (C).

5. Mark your answer on your answer sheet.


6. Mark your answer on your answer sheet.
7. Mark your answer on your answer sheet.

PART III
Questions 8 to 11.

Directions:
In this part of the test, you will hear some dialogues or monologues spoken in English. The dialogues or
monologues will be spoken twice. They will not be printed in your test book, so you must listen carefully to what
the speakers are saying. After you listen to the dialogue or monologue, look at the five pictures provided in your
test book, and decide which one would be the most suitable one for the dialogue or monologue you have heard.

8. …. 10. ….
A. B. C. A. B.

D. E. C. D.

9. . E.
A. B. C.

D. E.

SMA Alfa Centauri Bandung Halaman 165


MODUL PERSIAPAN UN & SBMPTN BAHASA INGGRIS 2017/ 2018

11. ….
A. C. D.

B.
E.

PART IV
Questions 12 to 15.

Directions:
In this part of the test, you will hear several monologues. Each monologue will be spoken twice. They will not be
printed in your test book, so you must listen carefully to understand what the speakers are saying.
After you hear the monologue and the question about it, read the five possible answers and decide which one
would be the best answer to the question you have heard.

Question 12 and 13 are based on the following description.

12. … 13. …
A. A military city. A. Ro build new railway museums.
B. A railway museum. B. To transport government troops.
C. A new rail way town. C. To preserve the steam locomotives.
D. King Willem I station. D. To build a new government station.
E. Steam locomotives station. E. To park the locomotives at the original
station.

Question 14 and 15 are based on the following monologue.

14. … 15. …
A. The cooking pot. A. The family were hungry.
B. The stinginess. B. It was the last rice they had.
C. The new place. C. He didn‘t want the rice to be stolen.
D. The wealth. D. The rice was still fresh from the pot.
E. Rice. E. Pak Pesut wished to share with the
villagers.

This is the end of the listening section

SMA Alfa Centauri Bandung Halaman 166


MODUL PERSIAPAN UN & SBMPTN BAHASA INGGRIS 2017/ 2018

Reading Section

This text is for questions 16 and 17.

PT Sinar Indah
Jln. Jenderal Sudirman 96-97
Palangka Raya, Kalimantan

January 15, 2015


Jln. Gatot Subroto 105
Semarang

Dear Sir,

I‘m writing to inform you that we are unable to make delivery of your purchase order dated on January
10, 2015.
We usually have our merchandise ready to ship within 10 days so that you will have received your order
on January 20, 2015. But, since we have not received the merchandise, we hope you can hold off until January
25, 2015.
However, we can assure you that if your order remains in force we will expedite delivery to you as soon
as we have received the merchandise.
Please accept our apology for this delay and thank you for your understanding.
Sincerely Yours,

Deddy Ilham Salman


Manager Order Department

16. Why did Mr. Deddy Ilham Salman send the 17. From the text, we infer that …
letter to PT Interkawula? A. PT Interkawula feels dissatisfied with the
A. Cancelling the merchandise. service
B. Receiving the merchandise. B. PT Sinar Indah feels satisfied with the
C. Making the delivery of the purchase order. customers
D. Informing the delivery of the purchase C. Mr. Deddy Ilham Salman is care of the
order. customers
E. Apologizing for the delay of delivering the D. PT Interkawula is patient with the order
purchase order. E. Mr. Deddy is a responsible manager

This text is for questions 18 and 19.


Kristin’s Babysitting Service
Red Cross Babysitting Certified
4 H Baby Massaging Certified
Dept. of Health Baby Sitting Certified
Merc Baby Bathing Certified
Experiences with infants and toddlers
Rates:
One child: $5.00 per hour
Two chidren: $7.00 per hour
$1.00 per each additional child
Book for one year, and have 5% discount
Please call to schedule a year night out;
Home: 6278989090
Cell: 0893738839

SMA Alfa Centauri Bandung Halaman 167


MODUL PERSIAPAN UN & SBMPTN BAHASA INGGRIS 2017/ 2018

18. What is the text about? 19. Based on the text, Kristin has been certified
A. Baby sitting service advertisement. from ….
B. Service rate announcement. A. Red Cross for Baby Massaging
C. A babysitting schedule. B. Red Cross for Baby Sitting
D. Certified Baby Sitters. C. 4 H for bathing baby
E. Infants and toddlers. D. Merc Baby Sitting
E. Dept. of Health

This text is for questions 20 to 22.

JAKARTA: National flag-carrier Garuda Indonesia begins its new flight service on Monday morning in an effort to
expand its international networks.
―We believe the opening of this route will further develop economics, business, social and tourism
activities between Indonesia and UK‖, Garuda sales and marketing director Erik Meijer said in a statement made
available to the Jakarta Post on Monday, adding that the Jakarta – London route would make a stop in
Amsterdam, the Netherlands.
For the service, the carrier will operate a wide-bodied Boeing B777-300 Extended Range (ER) aircraft,
which has the total capacity of 314 seats with the following configuration: 8 first-class seats, 38 business-class
seats, and 268 economy-class seats. First class passenger will get the privilege of having their meals cooked by
a chef on board.
The aircraft is also equipped with ―In-flight Connectivity‖ that includes exConnect facilities (internet
connection via WiFi) and Live TV (live telecasts from prominent international channels such as CNN
International, BBC World, BBC Arabic, NHK World Premium and Euronews).
Erik said that the airline would fly to Gatwick International Airport from the country‘s main gateway,
Soekarno-Hatta International Airport, five times a week.
(The Jakarta News Business)

20. What is the text about? B. increase


A. The marketing sales for international C. substract
networks. D. eliminate
B. The promotion of Boeing B777-300 E. deteriorate
Extended Range.
C. Jakarta – London route in a wide-bodied 22. What is the main idea of paragraph 2?
Boeing B777. A. Erik Meijer stated the news to Jakarta
D. The launching of Garuda‘s service from Post.
Jakarta to London. B. The flight to London will depart on Monday
E. The purpose of developing economic, morning.
Business, and social activities. C. The UK‘s tourism business will develop in
Indonesia.
21. From the text, we imply the opening Jakarta – D. The flight from London will stop at airport
London route will … the relationship between in Amsterdam.
UK and Indonesia in many financial sector. E. The opening of the route is aim at
A. decline developing relationship in some sectors.

This text is for questions 23 to 25.

Barcelona: Lionel Messi preffered to focus on winning trophies rather than attempting to better the 73
goals he scored for Barcelona last season, when he spoke at a news conference on Wednesday.
The world player of the year rattled in an unprecedented tally in all competitions in the last campaign,
helping Pep Guardiola end his four-year tenure as coach with another four trophies to add to the 10 he had
already won.
―I‘m not looking to beat my tally of goals, that is not my objective‖, Messi said. ―I want to try and develop
as a player and to give more to the team. I hope I still have margin to continue improving. My objective is to win
more titles‖.

SMA Alfa Centauri Bandung Halaman 168


MODUL PERSIAPAN UN & SBMPTN BAHASA INGGRIS 2017/ 2018

Real Madrid and Chelsea denied Barca the major prized last year, Jose Mourinho‘s side winning La
Liga and Chelsea knocking them out of the Champions League in the semi-finals on their way to a final victory
over Bayern Munich.
―I hope we can get back to winning everything again‖, he added.
―We know it is very difficult to do, but I am still hurting from losing the semi-final against Chelsea with
the penalty I missed, and La Liga with the defeat to Madrid at the Nou Camp‖. (News: Sport)

23. The text is about …. D. Real Madrid had draw with Barca in this
A. A final victory over Bayern Munich competition.
B. Barcelona, the major prize last year E. Barca neither won in champion league and
C. Messi‘s ambition to win more titles La Liga.
D. The defeat to Real Madrid at the Nou
Camp 25. From the text, we know that ….
E. The match between Real Madrid and A. Messi and Pep. Guardiola had made
Chelsea Barcelona an unbeatable team of the
world
24. Why did Chelsea deny Barca major prizes last B. Bayern Munchen and Real Madrid were
year? the two teams who defeated Barcelona
A. Chelsea didn‘t win in the championship C. Pep. Guardiola coached Barcelona for four
because of Barca‘s competence and luck. years and won 10 trophies
B. Real Madrid and Chelsea worried to Barca D. The only team which can beat Barcelona
to become the championship. is Chelsea from England
C. Barca won the championship because of E. Barcelona won in champion league and La
suicide goal. Liga

This text is for questions 26 to 28.

Lionel Messi (Luis Lionel Andres Messi) was born on June 24, 1987, in Rosario, Argentina. As a young
boy, he tagged along when his two older brothers played soccer with his friends, unintimidated by the bigger
boys. At the age of 8, he was recruited to join the youth system of Newell‘s old Boys, a Rosario-based club.
Recognizably smaller than most of the kids in his age group, Messi was eventually diagnosed by doctors as
suffering from a hormone deficiency that restricted his growth.
Messi‘s parents decided on a regimen of nightly growth-hormone injections for their son, though it soon
proved impossible to pay several hundreds dollars per month for the medication. So, at the age of 13, when
Messi was offered the chance to train at soccer powerhouse FC Barcelona‘s youth academy, La Masia, and
have his medical bills covered by the team, Messi‘s family picked up and moved across the Atlantic to make a
new home in Spain.
By the age of 16, Messi had made his first appearance for Barcelona. He put himself in the record
books on May 1, 2005, as the youngest player ever to score a goal for the franchise. Messi steered Barcelona to
a wealth of success, most notably in 2009. Amazingly, the diminutive soccer wizard continue to improve,
discovering new ways to elude defenders while leading Barcelona to La Liga and Spanish Super Cup
championship in 2010 and 2011, as well as the Champions League title. Messi embarked on an all-out assault
on the record books in 2012. He became the first player to score five goals in a Champion League match in early
March, and a few weeks later he surpassed Cesar Roriguez‘s club-record 232 goals to become Barcelona‘s all-
time leading scores. By the end of 2012, Messi had accumulated an astounding 91 goals in club and
international play. Fittingly, he broke one more record when he was named the FIFA Ballon ‗Or winner for the
fourth time in January 2013.

26. What is the text about?


A. FIFA Ballon d‘Or Winner.
B. Lionel Messi‘s life in brief.
C. Messi‘s reputation in FC Barcelona.
D. Messi‘s reputation in Football match.
E. The Messi‘s recovery from his hormone deficiency.

SMA Alfa Centauri Bandung Halaman 169


MODUL PERSIAPAN UN & SBMPTN BAHASA INGGRIS 2017/ 2018

27. Why did Messi leave for Spain?


A. The doctor diagnosing him was in Spain. 28. From the text, we conclude that Messi‘s … will
B. He had the manager pay the medical bills. lead him to be the one of the most famous
C. He wanted to perform his appearance in soccer player in the world.
Barcelona. A. fortune
D. He moved across the Atlantic to make a B. biography
new home in Spain. C. personality
E. He was given the opportunity to be trained D. achievement
at a soccer academy. E. competition

This text is for questions 29 to 31.

The Effects of Acid Soil

Soil with a pH of less than 7.0 are acid. The lower the pH, the more acid the soil. When soil pH falls
below 5.5, plant growth is affected. Crop yields decrease, reducing productivity.
Soils provide water and nutrients for plant growth and development. Essential plant nutrients include
phosphorus, nitrogen, potassium, and sulfur. Plants require other elements such as molybdenum, in smaller
quantities. Some elements e.g. aluminium and manganese, are toxic to plants.
Nutrients become available to plants when they are dissolved in water. Plants are not able to take up
phosphorus, nitrate, potassium, and sulfate ions in solution.
The solubility of nutrients changes with pH. In acid soils, (low pH), molybdenum become less soluble
and aluminium becomes more soluble. Therefore, plant growth may be affected by either a deficiency of
molybdenum or too much aluminium.
Both crop and pasture plants are affected by acid soils. There may be a range of symptoms. Crops and
pastures may be poorly established resulting in patchy an uneven growth. Plant leaves may go yellow and die at
the tips. The root system of the plant may be stunted. Crop may yield less.
Plants vary in their sensitivity to low pH. Canola and Lucerne are very sensitive to acid soils so they do
not grow well. Lupins and triticale are tolerant to soils of low pH so they still perform well.
Land can become unproductive if acid soil is left untreated. Incorporating lime into the soil raises the pH.
Therefore, liming soil can reverse the effects of acid soil on plants and return a paddock to productivity.

29. What is the advantage of soils with a pH of less E. Crops and pastures are established
than 7.0? resulting in patchy.
A. To reduce productivity of crop.
B. To increase the crop productivity. 31. ―Therefore, plant growth may be affected by
C. To make the leaves yellow. either a deficiency of molybdenum or too much
D. To be poorly established. aluminium‖. (Paragraph 4)
E. To cause the planet die. The underlined word is closest in meaning to
….
30. What is the main idea of the fifth paragraph? A. caused
A. Acid soils affect crop and plants. B. supported
B. The root system of plant is stunted. C. destroyed
C. Plant leaves turn yellow and die at the tips. D. empowered
D. Crop and pasture plants are not affected E. influenced
by acid soils.

This text is for questions 32 to 34.


There are a lot of discussions as to whether children should be given homework or not.
Some people claim that children do enough work in school already. They also argue that children have
hobbies that they want to do after school, such as sports or music. A further point they make is that a lot of
homework is pointless and doesn‘t help the child learn at all.

SMA Alfa Centauri Bandung Halaman 170


MODUL PERSIAPAN UN & SBMPTN BAHASA INGGRIS 2017/ 2018

However, there are also strong arguments against this point of view. Parents and teachers argue that it
is important to find out whether children can work on their own without the support from the teacher. They say
that the evening is a good time for children to sit down and think about what they have learned in school.
Furthermore they claim that the school day is too short to get anything done that needs doing and it
makes sense to send home tasks like independent reading or further writing tasks that don‘t need teacher‘s
support.
On balance, I think that some homework is a good idea but that it should only given at the weekend
when children have more time.

32. By giving the homework to the students, they C. the importance of the homework
can … in the evening. D. the weakness of giving the homework
A. prepare for the next lesson E. the benefit of the homework
B. review their lessons
C. enjoy their past time 34. What is the writer‘s suggestion about
D. do their hobbies homework?
E. test themselves A. Homework is pointless.
B. Homework is badly needed.
33. The third paragraph discusses … for the C. Homework should be given on weekend.
students. D. Student must frequently have homework.
A. the disagreement of giving the homework E. Student should not be given any
B. the function of doing the homework homework.

This text is for questions 35 to 37.

JUMP – Comic Martial Arts performance


By Yegam Korsa

Jumpstart your entertainment experience with a new brand of theatrics this June! Combining physical
comedy and Tae Kwon Do, Jump is a high-octane martial arts extravaganza from Korea that features gravity-
defying stunts such as slick acrobatics, sword fighting and wall-walking!
Jump tells the story of an idiosyncratic Korean family under the rule of a strict grandfather who insists on
keeping up the physical fitness of all family members through vigorous martial arts training. The performance
comes to a climax when two stupid thieves break into the house and confront three generations of kung-fu
masters, inspiring a series of ridiculous scenes.
A sell-out show in London‘s West End and the number one hit at the Edinburgh Fringe box office in
2005 and 2006, with sell-out shows in Korea, Hong Kong and Macau. Jump was also performed at Sadlers Wells
Theatre in London early this year and will tour Russia and Japan after Singapore.
Be awestruck by this exhilarating theatrical display! A truly entertaining experience for the whole family,
Jump is unique, funny and utterly spectacular.
―Go and see this show and take all your family and friends too. They will love you forever‖. – The British
Theatre Guide.

35. The text is about …. 36. What is Jump?


A. Entertaining the unique of comic A. It is a series of ridiculous scenes.
B. Reviewing the Jump-Comic Martial Arts B. It is unique, funny and utterly spectacular.
C. Performing the saddlers wells Theatre in C. It is a climax when two stupid thieves
London break into the house.
D. Telling the story of an idiosyncratic Korean D. It is a truly entertaining experience for the
family rest of the family.
E. Combining physical comedy and Tae E. It is a high-octane martial arts
Kwon Do extravaganza from Korea featuring gravity-
defying stunts.

SMA Alfa Centauri Bandung Halaman 171


MODUL PERSIAPAN UN & SBMPTN BAHASA INGGRIS 2017/ 2018

37. From the text, we can imply that Jump …. C. is a visual activity performing sword
A. is an interesting martial art performing not fighting only
only physical ability but also self-defense D. is not a good show because it does not
performances express anything
B. is not a comedy activities but it is only self- E. is a good martial art
defense performances

The following text is for questions 38 to 40.

Internet has been the most outstanding invention in the history of mankind. With internet, our earth has
become a global village. Yet, like many other human made technologies, it has its own advantages and
disadvantages.
There is plenty of software that can entertain us or help us to communicate with our beloved ones. In a
mere second, we can chat for hours with a person who is sitting at the other parts of the world, read our e-mail in
our smart phone or listen to our favorite radio broadcast by streaming it on our tablet.
With its easy access, internet also has its advantages. One of them is children may be exposed to
violence and explicit contents that only suitable for adults. There are thousands of such inappropriate contents
on the internet which can be easily found. It is a very serious issue and may harm children well-being.
Although, internet can create havoc and destruction, its advantages are more important that its
disadvantages.

38. What is one of the disadvantages of the C. People should not use internet
internet? D. Internet can make us lazy
A. It exposes children to violence. E. Internet brings more harm than good
B. It helps us chat with our beloved ones.
C. It helps us communicate from faraway. 40. From the passage we can assume that children
D. It streams our favorite radio broadcast. should be … by their parents while they are
E. It sends to our e-mail from our smart using the internet.
phone. A. taught
B. watched
39. The main idea of paragraph 2 is …. C. seen
A. The use of internet can be very fatal D. accompanied
B. Internet has some advantages E. observed

This text is for questions 41 to 43.

The Yaks

A yak is a wild ox of Asia. It inhabits the cold, dry plateaus of Tibet, often more than 4.870 meters above the sea
level.
Adult male yaks grow to an average height of two meters at the shoulder, and weigh about 1000 kg and
the females weigh a third of that.
A yak is covered with long and shaggy hair. It has a dense undercoat of soft, close matted hair. Wild
yaks can be brown or black, and both males and females have horns. Wild yaks secrete a special sticky
substance in their seat that helps keep their under-hair matted. This substance is also used in traditional
Nepalese medicine.
The indigenous tribes of the Tibetan plateaus have domesticated the yaks for many years. They rely
heavily on them for transportation and as pack animals because they are very strong and able to maneuvers
along dangerous ice and snow-covered land.
The domesticated yaks are also important to farmers as it is used to plough fields. Its meat and milk are
widely consumed. The soft hair is used to make clothes. Saddles, whips, boots, and other articles are made from
the hide. The bushy tail is used as fly chaser at ceremonial processions in India.

SMA Alfa Centauri Bandung Halaman 172


MODUL PERSIAPAN UN & SBMPTN BAHASA INGGRIS 2017/ 2018

41. According to the text, we know that …. B. Abundant of milk and thick hide
A. Wild yaks have sweat on their hair C. Great strength and heavy weight
B. Male yaks are twice bigger than females D. A dense close matted hair undercoat
C. Wild yaks are unable to live on snow E. Ability to maneuver the snow covered land
covered land
D. The color of wild yaks can be black, grey 43. What is the main idea of the fifth paragraph?
and brown A. The yak likes to hide.
E. Sticky substances in yaks sweat is useful B. The farmers drink milk.
for traditional medicine C. The yak is very advantageous to mankind.
D. Farmers domesticate the yaks for its meat.
42. Having … help(s) the yak to adopt itself to the E. A fly chaser is used in ceremonial
cold. processions.
A. A special sticky substance

The following text is for questions 44 and 45.

Company’s Name Change

Japan Telkom co, Ltd (Head Office: Minato_ku, Tokyo: President and CEO: Hideki Kurashige:
hereinafter ―Japan Telecom‖) announces that on October 1st, 2014 it will change its name to ―Softbank Telecom
Corp‖ and the corporate logo will change to the design used by Softbank Group Companies.
Softbank Group is a comprehensive I.T network infrastructure, digital platform and content company.
With this collective strength, our company offers innovative, board solution which contributes to the business
success of our customers.
With this company change, we aim to maximize group synergies to further build the relationship and
trust we have with our existing customers. We will use the power of the Softbank Group companies to develop
our company‘s strength.

44. What happened to the company on October 1st, 3. Set aside in a room temperature for 6
2014? hours.
A. It changed its name to Softbank Telecom 4. Remove from baking sheet to a wire tack
Corp. to cool.
B. It built further relationship with Japan 5. Bake at 350°F for 6 minutes.
Telkom. 6. Beat eggs, sugar and vanilla extract in a
C. It developed its company infrastructure mixing bowl until light-colored.
strength. 7. Combine flour and baking powder set
D. It maximized its digital business platform. aside.
E. It changed the company‘s president and
CEO. The correct order arrangement is ….
A. 7 – 6 – 2 – 1 – 3 – 4 – 5
45. Softbank Group is …. B. 7 – 6 – 1 – 2 – 3 – 5 – 4
A. A technology & information company C. 6 – 7 – 1 – 2 – 5 – 4 – 3
B. The group synergies company D. 1 – 7 – 6 – 2 – 3 – 5 – 4
C. The logo design company E. 1 – 6 – 7 – 2 – 3 – 4 – 5
D. The contents company
E. Japan company 47. Arrange the following phrases or sentences into
the correct order!
46. Arrange the following phrases or sentences 1. The doctor cleaned the leg and then he
into the correct order! bandaged it.
1. Stir in the flour and baking powder 2. She telephoned her sister, and her sister
mixture. came to her.
2. Drop the butter by teaspoonful about 2 3. An old woman live alone as her husband
inches apart onto the greased and floured was dead.
baking sheet. 4. Then he said, ―This leg‘s going to be bad
for a long time‖.

SMA Alfa Centauri Bandung Halaman 173


MODUL PERSIAPAN UN & SBMPTN BAHASA INGGRIS 2017/ 2018

5. She fell down the stairs and hurt her left The correct arrangement is ….
leg. A. 3 – 2 – 5 – 6 – 1 – 7 – 4
6. Her sister took her to see a doctor. B. 3 – 5 – 6 – 2 – 1 – 4 – 7
7. Don‘t run up and down the stairs in your C. 3 – 5 – 2 – 6 – 1 – 4 – 7
house for a few weeks. D. 3 – 6 – 1 – 4 – 7 – 5 – 2
E. 3 – 1 – 5 – 4 – 7 – 2 – 6

Questions 48 to 50, complete the following text with the words provided.

Henry and Mary had just married and everybody was enjoying their wedding party. There was plenty to eat and
plenty to drink. Everybody is (48) … very cheerful, when a very thin young man come into the room. He looked at
Mary (49) … and accusingly, walked slowly towards her, kissed her lovingly, and said, ―Why did you do it?‖ Then
he walked to the (50) … and disappeared. Nobody had ever seen the young man before-not even Mary.

48. ….
A. turning
B. having
C. changing
D. getting
E. enjoying

49. ….
A. sadly
B. gladly
C. merrily
D. joyfully
E. amusingly

50. ….
A. room
B. hall
C. door
D. table
E. auditorium

SMA Alfa Centauri Bandung Halaman 174


MODUL PERSIAPAN UN & SBMPTN BAHASA INGGRIS 2017/ 2018

UJIAN NASIONAL SMA/MA


TAHUN 2016
Listening Section

In this section of the test, you will have the chance to show how well you understand spoken English. There are
four parts to this section with special directions for each part.

PART 1
Questions 1 to 4.

Directions:
In this part of the test, you will hear some dialogues and the question spoken in English. The dialogues and
questions will be spoken twice. They will not be printed in your test book, so you must listen carefully to
understand what the speakers are saying.
After you listen to the dialogue and the question about it, read the five possible answers, and decide which one
would be the best answer to the question you have heard. Now listen to a sample question.

You will hear:


Man : I highly appreciate you for helping me choose the right loan I need.
Woman : My pleasure, Sir. I hope you can use the money to buy your dream house.

Narrator : What is the topic of the conversation?


A. Asking for help.
B. Giving opinion.
C. Giving direction.
D. Expressing pleasure.
E. Expressing gratitude.

Narrator : The best answer to the question ‖What is the topic of the conversation?‖ is expressing gratitude.
Therefore you should answer (E).

Now let‘s begin with number 1.

1. …. E. The woman supported the man to be a


A. She lost her Kitty. branch manager.
B. Her kitty was dead.
C. She could get a kitty. 3. ….
D. She did not have a pet animal. A. He wants to sit down.
E. She was allowed to get a kitty. B. He recognized the woman.
C. His plane has been delayed.
2. …. D. He wants to leave the place.
A. The man felt happy to meet the woman. E. He wants to wait for a plane.
B. The woman got a better position in the
office. 4. ….
C. The woman moved to a new bank in A. To enjoy the holiday on the beach.
Samarinda. B. To camp by the lake.
D. The man got a promotion to be a branch C. To go to a sea shore.
manager. D. Nothing for now.
E. To take a vacation by plane.

SMA Alfa Centauri Bandung Halaman 175


MODUL PERSIAPAN UN & SBMPTN BAHASA INGGRIS 2017/ 2018

PART II
Questions 5 to 7.

Directions:
In this part of the test, you will hear some incomplete dialogue spoken in English, followed by four responses,
also spoken in English. The dialogues and the responses will be spoken twice. They will not be printed in your
test book, so you must listen carefully to understand what the speakers are saying. You have to choose the best
response to each question.

Now listen to a sample question.


Girl : I really made a mistake.
Boy : What about?
Girl : I have broken your pen.
Boy : That pen? Oh no.
Girl : ….

Narrator : What does the girl probably respond?


A. I‘m sorry to hear that.
B. I really appreciate it.
C. I do apologize.
D. I love that pen.

Narrator : The best answer to the question ―What does the girl probably respond?‖ is ―I do apologize‖.
Therefore you should choose answer (C).

5. Mark your answer on your answer sheet.


6. Mark your answer on your answer sheet.
7. Mark your answer on your answer sheet.

PART III
Questions 8 to 11.

Directions:
In this part of the test, you will hear some dialogues or monologues spoken in English. The dialogues or
monologues will be spoken twice. They will not be printed in your test book, so you must listen carefully to what
the speakers are saying. After you listen to the dialogue or monologue, look at the five pictures provided in your
test book, and decide which one would be the most suitable one for the dialogue or monologue you have heard.
C.
8. ….
A.

D.
B.

E.

SMA Alfa Centauri Bandung Halaman 176


MODUL PERSIAPAN UN & SBMPTN BAHASA INGGRIS 2017/ 2018

9. ….
A. D.
D.

E.
B. E.

11. ….
C. A.

B.
10. ….
A.

C.

B.

D.

C.

E.

SMA Alfa Centauri Bandung Halaman 177


MODUL PERSIAPAN UN & SBMPTN BAHASA INGGRIS 2017/ 2018

PART IV
Questions 12 to 15.

Directions:
In this part of the test, you will hear several monologues. Each monologue will be spoken twice. They will not be
printed in your test book, so you must listen carefully to understand what the speaker is saying.
After you hear the monologue and the question about it, read the five possible answers and decide which one
would be the best answer to the question you have just heard.

Question 12 and 13 are based on the following monologue.

12. …. 13. ….
A. The cure for the illness. A. Jungle plant.
B. Beautiful lady. B. The King‘s spell.
C. A lotus flower. C. The King‘s dream.
D. A rose flower. D. The beautiful flower.
E. A pond. E. Pond‘s water.

Question 14 and 15 are based on the following monologue.

14. …. B. The people inhabiting outside the


A. National disaster mitigation. dangerous zone.
B. North Sulawesi announcement. C. The people staying alert with the current
C. The eruption of Mount Lokon. situation.
D. Dangerous areas for people. D. The villager staying calm while the mount
E. Volcanic zone for living. is erupting.
E. The residents having no houses within the
15. …. dangerous zone.
A. The residents living within 2.5 kilometers
radius from the mountain park.

This is the end of listening section

Reading section
The following text is for questions 16 to 18.

Announcement
It is our pleasure to announce the promotion of Bill William as our new Sales Manager. Although Bill has been
with us for merely five years, he had shown us his full commitment, dedication and more importantly his great
achievement in the computer industry. The thirteen years experiences he had earlier in Eclipse Enterprises,
Spark Software and Cloud Development have proven to strengthen not only his skills and capacity, but also
strong networking. The company would also like to extend its highest appreciation to Ms. Rachel Brown, our
former Sales Manager, for the dedication and commitment she showed us until the day of her retirement.

HRD Manager

16. The announcement is mostly concerned with ….


A. Manager substitution
B. Bill‘s experience
C. Computer industry
D. Bill‘s achievement
E. The new Sales Manager

SMA Alfa Centauri Bandung Halaman 178


MODUL PERSIAPAN UN & SBMPTN BAHASA INGGRIS 2017/ 2018

17. What made Mr. William get the promotion? 18. It can be inferred from the text that ….
A. Ms. Brown‘s retirement. A. Bill William is a skillful IT engineer
B. Mr. William‘s experience. B. Bill William is a new Sales Manager
C. Ms. Brown‘s recommendation. C. Bill William is a skillful IT Manager
D. Mr. William‘s achievement. D. Bill William has 13 years of experience in
E. Mr. William‘s networking. computer industry
E. Bill William replaces Ms. Brown‘s position

The following text is for questions 19 and 20.

Dear Sir/Madam,

I am writing to complain about an article in last Monday‘s newspaper, ―Schools fail low-income children‖.
As the head teacher of one of the largest junior high schools in the city, I feel strongly that the article does not
describe our school.
My school has children from many different economic backgriunds, but a large percentage comes from
lower-income families. Nevertheless, our tests show that the children do well and benefit from the excellent work
our teachers do. Last year our school moved from 24th to 5th place according to the official assessment by the
state government. This is clearly not the ―failure‖ that the article suggests.
I would strongly recommend that more careful research should be conducted in the future so that your
newspaper will more realistically describe the true situation in our schools.

Yours sincerely,

Joan Gonzalez

19. The letter shows Ms. Gonzalez‘s strong E. low-income children are not as good as
disagreement with the newspaper‘s claim that high-income children
….
A. children from low-income families do not 20. What is expected by Joan Gonzalez from
perform well in the official assessment Monday‘s newspaper?
B. the poor students in her school fail in the A. To present information based on facts.
national examination B. To conduct more research.
C. her school only admit students from C. To fix the information given.
economically advantaged families D. To explain more about that school.
D. many of her students fail because of their E. To be responsible for the report.
parents‘ low income

The following text is for questions 21 to 23.


Please do the following steps:

1) Plug the electronic cord of the microwave


into a socket;
2) After that, put the food into the microwave;
3) Don‘t forget to press start button to heat
the food;
4) Wait for the beep sounds, it indicates that
the heating process is done;
5) Finally, take out the food and enjoy it;
6) Don‘t forget to wash hands after cleaning
the microwave.

SMA Alfa Centauri Bandung Halaman 179


MODUL PERSIAPAN UN & SBMPTN BAHASA INGGRIS 2017/ 2018

21. What is the text about? D. It alerts you that this is the time to enjoy
A. The steps to cook in a microwave. your food.
B. The steps to use a microwave. E. It alerts you that the heating process is
C. The steps to heat food in a microwave. complete.
D. The steps to operate a microwave.
E. The steps to defrost food in a microwave. 23. What will you do after putting the food into the
microwave?
22. What does the beep signal you? A. Wait for the beep sounds.
A. It alerts you that the food is ready. B. Connect to the electric cord into a socket.
B. It alerts you that the foods ready to C. Take out the food and enjoy it.
process. D. Press the power button.
C. It alerts you that the heating is on E. Press the timer button.
progress.

The following text is for questions 24 and 25.

Independence Day Celebrated Peacefully Nationwide.

1000 flags, one love. A young boy and girl banded together to secure a flagpole to the ground in Mt. Merapi
valley in Yogyakarta on Sunday. The pole was one of 1000 flags erected in the valley during a flag-raising
ceremony entitled, ―Seribu Bendera Satu Cinta‖ (A thousand flags, One Love).

In Medan, North Sumatra; the residents of Aurvillage held their flag-raising ceremony. The local governments
paid attention to Usmarlin, a participant who said that the condition of the river had become a matter of great
concern because of pollution from both factories and domestic waste.

Meanwhile, 2.343 prisoners in South Sulawesi received sentence reductions due to the Independence Day
celebration, 92 of whom were released from prison.

The Jakarta Post, August 18 2014

24. Who enjoyed freedom in this Independence B. Aurvillage residents conserve their river by
Day celebration? holding flag-raising ceremony.
A. A young boy and girl. C. The bad condition of the river had become
B. Aurvillage residents. Aurvillage residents concern.
C. The local governments. D. The flag-raising ceremony in Aurvillage is
D. The 2.343 prisoners. needed to conserve the river.
E. The 92 prisoners. E. The residents of Aurvillage received
attention from the local government when
25. What is the main idea of paragraph two? holding a flag-raising ceremony.
A. Aurvillage residents held their flag-raising
ceremony in North Sumatra.

SMA Alfa Centauri Bandung Halaman 180


MODUL PERSIAPAN UN & SBMPTN BAHASA INGGRIS 2017/ 2018

The following text is for questions 26 and 27.

Chinese Group Demands


That Japanese Emperor Return Ancient Artefact

By Ankit Panda
February 13, 2016

Adding to a growing list of diplomatic siputes between China and Japan, a Chinese group is demanding
that Japan‘s emperor return a 1.300-year-old artefact that was allegedly looted by Japanese Soldiers in the 1930s.
According to a Xinhua report, the artefact in question in the Honglujing Stele, originally from ―north eastern China‖.
The request was made in a letter addressed to Emperor Akihito of Japan by China Federation of Demanding
Compensation from Japan (CFDC) via Japan‘s domestic and international media outlets, suggesting that part of
the intent is to shame Japan for its actions during the first half of the 20th century.
From Xinhua‘s report, it is unclear the extent to which the Chinese government is involved in the request
for the return of the artefact. The China Federation of Demanding Compensation (CFDC) is a civic group,
independent of the government. According to Xinhua, ―This is the first time a Chinese civic group has asked the
Japanese imperial family for the return of a looted Chinese relic. ―The group‖ seeks compensation for personal,
material and spiritual damage caused by Japanese militarism during the country‘s aggression against China in the
20th century‖.

26. What can be inferred from the text?


A. China and Japan had a great relationship 27. ― …, a Chinese Group is demanding that
during the first half of the 20th century. Japan emperor return a 1.300-year-old
B. China Federation of Demanding artefact that was allegedly looted by Japanese
Compensation (CFDC) has asked Japan to soldiers in the 1930s.‖ (Paragraph 1). The
return the artefact without any condition. underlined word is closest in meaning to ….
C. Japan was ruled by China in the 20th A. stolen
century. B. damaged
D. CFDC wanted the Japan emperor to admit C. neglected
their aggression to China in the 20th D. borrowed
century. E. taken
E. CFDC may have sent a letter directly to the
Japan emperor without involving their
government.

The following text is for questions 28 to 30.

Webcams Make Alaska Bears Accessible

Marks Thiessen, Associated Press, Anchorage, Alaska Sci-Tech Tue, July 24 2012, 6:15

A new video initiative is bringing the famed brown bears of Alaska‘s Katmai National Park directly to
computers and smartphones.
Without having to go there, you‘ll be able to watch mature bears compete for salmon at Brook Falls and
other sites and cubs tumbling over each other as they play. Starting Tuesday, a live web stream
(http://is.gd/bfPAs8) will allow the public to log on and see the brown bears in their natural habitat.
―I think it‘s an unparalleled opportunity for people to get that front row seat of the lives of the bears at
Brooks Camp‖, said Roy Wood, chief of interpretation for Katmai National Park and Preserve.
The project is a partnership with explore.org, which set up four high-definition cameras in Katmai,
spokesman Jason Damata told the Associated Press. Three of them are at existing viewing stands where bear
fans come to watch the animals.
The cameras provide access to a national park that is difficult to reach and expensive for most tourists.
It is about 275 miles southwest of Anchorage, but no roads lead to Katmai. A trip there involves multiple

SMA Alfa Centauri Bandung Halaman 181


MODUL PERSIAPAN UN & SBMPTN BAHASA INGGRIS 2017/ 2018

airplanes and a lot of advanced planning: it‘s hard to get a lodge reservation at Brooks Camp before 2014.
Camping is allowed, but on a reservation system that goes online Jan 5.
―It takes a lot of time, a lot of effort and a lot of money, and the webcams will make it accessible to
anyone with access to a computer, a smartphone, a tablet device‖, Wood said.

28. What is the text about? C. we can use the new video to shoot bears
A. A video display of Katmai‘s bears for without going to their habitat
computers and smartphones. D. Katmai National Park has made a video of
B. The life of Alaskan bears in their natural their bears in nature environment
habitat. E. webcams enable us to see bears‘ life in
C. How to protect the endangered Alaskan nature on a computer or a smartphone
bears.
D. The bears of Katmai National Park in 30. ― … to watch mature bears compete for
Alaska. salmon at Brook Falls.‖ (Paragraph 2)
E. Webcams with the newest technology. The underlined word is the most closely
associated to ….
29. It‘s proudly stated in the text that …. A. fight
A. a new video can be used with a computer B. kill
B. the project is partnership with explore.org C. hug
and Katmai D. defeat
E. defend
The following text is for questions 31 to 33.

The name Studebaker is well known today due to five Studebaker brothers‘ action. They were
responsible for one of the oldest vehicle manufacturing companies in the United States.
The Studebaker brothers were born in the first half of the nineteenth century. In 1852, two of the
Studebakers brothers opened a wagon-building shop. Their entire resources were some tools for building
wagons and 68 dollars. They managed to build three wagons in their first year of operations and sold two of
three wagons. Their business continued to increase steadily. By the time of the Civil War in the 1860s, they had
a government contract to build wagons for the war.
After the war ended, the brothers added a new division which successfully produced well-known
carriages. At the end of the nineteenth century, the Studebaker company became the largest and best-known
manufacturer of horse-drawn wagons and carriages in the world.
In 1897, the company started experimenting with vehicles that run under their own power. The company
began making their first electric automobiles and later developed gasoline automobiles. By the 1920, the
company stopped making wagons and started producing cars. The Studebaker Company stayed in business
until 1966.

31. What did the Studebaker brothers have when D. The time the brothers started their
they started their first company? company.
A. Tools for repairing wagons. E. The time the brothers received the
B. A small amount of money. government contract.
C. A business contract with the government.
D. Money to build three wagons. 33. It can be inferred from the text that the
E. A government contract. Studebaker brothers ....
A. had three wagons in the first year of
32. What is the main idea of paragraph two? operations
A. The time the brothers were born. B. started producing wagons in 1852
B. The time the brothers developed their C. stopped producing automobiles in 1920
wagon shop. D. developed gasoline cars before electric
C. The time the brothers managed to sell their cars
wagon. E. were one of the pioneers of vehicle
manufacturing companies in U.S

SMA Alfa Centauri Bandung Halaman 182


MODUL PERSIAPAN UN & SBMPTN BAHASA INGGRIS 2017/ 2018

The following text is for questions 34 to 36.

SNAKES
Snakes are reptiles (cold-blooded creatures). They belong to the same group as lizards (the scaled
group, Squamata) but form a sub-group of their own (Serpentes).
Snakes have two legs but a long time ago they had claws to help them slither along. Snakes are not
slimy. They are covered in scales which are just bumps on the skin. Their skin hard and glossy to reduce friction
as the snake slithers along the ground.
Snakes often sun bathe on rocks in the warm weather. This is because snakes are cold-blooded, they
need the sun‘s warmth to heat their bodies up.
Most snakes live in the country. Some types of snakes live in trees, some live in water, but most live on
the ground in deserted rabbit burrows, in thick, long grass and in old logs.
A snake‘s diet usually consists of frogs, lizards, and mice and other snakes. The anaconda can eat
small crocodiles and even bears. Many snakes protect themselves with their fangs. Some snakes are protected
by scaring their enemies away like the cobra. The flying snakes glide away from danger. Their ribs spread apart
and the skin stretches out. Its technique is just like the sugar gliders.

34. Since the snakes are cold-blooded, they …. C. have two legs and claws
A. like sucking the cool blood D. use their legs to climb the tree
B. avoid sun-bathing to their skins E. use their claws to slither along the ground
C. never sun bathe in the warm weather
D. live on the ground in deserted burrows 36. How do flying snakes protect themselves?
E. require the sun‘s warmth to heat their A. They fly away.
bodies B. They use their fangs.
C. They scare their enemies.
35. We know from the text that snakes …. D. They stretch out their skin.
A. do not have claws E. They eat the other animals.
B. do not like sunlight

The following text is for questions 37 to 39.

Apartments Offer Practical Living

The number of high-rise apartment buildings continues to mushroom not only in the center of the city
but also in suburban areas. This shows that many Indonesians enjoy living in apartments. This is true for a
number of reasons.
In the first place, people prefer to live in an apartment because it is located near strategic areas. They
can save time and energy as they don‘t have to experience terrible traffic jam.
In addition, they enjoy the new life style because an apartment offers comfort and privacy so they don‘t
have to worry abou the city‘s high crime rate as most apartments have 24-hour security systems.
Moreover, people now enjoy practicality and this is reflected in the design of their place. Living rooms
become smaller and a veranda is no longer considered an important part of the house.
From the facts above, it is clear that apartments offer practical living which thus make them
mushrooming.

(source: Indonesian Property Business)

37. According to paragraph two, what does an apartment offer?


A. Comfort.
B. Security.
C. Practicality.
D. Privacy.
E. Safety.

SMA Alfa Centauri Bandung Halaman 183


MODUL PERSIAPAN UN & SBMPTN BAHASA INGGRIS 2017/ 2018

38. It can be inferred from the text that …. 39. ―The number of high-rise apartment buildings
A. many Indonesian people right now enjoy continues to mushroom not only in the center
living in a small house ….‖ (Paragraph 1)
B. practicality means security The underlined word is closest in meaning to
C. people prefer living in an apartment ….
because it is located near office areas A. increase
D. living in an apartment creates practical life B. enhance
style C. boost
E. secure life style is mostly preferred now D. intensity
E. raise

The following text is for questions 40 to 42.

Promoting Tourism: Good or Bad?

Lots of tourists visit our town, but many people argue whether this is good or bad for the community.
Each side has their own viewpoints.
Those in favor of the tourist industry argue that tourism brings money to the town and creates
employment. Business owners also support this saying that tourism promotes development in town. Moreover,
everyone benefits from having better facilities and more shops. Others also argue that tourism develops a better
understanding of other cultures so people become more tolerant.
People against tourism, however, say that most of tourist‘s dollars are spent in international hotels
owned by foreign interest groups. Consequently, the town receives little financial benefit. In addition, they point
out that the new developments have caused traffic and water supply problems. They argue that several
unpleasant incidents in the community indicate that the local community resentment towards tourists.
Undoubtedly, the local council should please both sides. Not only should it promote tourism, but also
restrict new development that would cause traffic and water supply problems.

40. From the viewpoint of those who disagree with C. Business, development, investment and
tourist industry, who will likely receive the most social.
financial benefit? D. Public facilities, development, finance,
A. Business owners. social.
B. The local people. E. Finance, infrastructure, cultural
C. The local community. understanding, community development.
D. The local government.
E. International hotel owners. 42. ―Not only should it promote tourism, but also
restrict new development ….‖ (Paragraph 4)
41. What are the factors that make the people The underlined word is closest in meaning to
agree to the tourist industry? ….
A. Business, finance, culture, and A. ban
infrastructure. B. open
B. Investment, community building, cultural C. curb
understanding. D. limit
E. prohibit

The following text is for questions 43 to 45.

The guitar is one of the oldest instruments. It probably originates the vicinity of China. There were
guitars in ancient Egypt and Greece as well, but the written history of the guitar started in Spain in the 13 th
century. By 1500, the guitar was popular in Italy, France, and Spain. A French document of that time concludes
that many people were playing the guitar. Stradivarius, the undeniable king of violin makers, could not resist
creating a variety of guitars.
Haydn, Schubert, and others wrote guitar music. When the great Beethoven was asked to compose
music for the guitar, he went into a rage and refused, but eventually even Beethoven could not ignore the

SMA Alfa Centauri Bandung Halaman 184


MODUL PERSIAPAN UN & SBMPTN BAHASA INGGRIS 2017/ 2018

challenge; a legend tells us he finally called the guitar a miniature orchestra! Perhaps that is why in rural areas
around the world the guitar has been a source for millions of music to enjoy.

43. The text mainly talks about …. 4) Jason Thomas Mraz is an American song
A. the oldest instrument writer who was born on June 23, 1977.
B. the popular instrument in Italy 5) In July 2014, Mraz sold over seven million
C. the guitar now and then albums and over million digital singles.
D. the history of the guitar 6) Mraz then nominated in two categories for
E. the origin of the guitar the Grammy Awards and won two awards.

44. Why did Beethoven call guitar a miniature The correct arrangement is ….
orchestra? A. 2 – 3 – 1 – 6 – 5 – 4
A. The guitar is the instrument of all time. B. 3 – 2 – 1 – 6 – 4 – 5
B. The guitar is the oldest instrument. C. 4 – 2 – 1 – 3 – 5 – 6
C. The guitar is the source of all music. D. 5 – 2 – 1 – 3 – 6 – 4
D. The guitar produces the elements of E. 6 – 2 – 1 – 3 – 4 – 5
orchestra sounds.
E. The sound of guitar is off-harmonious- 47. Rearrange the following jumbled sentences
colors sounds. into the correct and meaningful procedure.
How to operate a digital thermometer:
45. Which of the following information is 1) Wait for few seconds until you hear its
mentioned in the text? beep sounds.
A. The reasons why Beethoven was furious. 2) To make sure the thermometer is working,
B. The reasons why guitar becomes the switch it on first.
sources of enjoyment. 3) Set the temperature to zero as you can
C. The sounds of the guitar resemble see it from the indicator.
orchestra. 4) Place the tip of the thermometer under
D. The reasons why Stradivarius could not your tongue, keep it in that position by
resist creating guitars. closing your lips.
E. The sound of guitar resemble kecapi. 5) Take the thermometer out of your mouth
and see your temperature.
46. Rearrange the following jumbled sentences
into the correct and meaningful paragraph. The correct arrangement is ….
1) In 2002, he released his debut studio A. 2 – 3 – 4 – 1 – 5
album called ―Waiting for my Rocket to B. 2 – 3 – 4 – 5 – 1
come‖. C. 1 – 3 – 4 – 2 – 5
2) The first public performance was in San D. 2 – 1 – 4 – 3 – 5
Diego Coffee house in 2000. E. 3 – 1 – 4 – 2 – 5
3) In 2008, he released his third studio album
entitled ―We Sing, We Dance, We Steal
things‖.

SMA Alfa Centauri Bandung Halaman 185


MODUL PERSIAPAN UN & SBMPTN BAHASA INGGRIS 2017/ 2018

For questions 48 to 50, complete the following text with the words provided.

Nyi Roro Kidul, The Queen in Southern Sea

Pajajaran Kingdom lied in West Java from 1333 AD to 1630 AD. Pajajaran‘s greatest ruler was Prabu (King)
Siliwangi. He had a bride in his harems and a very beautiful daughter, Dewi Kadita and her mother made other
harems jealous, and they had a (48) … against them.
The harems used some kind of black magic to make the bodies of Dewi Kadita and her mother to be filthy
and they turn to be so ugly and disgusting. Prabu Siliwangi got angry against the mother and her daughter and
(49) … them to get out of the palace, as they were thought to be bad luck for the Kingdom.
After wandering around the country, she returned and turned into a supranatural form of life. Since then,
she ruled all creatures in the southern coast of Java Island, and she was known as Nyi Roro Kidul (Javanese of
―Lady of South Sea). To avenge her father, she became the primary bride for Mataram Kings, the (50) … of
Pajajaran Kingdom.

48. ….
A. group
B. partner
C. club
D. conspiracy
E. percussion

49. ….
A. powered
B. forced
C. captured
D. toughen
E. dominated

50. ….
A. adversary
B. contrary
C. friend
D. leader
E. rival

SMA Alfa Centauri Bandung Halaman 186


MODUL PERSIAPAN UN & SBMPTN BAHASA INGGRIS 2017/ 2018

UJIAN NASIONAL SMA/MA


TAHUN 2017
Listening Section
In this section of the test, you will have the chance to show how well you understand spoken English. There are
four parts to this section with special directions for each part.

PART I
Questions 1 to 4.

Directions:
In this part of the test, you will hear some dialogues and questions spoken in English. The dialogues and the
questions will be spoken twice. They will not be printed in your test book, so you must listen carefully to
understand what the speakers are saying.
After you listen to the dialogue and the questions about it, read the five possible answers, and decide which one
would be the best answer to the question you have just heard. Now listen to a sample question.

You will hear :


Woman : Didn‘t Kathy go travelling to Japan with you last month?
Man : Are you kidding? Even if it didn‘t cost anything, she‘d rather stay at home.
Narrator : What does the man think about Kathy?
A. Kathy likes travelling.
B. Kathy went to Japan alone.
C. Kathy isa thrifty woman.
D. Kathy is a stay at home woman.
E. Kathy is an extravagant woman.
Narrator : The best answer to the question ―What does the man think about Kathy?‖ is ―Kathy is a stay
at home woman‖. Therefore you should answer (D).

Now let‘s begin with number 1.


1. ....
A. Regret. 3. ....
B. Congratulations. A. Something wrong
C. Sympathy. B. A terrible cough
D. Expectation. C. A problem to sleep
E. Hope. D. How to stop making
E. The effect of smoking
2. ....
A. Make themselves crazy 4. ....
B. Come late to school A. She has met her friend
C. Call the teacher B. She has been on the news
D. Hope and pray C. She has been accepted at medical faculty
E. Miss the first lesson D. She has become a doctor
E. She deserves to study hard

SMA Alfa Centauri Bandung Halaman 187


MODUL PERSIAPAN UN & SBMPTN BAHASA INGGRIS 2017/ 2018

PART II
Questions 5 to 7.

Directions:
In this part of the test, you will hear some incomplete dialogues spoken in English, followed by four responses,
also spoken in English. The dialogues and the responses will be spoken twice. They will not be printed in your
test book, so you must listen carefully to understand what the speakers are saying. You have to choose the best
response to each question.

Now listen to a sample question.


Woman : Hi, Tom.
Man : Hi, Nancy. What are you doing here?
Are you waiting for someone?
Woman : No, I‘m waiting for a taxi. I want to go to a book store.
Man : ....
Narrator : What is the best response to the woman‘s statement?
A. Let me take you to the book store.
B. I‘ll find a good book for you.
C. The store is open till 7 p.m.
D. Oh, the book store is somewhere else.
Narrator : The best answer to the question ―What is the best response to the woman‘s statement‖ is ―Let me take
you to the book store‖. Therefore you should choose answer (A).

5. Mark your answer on your answer sheet.


6. Mark your answer on your answer sheet.
7. Mark your answer on your answer sheet.

PART III
Questions 8 to 11.

Directions:
In this part of the test, you will hear some monologues spoken in English. The monologues will be spoken twice.
They will not be printed in your test book, so you must listen carefully to what the speakers are saying. After you
listen to the monologues, look at the five pictures provided in your test book, and decide which would be the
most suitable one for the monologue you have just heard.

8. .... C.
A.

D.
B.

SMA Alfa Centauri Bandung Halaman 188


MODUL PERSIAPAN UN & SBMPTN BAHASA INGGRIS 2017/ 2018

E. C.

D.

9. ....
A.

E.

B.

C. 11. ....
A.

D.
B.

E.

C.

10. ....
A.

D.

B.

SMA Alfa Centauri Bandung Halaman 189


MODUL PERSIAPAN UN & SBMPTN BAHASA INGGRIS 2017/ 2018

E.

PART IV
Questions 12 to 15.

Directions:
In this part of the test, you will hear several monologues. Each monologue will be spoken twice. They will not be
printed in your test book, so you must listen carefully to understand what the speaker is saying. After you hear
the monologu and the question about it, read the five possible answers and decide which one would be the best
answer to the question you have just heard.

12. ....
A. West Nusa Tenggara (NTB)
B. Hosting a festival in NTB
C. Coral reefs
D. The conservation destinatuons
E. The environmental issues

13. ....
A. More hotels and restaurants will be free
B. More festibals will be conducted by the tourists
C. More income will be generated for the government and local peopla
D. More islands will be explores for hotels and restaurants
E. More trees will be removed from the islands

14. ....
A. A biography of General Soedirman
B. The family of General Soedirman
C. The death of General Soedirman
D. A spirit of General Soedirman for the Indonesian Armed Forces
E. The military forces commanded by General Soedirman

15. ....
A. His uncle‘s name was also Soedirman
B. January is the month of Maulud
C. Soedirman was shot and died in the military war
D. Soedirman died when he was relatively young
E. Soedirman died on 1 March 1949

THIS IS THE END OF THE LISTENING COMPREHENSION SECTION

SMA Alfa Centauri Bandung Halaman 190


MODUL PERSIAPAN UN & SBMPTN BAHASA INGGRIS 2017/ 2018

This text is for questions 16 and 17.

Luis Lionel Andress Messi, born June 24, 1987, is an Argentina football player for F.C Barcelona. He is not very
tall, mainly, due to the growing problem he had when he was younger. His eyes are brown. He never has short hair.
Lionel Messi started playing footbal at a very early age in his hometown‘s Newell‘s Old Boys. From the age of 11,
he suffered from a hormone deficiency and as Lionel‘s parents were unable to pay for the treatment in Argentina, they
decided to move to Barcelona, Spain.
In the 2003 – 2004 season, when he was still on 16 years old, Messi made his first team debut in a friendly with
Porto that marked the opening of the new Dragao stadium. The following championship winning season, Messi made his
first appearance in an official match on October 16, 2004, in Barcelona‘s derby win against at the Olympic Stadium (0-1).
And now, in 2010, 2011, and 2012 he is the best player in the world with collect 3 Balloon D‘Or.

16. According to the text, Messi‘s parents moved 17. What is the main idea of the third paragraph?
to Barcelona .... A. He hasn‘t really attractive face, but he‘s a
A. Because they were very poor in Argentina very good football player.
B. Because they wanted Messi to be B. Lionel Messi is a good player for FC.
successful in soccer Barcelona.
C. So that Messi could learn in the best C. His best characteristics are on the foot has
soccer club competitiveness.
D. To get Messi‘s health problem cured D. He appears to be a quite good and modest
E. To find the best treatment to cure Messi‘s person.
health problem E. He has long black hair and brown eyes.

This text is for questions 18 and 19.

We are announcing today that we are bringing the Milestone and Ever Green brands even closer together.
Effective as of 5th December 2013, our official name will be:
GREEN MILES WEST
The substitution of "West" in our name---replacing "Cianjur"---is the result of an agreement we reached with
Cianjur Gardening Association, following a protest over the original use of "Cianjur" in our name.

We hope this does not create any confusion among our loyal consumers. While this represents a change from
our initial name introduction, it does not change the quality of products we offer to our consumers.

18. Who issues the announcement? 19. ―The substitution of ―West‖ in our name ....‖
A. Green Miles West Company. The word ―substitution‖ has closest in meaning
B. Green Miles West Customers. to ....
C. The co-worker of Green Miles West. A. merger
D. The partnership of Green Miles West. B. insertion
E. The loyal customers of Green Miles West. C. development
D. replacement
E. improvement

SMA Alfa Centauri Bandung Halaman 191


MODUL PERSIAPAN UN & SBMPTN BAHASA INGGRIS 2017/ 2018

This text is for questions 20 to 22.

Jl. Jambu 129


Bandung
30th January 2017

Dear Fred,
It was a real sorrow that I heard this morning of your great loss. I knew your mother was ill. For your brother told
me several weeks ago. However, as he at that time did not seem to think. The illness was very serious, the news
of your mother‘s death came to me as a shock. You have my sincere and heartfelt sympathy, my dear fellow, in
your sorrow. I know you will feel it deeply, for you always thought so much of your mother and loved her so truly.
I feel it also as a personal loss to myself for your mother was always very kind to me. I admired her a good and
noble woman. Her death must be a terrible gried to your father too. Please assure him of my sincere sympathy.
Words, I konw, are poor comforters. ―The heart knows its own sorrow‖, and in such sorrows we are always alone.
However, it is not mere words when I say that I feel with you in your sorrow.

Your sincere friend,

Jack

20. What is the possible relation between the D. Jack had known Fred‘s mother before
sender and the recipient? E. Fred is the only child in his family.
A. Relatives.
B. Employers. 22. ―You have my sincere and heartfelt sympathy,
C. Friends. my dear fellow, in your sorrow‖. What is the
D. Siblings. closest meaning of the underlined word?
E. Employees. A. Real.
B. Caring.
21. From the letter we know that .... C. Generous.
A. Fred‘s mother was not ill before her death D. Honest.
B. Jack sent letter to Fred several weeks ago E. Curious.
C. Jack was very sorrowful to send the letter
to his mother

The following text is for questions 23 to 26.

If someone is having a nose bleed, your priority is to control the bleeding and keep their airway open.
Get them to sit down (not lie down) as keeping the nose above the heart will reduce bleeding.
Get them to lean forward (not backwards), to make sure the blood drains out through their pause every ten
minutes, until the bleeding stops.
Encourage them not to speak, swallow, cough, spit or sniff because this may break blood clots that may have
started to form in the nose.
If the bleeding is severe, or if it lasts more than 30 minutes, call for medical help.

23. What is the purpose of the text? B. To report the procedural steps in helping
A. To inform the readers the methods to help nose bleeding patients.
nose bleeding patients. C. To illustrate the preventive methods of
nose bleeding.

SMA Alfa Centauri Bandung Halaman 192


MODUL PERSIAPAN UN & SBMPTN BAHASA INGGRIS 2017/ 2018

D. To provide instant help for nose bleeding B. he/she will suffer from severe bleeding
patients. C. his/her throat will block the airway
E. To describe the control methods of nose D. his/her blood clot will not form
bleeding. E. his/her nose bleeds again

24. The text is mostly useful in a condition .... 26. ― ... to make sure the blood drains out ...‖
A. when the bleeding cannot be controlled (paragraph 3)
B. when the patient‘s airway is blocked The underliend word is closest in meaning to
C. when the nose bleed occurs regularly ....
D. when the nose bleed just accured A. travel
E. when the patients keep coughing B. portray
C. remark
25. In case of nose bleeding, if the person speaks, D. steam
.... E. proceed
A. his/her blood drains out though his/her
nose

The following text is for questions 27 to 30.

A kangaroo is an animal found only in Australia. It has a smalll relative called a wallaby, that lives in
Tasmania and New Guinea. Kangaroos eat grass and plants. They have short front legs, long and strong back
legs and a tail. These are used to sit up and jump. Kangaroos are well known for their 8-meter forward jumps and
more than 3-meters high leap across fences. They can also run at the speed of over 45 kilometers per hour. The
largest kangaroos are the Great Grey Kangaroos and Red Kangaroos. Adult kangaroos grow to a meter in length
and 90 kilos in weight. Kangaroos are marsupials. This means that the female kangaroo has an external pouch
on the front of her body. The baby kangaroo is very tiny when it was born. It right away crawls into its mother‘s
pouch where it spends the first five months of its life.
Adapted from Peter Haddock Ltd, Ref.083

27. What is the purpose of the text?


A. To inform Kangaroo‘s uniqueness.
B. To report the features of Kangaroo.
C. To encourage Kangaroo‘s preservation.
D. To describe how a Kangaroo looks like.
E. To raise awareness of Kangaroo‘s special status.

28. We can conclude from the text that ....


A. the largest kangaroos are the males
B. the male kangaroos do not have pouch
C. kangaroos only eat grass and plants that grow in Australia
D. Tasmania‘s and New Guinea‘s wallabies are different in size
E. the new born kangaroo is also skillful in jumping and leaping

29. Which of these statements is contrary to the fact?


A. Kangaroos are not carnivorous.
B. Kangaroo is native animal of Australia.
C. All wallabies are of smaller size than Kangaroos.
D. All marsupial females have pouch on the front of their body.

SMA Alfa Centauri Bandung Halaman 193


MODUL PERSIAPAN UN & SBMPTN BAHASA INGGRIS 2017/ 2018

E. Baby Kangaroos leave their mother‘s pouch at the age of five months.

30. Look at the underlined word ‗pouch‘. Which of the following words is closest in meaning to it?
A. Saving.
B. Purse.
C. Compartment.
D. Pocket.
E. Holding.

The following text is for questions 31 to 34.

The Dangers of Typing SMS while Driving

The popularity of mobile devices has had some dangerous consequences. We know that mobile
communications are linked to a significant increase in distracted driving which results in injury and loss of life.
The National Highway Traffic Safety Administration reported that in 2010 driver distraction was the
cause of 18 percent of all fatal crashes – with 3,092 people killed – and crashes resulting in an injury – with
416,000 people wounded.
The Virginia Tech Transportation Institute found that text messaging creates a crash risk 23 times worse
than driving while not distracted.
Eleven percent of drivers aged 18 to 20 who were involved in an automobile accident and survives admitted
they were sending or receiving texts when they crashed. Distracted driving endagers life and property and the
current levels of injury and loss are unacceptable.

31. What is the main idea of the passage? 33. From the text, we know that ....
A. The warning of texting and driving. A. the mobile phone should be banned in the
B. The debatable issue of texting and driving. street
C. The involvement of mobile devices while B. distracted driving makes accident more
driving. rarely to happpen
D. The risks of texting while driving. C. most of the accident caused by the
E. The consequences of not paying attention condition of the road
to traffic. D. 18 percent of fatal crashes were caused by
unnoticed street signs
32. What does the passage tell us about the E. drivers involved in car accidents admitted
writer‘s opinion on the issue at hand? they were texting when they crashed
A. Text messaging creates more risk than
undistracted driving. 34. Distracted driving endangers life ... it is not an
B. Mobile communication doesn‘t have acceptable action.
relation with accident. A. as
C. Many people lost their lives becaus of B. if
injury. C. so
D. Distracted driving is still safe for the D. and
drivers. E. yet
E. Only adult drivers involved in the accident.

SMA Alfa Centauri Bandung Halaman 194


MODUL PERSIAPAN UN & SBMPTN BAHASA INGGRIS 2017/ 2018

This text is for questions 35 to 38.

Galileo Galilei (1564 – 1642) – Astronomer and Scientist. Galileo developed a superior telescope and
made many significant discoveries in astronomy. He was sentenced to life imprisonment by the inquisition for his
support for the Copernican theory that the sun was at the center of the solar system.
Galileo was born in Florence, Italy in 1564 to a poor but noble family. His parents recognised their
child[s innate intelligence and talents and so made sacrifices to have him educated. At his father‘s insistence,
Galileo studied the profitable career of medicine. But, at the University of Pisa, Galileo became fascinated in a
wide range of subjects. He was also critical of many of Aristotle‘s teaching which had dominated education for
the past 2,000 years.
Galileo was appointed to be a mathematics professor at the University of Pisa, but his strident criticisms of
Aristotle left him isolated among his comtemporaries. After three years of persecution, he resigned and went to
the University of Padua. Where he taught maths. His entertaining lectures attracted a large following and he was
able to spend the next 18 years pursuing his interests in astronomy and mechanics.

35. Something that we can learn from Galileo 37. According to the biography, why did Galileo
Galilei‘s biography is .... Galilei resign from his teaching job at the
A. we must always believe whatever the University of Pisa?
society believes to avoid getting A. He had attracted many followers.
persecuted B. He was persecuted for three years.
B. we must never go against the believe of C. He was appointed as a mathematics
the society to avoid getting imprisoned professor.
C. we should believe in something and stay D. He had a better offer from the University of
faithful to it no matter how hard Padua.
D. we must sacrifice everything to get E. He wanted to pursue his interests in
ourselves educated astronomy and mechanics.
E. we should spend 18 years to pursue our
interests 38. ―His parents recognised their child‘s innate
intelligence and talents and so made sacrifices
36. From the fact that Galileo Galilei was to have him educated‖. (Paragraph 2)
imprisoned for supporting the Copernican The underlined word is closest in meaning to
theory, which later was proven to be true, we ....
know that Galileo Galilei was .... A. acquired
A. reckless B. ingenious
B. stubborn C. multiple
C. tenacious D. inborn
D. arrogant E. strident
E. foolish

SMA Alfa Centauri Bandung Halaman 195


MODUL PERSIAPAN UN & SBMPTN BAHASA INGGRIS 2017/ 2018

This text is for questions 39 to 42.

A tsunami is a series of ocean waves that sends surges of water, sometimes reaching heights of
over 100 feet (30.5 meters), onto land. These walls of water can cause widespread destruction when they
crash ashore.
These awe-inspiring waves are typically caused by large, undersea earthquakes at tectonic plate
boundaries. When the ocean floor at a plate boundary rises or falls suddenly, it displaces the water above
it and launches the rolling waves that will become a tsunami.
Most tsunamis–about 80 percent–happen within the Pacific Ocean‘s ―Ring of Fire,‖ a geologically
active area where tectonic shifts make volcanoes and earthquakes common.
Tsunamis may also be caused by underwater landslides or volcanic eruptions. They may even
be launched, as they frequently were in Earth‘s ancient past, by the impact of a large meteorite plunging
into an ocean.
Tsunamis race across the sea at up to 500 miles (805 kilometers) an hour—about as fast as a jet
airplane. At that pace, they can cross the entire expanse of the Pacific Ocean in less than a day. And
their long wavelengths mean they lose very little energy along the way.
In deep ocean, tsunami waves may appear only a foot or so high. But as they approach shoreline and
enter shallower water they slow down and begin to grow in energy and height. The tops of the waves
move faster than their bottoms do, which causes them to rise precipitously.
A tsunami‘s trough, the low point beneath the wave‘s crest, often reaches shore first. When it
does, it produces a vacuum effect that sucks coastal water seaward and exposes harbor and sea floors.
This retreating of sea water is an important warning sign of a tsunami, because the wave‘s crest and its
enormous volume of water typically hit shore five minutes or so later. Recognizing this phenomenon can
save lives.
A tsunami is usually composed of a series of waves, called a wave train, so its destructive force
may be compounded as successive waves reach shore. People experiencing a tsunami should remember
that the danger may not have passed with the first wave and should await official word that it is safe to
return to vulnerable locations.
Some tsunamis do not appear on shore as massive breaking waves but instead resemble a
quickly surging tide that inundates coastal areas.

41. From the text, we know that Tsunami can be


39. Why did the author write the report? very destructive because ....
A. to raise people‘s awareness about A. they come after earthquakes
tsunami. B. they are caused by volcanic eruptions
B. to warn people about an upcoming C. they are tall, fast, forceful and repetitive
tsunami. D. they have a vacuum effect
C. to inform people about past tsunamis. E. they occur suddenly
D. to inform people about the different types 42. ―Some tsunami do not appear on shore as
of tsunami. massive breaking waves but instead resemble
E. to warn people about the dangers of a quickly surging tide that inundates coastal
tsunami. areas‖. (Paragraph)
The underlined word is closest in meaning to
40. Tsunamis are usually the result of .... ....
A. the sudden rise or fall of ocean floors A. floods
B. Pacific Oceans ―Ring of ocean floors‖ B. covers
C. the awe-inspiring waves C. fills
D. volcanoes D. attacks
E. landslides E. submerges

SMA Alfa Centauri Bandung Halaman 196


MODUL PERSIAPAN UN & SBMPTN BAHASA INGGRIS 2017/ 2018

This text is for questions 43 to 46.

Pakistan’s houbara bird facing extinction

The indiscriminate hunting of endagered houbara bustard birds in Pakistan by ricj hunting parties from
the Gulf nations have pushed the species toward extinction. The hunting was initially banned by the Pakistan
courts, but under continuous pressure from the provincial governments of Sind, Baluchistan, and Punjab, as well
as the federal government, recently the courts lifted the ban on the hunting of this vulnerable species.
Some people in Pakistan have been advocating for sustainable hunting of the species instead of a
complete ban, but this is highly debatable. Reckless hunting for decades has pushed this beautiful species
toward extinction and it is now critically endangered in Pakistan.
No reliable or credible statistics on the pooulation dynamics of the soecies are available either. Unless
specially protected by law, the species runs the risk of becoming extinct in the wild in the next few decades.
Under these circumstances, the only logical approach to save the species from extinction in the not-so-
distant future is to immediately start comprehensive captive breeding program for the species across the nation.
If the captive breeding program runs successfully and produces an appreciable number of chicks, then
reintroduction programs could be initiated in their wild habitats.
Only after rebuilding the population for some time in the wild and allowing them to breed naturally to
reach a sustainable population could some restricted hunting be planned. Currently, the low numbers have made
this species vulnerable and with zero or poor monitoring, sustainable hunting practices in Pakistan for this
dwindlung species will not work.
Game hunting works only when a stable population is achieved with stricts monitoring on the ratio of
males to females hunted per season. If the females (no true statiscics available) are being taken out of the
population through random hunting, the population is sure to pass through a bottleneck in the wild habitat, with
high vulnerability to eventual extinction.
The people of Pakistan need to decide what is more important-a few petrodollars or protecting the
biodiversity of the nation.

43. What is the subject of the text? 45. According to the text, why does the writer want
A. The extinction of Pakistan‘s houbara bird. all kids of houbara bird hunting be banned?
B. The hunting of Pakistan‘s houbara bird. A. The population of houbara bird is getting
C. The conservation of Pakistan‘s houbara smaller and smaller.
bird. B. The captivation of houbara bird has been
D. The rehabilitation of Pakistan‘s unsuccessful.
biodiversity. C. The houbara bird is difficult to catch.
E. The protection of Pakistan‘s biodiversity. D. The houbara bird is beautiful.
E. The hunters are reckless.
44. From the text, we know that the writer ....
A. enjoys hunting the houbara bird 46. From the passage, we can conclude that
B. advocates for sustainable houbara bird people can resume hunting the houbara bird
hunting only ... a sustainable population is reached.
C. approves all kinds of houbara bird hunting A. after
D. tries to persuade people to conserve B. when
houbara bird C. because
E. breeds houbara bird in captivation D. if
E. before

SMA Alfa Centauri Bandung Halaman 197


MODUL PERSIAPAN UN & SBMPTN BAHASA INGGRIS 2017/ 2018

This text is for questions 47 to 50.

Nowadays, the police have been applying the new regulation concerning the use of seat belts. In
European countries, this regulation has been applied for a long time. However, this new regulation has become
controversial and is an interesting topic to discuss. Here are some of the arguments.
The use of seat belts has been proven to reduce the risk of injury or death in an accident. Seat belat
has become a standard component in cars. The research shows that most car accident will cause an injury to the
head. Frequently, drivers or the passengers driving without seat belts die because of this. By wearing the seat
belts, the injury will not happen since belts restrain our body on the car seat when the accident happens.
Unfortunately, many cars, especially the old ones, don‘t have seat belts. This is because the traffic
conditions in the past were unlike the recent traffic conditions. The designer of old cars didn‘t consider a seat belt
as an important part. Besides, the drivers wearing the seat belt will think that they are completely safe, so they
may drive carelessly. They are safe, indeed, but how about safety of other?
The seat belt is only one of the ways to reduce the risk of car accidents. It doesn‘t mean that we are
completely safe. In short, our safety depends on ourselves.

47. What is the main idea of paragraph 3?


A. Old cars do not use seat belts.
B. The old cars have seat belts for their safety.
C. The drivers wear seat belts may drive carefully.
D. The traffic conditions in the past were similar to the recent.
E. The old cars‘ designer considered a seat belt as an important part.

48. What is the function of seat belts?


A. To reduce the traffic conditions.
B. To become a standard component in cars.
C. To reduce the risk of injury or death in an accident.
D. To prevent the drivers driving cars carefully.
E. To increase the risk of injury in car accidents.

49. Who considered a seat belt as an unimportant part?


A. Police.
B. People.
C. Drivers.
D. Passengers.
E. Old cars designer.

50. ― ... that they ara completely safe‖. (Paragraph 3)


What does the word ‗they‘ refer to ?
A. Cars.
B. Seat belts.
C. Drivers.
D. Designers.
E. Others.

SMA Alfa Centauri Bandung Halaman 198


MODUL PERSIAPAN UN & SBMPTN BAHASA INGGRIS 2017/ 2018

SET 1

Choose the correct answer.


Questions 1 to 4 are based on the following text.

Two new species of dinosaurs, one a quick – moving meat-eater and the other and the other a giant-plant
eater, have been discovered in Antartica. The 70 million year-old fossil of the carnivore would have rested for milleniums
at the bottom of the Antartica Sea, while the remains of the 30 meter long eater were found on the top of a mountain.
The little carnivore about 1.8 meters tall was found on James Ross Island, of the coast of the Antartic
Peninsula.
Not yet named, the animal probably floated out to the sea after it died and settled to the bottom of what was
then a shallow area of the Weddell Sea. Its bones and teeth suggest it may represent a population of two-legged
carnivores that survived in the Antartic long after other predators took over elsewhere on the globe. ―For whatever
reason, they were still hanging out on the Antartic continent‖, Case said in a statement.
A second team led by William Hammer of Augustuna College in Rock Island, Illinois, found the 200 million year
old plant eater‘s fossils on a mountain top 13,000 feet (3,900 meters) high near the Beardmore Glacier now known as
Mt. Kirk Patrick, the area was once a soft riverbed.

1. The article informs us about ....


A. two species of dinosaurs 3. The following information is about the giant plant
B. the two kinds of new dinosaurs eater dinosaur, except ....
C. the newest discovery in Antartica A. it is 1.8 meters tall
D. the discovery of two species of dinosaurs in B. it is 30 meters long
Antartica C. it is 200 million years old
E. a team of researchers was funded by the D. it is found by William Hammer
National Science Foundation E. it is found on the top of a mountain

2. The main idea of the first paragraph is .... 4. ―Its bones and teeth suggest it may represent
A. the two species of dinosaurs are carnivores population of two legged carnivores that survived
and herbivores in the Antartic ....‖ (Paragraph 3). The underlined
B. the two species of dinosaurs have been word means ....
discovered in Antartica A. show
C. the two species were funded by the National B. inform
Science Foundation C. portray
D. the two species are a quick moving meat-eater D. explain
and a giant plant-eater E. symbolize
E. the two species of dinosaurs were floating at
the bottom of the Weddell Sea

Questions 5 to 8 are based on the following text.

The world‘s economic and political landscape (5) ... radical transportation since the movement was born at a
summit meeting on September 1st, 1960 in Belgrade, the capital of what was then Yugoslavia.
In the cold war climate of those days, leaders of the 25 developing countries all African and Asian with the
exception of Yugoslavia and Cyprus agreed (6) ... a strategy of political (7) ... or non alignment with regard to the two
major political blocks (8) ... by the Soviet Union (now Russia) and the United State.

SMA Alfa Centauri Bandung Halaman 199


MODUL PERSIAPAN UN & SBMPTN BAHASA INGGRIS 2017/ 2018

5. ....
A. undergo 7. ....
B. underwent A. neutral
C. had undergone B. neutral
D. has undergone C. neutrality
E. have undergone D. neutralize
E. neutralization

6. .... 8. ....
A. adopts A. led
B. adopted B. staved
C. to adopt C. steered
D. adopting D. injected
E. to be adopted E. submitted

Questions 9 to 10 are based on the following text.

Often people who hold higher positions in a given group overestimate their performance, while people in the
lower levels of the group underestimate theirs. While this may not always be true, it does indicate that often the actual
position in the group has much to do with the feeling of personal confidence a person may have. Thus, members who
hold high positions in a group or feel that they have an important part to play in the group will probably have more
confidence in their own perfomance.

9. The topic of the text is .... 10. From the text we may conclude that ....
A. different positions in a group A. if we let people know that they are an
B. the role of leaders in a group important part of the group, they will become
C. the attitudes of group members more self-confident
D. self-confidence of group members B. if people have high positions, they are
E. the relation between high position and self- confident that they perform better than others
confidence C. people who hold low positions in a group often
overestimate their performance
D. if people are given high positions, they will
underestimate people of low positions
E. people will get a higher position if they have
great self confidence

Questions 11 to 15 are based on the following text.

Belize, capital of British Honduras, has always been the colony‘s administrative, cultural, and geographic
center. It is a unique waterfront community characterized by large frame house with rambling, screened verandas.
Located on the periphery of teh hurricane zone, and at an average elevation of two feet above sea level, the town is
vulnerable to any feet tidal wave. Hence most buildings are on stilts, and many others have Spartan furnishings at the
street level.
The cooling effect of sea breezes in a community surrounded on three sides by salt water relieves the
otherwise oppressive climate. Located on the Carribean coast of Central America slightly more than eight hundred miles
south of New Orleans and about the same distance west Jamaica. Belize had a population of nearly six thousand in
1859, ten thousand in 1900, and reached thirty thousand in the 1960s.

SMA Alfa Centauri Bandung Halaman 200


MODUL PERSIAPAN UN & SBMPTN BAHASA INGGRIS 2017/ 2018

11. The following statements are true about Belize, Questions 16 to 20 are not based on the text.
except ....
A. cultural 16. ―Whom did you invite to this meeting?‖
B. geographic center ―The managers ... reports you read this morning‖.
C. capital of British Honduras A. whom
D. the colony‘s administrative B. which
E. it is not located on the periphery of the C. whose
hurricane zone D. of whom
E. of which
12. Because of the dangers from storms and waves,
the houses of Belize .... 17. ―I visited the University of California last year‖.
A. are dirty ―That is the place ... my brother is studying now‖.
B. are built on stilts A. when
C. are on the waterfront B. whom
D. enjoy cooling sea breezes C. which
E. have large, screen verandas D. where
E. whose
13. The word ‗vulnerable‘ (paragraph 1) means most
nearly .... 18. My girl loves bargain hunting as if she could earn
A. immune to her own money. This means that my girl ....
B. attracted to A. is very thrifty
C. protected against B. has already got a job
D. provide something C. does not like shopping
E. defenseless against D. has been very generous
E. does not make her own money
14. Between 1900 and 1960, the population of Belize
.... 19. He orders people around us as though he were
A. tripled the owner of the restaurant. From the above
B. destroyed statement we may conclude that ....
C. almost doubled A. he is just an ordinary employee
D. stayed about the same B. he is a successful businessman
E. increased by an unspecified amount C. he has been the owner for years
D. he is very proud of his restaurant
15. Conjunction ‗hence‘ (paragraph 1) could be E. he has worked hard to own the restaurant
replaced by ....
A. but 20. ―Why weren‘t you at Reza‘s birthday party last
B. however night?‖
C. moreover ―Oh, I ... overtime‖.
D. therefore A. must work
E. nevertheless B. had to work
C. should work
D. must have worked
E. ought to have worked

SMA Alfa Centauri Bandung Halaman 201


MODUL PERSIAPAN UN & SBMPTN BAHASA INGGRIS 2017/ 2018

SET 2

Choose the correct answer.


Questions 1 to 5 are based on the following text.

Everyone likes to group things. Language students group words as verbs, nouns, and so on; collections of
words are classified as phrases, or clauses, or sentences, and these again are reclassified according to their function. In
the same way, botanists classify plants as algae, or fungi, or gymnosperms, etc. Zoologist classify animals as
vertebrates and invertebrates. The vertebrates can be further classified as mammals, reptiles, birds, fish, etc.
Classification enables us to keep hold of more information and, if it is based on the right data, enables us to understand
better the ideas we are studying.
Chemists are no exception. The chemical classification of materials, it it is based on a good system, should
enable us to understand better the many substances which exist in our world. What is to be the basis of our
classification? Perhaps the most obvious on its appearance. Materials could be classified as solid, liquid or gas with
some mixed types as, for example, mud being solid/ liquid material and steam a liquid/gas material. Appearance could
enable us to subdivide our main classification groups a little further; the solid may be green, or black, powdery or
crystalline; the liquid may be colored, oily, thick, or free flowing; the gas may be colored. However, we soon realize that
many probably quite different materials have the same appearance. Both air and the deadly carbon-monoxide gas, are
colorless, odorless gasses, but we would not like to group them as the same thing. Many different liquids are colorless,
water-like materials.

1. The examples provided in paragraph 2 clarify that 3. The sentence ―Chemists are no exception‖
.... (paragraph 2) could possibly be restated as ....
A. many kinds of liquid should be grouped as one A. chemical materials can also be put into
B. different kinds of gas can be colorless and classification
odorless B. classification of chemical materials is without
C. materials in chemistry should be classified exception
differently C. chemist may also classify materials using
D. chemistry materials have more complicated certain criteria
classification D. when appearance is the basis, chemists are
E. taxonomy can be made and applied further to not involved
other areas E. in material classification, chemicals should not
be included
2. Paragraph 2 exemplifies the idea about
classification that ....
A. chemicals may be solid, liquid and gaseous 4. The paragraph following the passage most likely
B. appearance is not a useful basis in chemistry deals with the classification of ....
C. the use of colors is better than that of A. human sounds
appearance B. flora and fauna
D. both colors and appearance should be C. liquid and gases
considered D. human behaviors
E. colors should be included for identifying E. words and phrases
appearance

SMA Alfa Centauri Bandung Halaman 202


MODUL PERSIAPAN UN & SBMPTN BAHASA INGGRIS 2017/ 2018

5. How does the author organize the ideas?


A. Putting the main idea with examples.
B. Presenting causes followed by effects.
C. Interpreting different ways of classifying.
D. Presenting the strength of the main idea.
E. Expossing supporting details choronologically.

Questions 6 to 10 are based on the following text.

Although photography was first made public in 1839, the theory behind the principles of the medium begins
with Aristotle‘s description of how light waves behave when projected through a small aperture.This is fundamentally the
description of how a lens or camera‘s apperture operates when it projects an image onto the film at the back of a
camera. In the Middle Ages, Alhazen and Francis Bacon extended the principle to include a large, darkened room with a
small opening in one wall. In the 15th to 18th centuries this camera obscura, as it came to be called was reduced in size
and made convenient for artist to use in tracing scenic design and architectural perspective.
The chemical principles basic to photography were also described well before photography was ‗invented‘.
Johann Schulze, in 1727, demonstrated that silver salts turned dark when exposed to light. Carl W Scheele, in 1777,
showed that ammonia retarded the effects of light, and he indicated a possible way of stabilizing the photochemical
process. By the end of the 18th century, the necessary equipment (the camera obscura) were available at least to
produce semi permanent photographic images.
The artistic style and aesthetic of Renaissance and post-Renaissance Europe placed a high value on a
naturalistic rendering of nature and thus legitimatized the use of machines like the camera obscura by artist. By the mid-
18th century, a public demand had made itself or realistic portraits, which was partially satisfied by other, machines for
recording human likenesses. Beginning in the 1790s, Jacques Charles conducted experiments in the automatic; if
permanent, recording of portrait silhouettes on photosensitive paper. Two imperatives-the need for perspectively
accurate landscape and architectural scenes and for objectively truthful portraits-created a climate for certain types and
styles of pictures that, after 1839, would be achieved easily by photography.

6. Which is most likely the topic of the paragraph following this text?
A. Portraits of inanimate objects.
B. Process of publishing portraits.
C. Photographers‘ status in society.
D. More recent photography application.
E. Photography as an exclusive form of art.

7. What is the main idea of paragraph 1?


A. Aristotle discovered the wonder of light waves.
B. A lens projected images accurately onto the camera film.
C. Alhazen and Francis Bacon were the dark room inventors.
D. Camera obscura used to be enormous before it was reduced in size.
E. The initial idea of photography came up long before it was publicized.

8. Which of the following can be inferred from the text?


A. Camera sizes became small due to the fashion at that time.
B. Before the mid-18th century the photochemical processes were unstable.
C. The upper-class was the common portrait object in the Renaissance era.
D. It took some decades before society was aware of the art of photography.
E. Camera was the only tool to record human likeness in the mid-18th century.

SMA Alfa Centauri Bandung Halaman 203


MODUL PERSIAPAN UN & SBMPTN BAHASA INGGRIS 2017/ 2018

9. How is the information in the text presented? 10. The word ‗he‘ in paragraph 2 refers to ....
A. The advantages of photography are described. A. Aristotle
B. Characteristics of photographers are outlined. B. Francis Bacon
C. Types of photography objects are contrasted. C. Carl W Scheele
D. Developments of photography are explained. D. Johann Schulze
E. The styles of photographers are compared. E. Jacques Charles

Questions 11 to 15 are based on the following text.

Paper is different from other waste products because it comes from trees. Unlike plastic and metal waste
products, which cannot disintegrate, paper is biodegradable, it can be broken down into substances by the action of
bacteria, and are not harmful to the environment. While 45 out of every 100 tones of wood fiber used to make paper in
Australia comes from waste paper, the rest comes from virgin fiber from forests and plantations. By the world standard
this is a good performance since the world-wide average is 33 percent waste paper. Governments have encouraged
waste paper collection and sorting schemes and at the same time, the paper industry has responded by developing new
recycling technologies that have paved the way for even greater utilization of the use fiber. As a result, industry‘s use of
recycled fibers is expected to increase at twice the rate of virgin fiber over the coming years.
Already, waste paper constitutes 70% of paper used for packing and advances in the technology required to
remove ink from the paper have allowed a higher recycled content in newsprint and writing paper. To achieve the
benefits of recycling, the community must also contribute. We need to accept a change in the quality of paper products,
for example, stationary may be less white and rougher texture. There also needs to be support from community for
waste paper collection programs. Not only we do need to make the paper available to collectors but it also needs to be
separated into different types and sorted from contaminants such as staples, paperclips, string, and other miscellaneous
items.
There are technical limitations to the amount of paper which can be recycled and some paper products
cannot be collected for re-use. These include paper in the form of books and pernanent records, photographic paper
and paper which is badly contaminated. The four must common sources of paper for recycling are factories and retail
stores which gather large amounts of packaging material in which goods are delivered, also offices which have
unwanted business documents and computer output, paper converters and printer and lastly households which discard
newspaper and packaging material. The paper manufacturer pays a price for the paper and may also have to pay the
collection cost.

11. Governments have encouraged waste paper A. moreover


collection and sorting schemes and at the same B. furthermore
time, the paper indsutry has responded by C. therefore
developing new recycling technologies. The D. however
underlined word means .... E. due to
A. given support
B. ignored attempt 13. The following statements about recycled paper
C. admitted progress are true, except that ....
D. obtained additional A. it is mostly used for packaging
E. collaborated works B. it comes from used paper
C. it helps preserve forests
12. As a result, industry‘s use of recycled fibers is D. it has a smooth surface
expected to increase at twice the rate of virgin E. it is less white
fiber over the coming years. The underlined
conjunction can also be replaced by ....

SMA Alfa Centauri Bandung Halaman 204


MODUL PERSIAPAN UN & SBMPTN BAHASA INGGRIS 2017/ 2018

14. Recycling waste paper has been encouraged by 17. ―Did you get the cheap second hand computer
governments because .... advertised in the newspaper?‖―No luck. It ... when
A. the community is not paid for the collection of I called them‖.
waste paper A. is sold
B. new technology can produce perfectly white B. has sold
paper from it C. had sold
C. it can pave the way for greater utilization of D. is being sold
virgin fiber E. had been sold
D. it primarily enables us to conserve the existing
forest 18. ―I‘ve lost a lot of money by investing it in your
E. its production decreases the rate of friend‘s business‖.
unemployment ―I‘m sorry, you ... about his business‖.
A. ought to consult me
15. Producers of recycled paper spend a lot of B. had better consult me
money to acquire the waste paper, however, they C. would rather consult me
do not accept .... D. must have consulted me
A. paper that has been used in photography E. should have consulted me
B. packaging material for delivering goods
C. discarded business documents 19. ―What can we do to improve our employees‘
D. all contaminated waste paper skills?‖
E. collected old newspapers ―... them more training is the first what we can do‖.
A. giving
Questions 16 to 20 are not based on the text. B. to give
C. we give
16. ―Either the management or the education system D. it is giving
of our institute ... to be reviewed to increase the E. to be given
number of graduates next year‖.
―It will be evaluated immediately‖. 20. ―Impressed by the perfomance of the new
A. needs student, ....‖
B. to need ―He is also very kind and polite student‖.
C. needing A. he became chairman of the class
D. they need B. the class needed him as chairman
E. that needs C. a chairman of the class was appointed
D. the class had a new energetic chairman
E. the teacher appointed him chairman of the
class

SMA Alfa Centauri Bandung Halaman 205


MODUL PERSIAPAN UN & SBMPTN BAHASA INGGRIS 2017/ 2018

SET 3
Choose the correct answer.
Questions 1 to 5 are based on the following text.

Population growth occurs because there are currently three births for every death. In the past, the crude birth
rates were only slightly higher than death rates, but with improvements in medicine and economic growth, the death rate
fell more than birth rates. Much of the world‘s population growth is occuring in less – defeloped countries, which are
unable to support such growth. The causal effect between poverty and population growth can be looked at in two ways.
First, population growth causes poverty as the limited resources are depleted and there are too many people for the
available goods, resulting in poverty. The other view is that poverty causes high population growth because lack of
education, lack of health care, and lack of a reasonable standard of living cause high population growth. Also, parents
believe that having many children will ensure that several of them will survive to take care of them in their old age.
Cultures in which children are a form of security encourage high population growth, as children reach reproductive age
and have large families.
Much of the growth is happening disporpotionately in urban areas. Cities are environmentally harmful
because they import many resources for the people that live there, and they export their wastes. They also have an
impact on the local and regional meteorology and are centers for social problems such as crime, homelessness, and
unemployment to name a few. Cities do offer many amenities such as cultural opportunities, jobs, and education. There
are some good environmental aspects to cities, such as the promotion of efficiency in transportation, housing, utilities,
the provision of necessary goods and services, and the accomodation of large number of people within a relatively small
space. The biggest problem with cities is that people want the benefits of a city while still living in the country. This leads
to urban sprawl and suburbia.

1. The writer organizes the ideas in the text by .... E. Large families are tolerated in less-developed
A. showing the causes of population growth and countries due to their predominant belief.
their effect
B. defining population growth followed with 3. The purpose of the text is to ....
several examples A. show negative impacts of population growth
C. arguing against population growth, followed B. describe factors contributing to poverty in
with a case in urban places cities
D. comparing effects of population growth in C. analyze causes of economic growth to
countries of different development population
E. describing a historical account of population D. compare population growth in the past and the
growth in the past and the present present
E. discuss population problems in urban areas of
2. Which of the following is true according to the developing countries
text?
A. Economic growth gives little contribution to 4. The pronoun ‗they‘ in paragraph 2 refers to ....
population growth. A. areas
B. In the past there were three births for every B. cities
death in less-developed countries. C. people
C. Parents now tend to have few children in less- D. families
developed and developed countries. E. resources
D. High population growth results in a problem
where cities cannot provide enough jobs.

SMA Alfa Centauri Bandung Halaman 206


MODUL PERSIAPAN UN & SBMPTN BAHASA INGGRIS 2017/ 2018

5. It can be inferred from the text that .... C. the more children parents have, the better
A. life quality of cities is worse than that of their economy will be
suburbans D. the less developed a country is the worse its
B. poverty in a country is linked to its rapid birth economic growth is
rates E. less developed countries tend to show a
higher rate of population growth

Questions 6 to 10 are based on the following text.

Curitiba in southeastern Brazil has earned an international reputation for good city management with its
innovative programs for public transport, industrial pollution control and waste recycling. As one of Brazil‘s fastest
growing cities, it has avoided many of the problems that usually go with rapid expression. Early in its development,
Curitiba adopted a plan for linier growth along radial axes, using the areas in between for green space and leisure
activities as well as for industrial and housing development.
Curitiba has few traffic jams, despite havening more cars per capita than any other Brazilian city except
Brasilia. Roads running along the structural axes include special ‗busways‘, which provide rapid transport of people to
and from the city center. A sophisticated bus system has been developed, featuring red express buses, green inter
district buses and yellow ‗feeder‘ buses. There are regular services, which are closely linked, so that it is easy and quick
to switch from one route to another. A single fare operates for all journeys within the city limits, which tickets
interchangeable on all routes.
The transport network is managed by a city authority which lays down operating rides, set timetables and
monitor performance. The buses themselves are run by private companies, licensed by the city authority. The bus
system in Curitiba which was chosen to be focused on by the authority is far cheaper in terms of capital cost than
underground metro or light rail. It is a simple transparent system and it works some 75% of commuters travel by bus (In
Sidney, by contrast, over 60% go to work by car, in Los Angeles 90%.
Another innovation has been the preservation of green space. During the last 20 years, green space per
capita has increased one-hundred fold, which is all the more amazing as this increase took place during the period of
rapid population growth.
A key factor in Curitiba‘s civic development has been its mayors, whose enthusiasm and persistence have
been maintained for over 20 years. But as they have always stresses, it is ultimately the people themselves to whom the
city belongs.

6. In spite of having more cars than other Brazilian 8. Who has succeeded in making Curitiba a city with
cities, Curitiba can greatly reduce in traffic good management?
problems due to .... A. A city authority.
A. the availability of special buses B. Private companies.
B. the regular services of public transport C. Brazilian commuters.
C. the provision of rapid transport for people D. Transport network manager.
D. the well-developed and organized bus system E. The mayors and the people of the city.
E. the convenient switch from one bus to another
9. The following innovations make Curitiba one of
7. Curitiba is internationally known for its .... the Brazil‘s fastest growing cities, except ....
A. successful programs for city management A. launching waste recycling programs
B. fast growing population and rapid expansion B. creating green space and leisure facilities
C. efforts to create non-polluting public transport C. establishing a sophisticated transport system
D. creation of green space amidst industrial area D. building the mayor‘s enthusiasm and
E. innovative programs to avoid population persistence
growth

SMA Alfa Centauri Bandung Halaman 207


MODUL PERSIAPAN UN & SBMPTN BAHASA INGGRIS 2017/ 2018

E. finding solutions to problems of rapid city B. buses are operated by private companies
expansion C. the operation of buses is much easier to
monitor
10. Curitiba prefers concentrating on the use of the D. buses are owned and managed by the city
buses for its public transport to any other kind of government
transport because .... E. commuters find the fee of other public
A. buses require much cheaper capital cost transport expensive

Questions 11 – 15 are based on the following text.

The option of home schooling has been around for a long time; (11) ... until recently it had not been so
popular. The idea of home schooling seems like a cure-all to many parents due to the advantages this type of education
provides over traditional schools. Children who are home schooled can (12) ... many of the problems schools have
become known for. For one, the environment is less threatening. Children can learn without (13) ... other students,
aggresive or nasty teachers, and be under the constant supervision of parents. (14) ... , home schooling allows parents
to dictate the academic course of their children. Home schooling also allows students to proceed at their own speed. If a
child is weak at (15) ..., a parent can focus lessons on this skill in favor of another skill that the child might grasp rather
easily.

11. .... 15. ....


A. however A. multiple
B. whereas B. multiply
C. in addition C. multiplied
D. even though D. multiplying
E. in other words E. multiplication

12. .... Questions 16 to 20 are not based on the text.


A. break
B. avoid 16. ―Everybody believes that either the members or
C. ignore the chairperson of the parliament ... the matter
D. pursue seriously‖.
E. encounter ―They want to find out the solution of their
problem‖.
13. .... A. discuss
A. feared B. is discussing
B. fear of C. are discussing
C. fearing D. have discussed
D. scaring E. have been discussing
E. afraid of

14. ....
A. So
B. Thus
C. However
D. Therefore
E. In addition

SMA Alfa Centauri Bandung Halaman 208


MODUL PERSIAPAN UN & SBMPTN BAHASA INGGRIS 2017/ 2018

17. ―The workers are demanding not only a raise in salary but also ....‖
―They don‘t feel satisfied with the condition right now‖.
A. a longer annual leave
B. to have a longer annual leave
C. in getting a longer annual leave
D. they want a longer annual leave
E. a longer annual leave they asked

18. ―When did you realize you had lost handphone?‖


―When I ... my friend to submit my English paper on time‖.
A. called
B. am calling
C. had called
D. was calling
E. have called

19. ―Could you please book me on the next flight to Mexico city?‖
―I am sorry, Madam. Our airline ... to Mexico city‖.
A. will not fly
B. did not fly
C. does not fly
D. was not able to fly
E. has not been flying

20. ―Since when hasn‘t she been feeling well?‖


―Since she ... from her long trip‖.
A. returned
B. had returned
C. has returned
D. was returning
E. has been returning

SMA Alfa Centauri Bandung Halaman 209


MODUL PERSIAPAN UN & SBMPTN BAHASA INGGRIS 2017/ 2018

SET 4
Choose the correct answer.
Questions 1 to 4 are based on the following text.

Text A
Advocates for virtual education say that it has the power to transform an archaic K-12 system of schooling in
the primary and secondary levels. Instead of blackboards, school houses, and a six-hour school day, interactive
technology will personalize learning to meet each student‘s needs, ensure all students have access to quality teaching,
extend learning opportunities to all hours of the day and all days of the week, and innovate and improve over time.
Indeed, virtual education has the potential not only to help solve many of the most pressing issues in K-12
education, but to do so in a cost-effective manner. More than 1 million public-education students now take online
courses, and as more districts and states initiate and expand online referrings, the numbers continue to grow. But to
date, there is little research or publicity available data on the outcomes from K-12 online learning. And even when data
are publicly available, as is the case with virtual charter schools, analysts and education officials have paid limited
attention to – and have few tools for analyzing – performance.

Text B
Virtual education is in a period of rapid growth, as school districts, for profit providers, and nonprovit start-ups
all move into the online learning world. But without rigorous oversight, a thousand flowers blooming will also yield a lot of
weeds.
Real accountability, including the means to identify and end ineffective practices and programs, must be
constantly balanced with the time required to refine new, immature technologies and approaches to learning. Both virtual
education advocates and education policymakers should learn from nearly two decades of experience with charter
schooling, another reform movement predicated on innovation and change within public education. After nearly 20 years
of practice, the charter school movement provides important lessons on how to ensure the improved student outcomes
remain the top priority.

1. The theme of these two texts would most likely be E. the effectiveness of the learning mode is not
about .... yet revealed
A. virtual education
B. modern schooling 3. With regard to online education in the text, it can
C. alternative learning be hypothetically predicted that ....
D. innovative education A. learning will be more difficult but joyful
E. technology-based learning B. students‘ learning will be more optimum
C. students can have classes at their homes
2. The ideas in the following sentences reflects D. more students can learn at their own pace
factual information in either text, except .... E. teachers‘ role will be replaced by machines
A. this kind of education has been applied to K-
12 levels 4. The two texts above similarly deal with these
B. the education discussed reflects individualized points about online education, except ....
learning A. virtual charter schools
C. online learning has reached different levels of B. teachers‘ responsibilites
education C. quality teaching practices
D. an innovative school provides an excellent D. evaluation of its effectiveness
model to learn E. increasing trends of application

SMA Alfa Centauri Bandung Halaman 210


MODUL PERSIAPAN UN & SBMPTN BAHASA INGGRIS 2017/ 2018

Questions 5 to 8 are based on the following text.

Tetris, one of the world‘s most popular games is thirty years old. Soviet scientist, Alexey Pajitnov, invented it in
1984. A few years later, it made its way to the west where it was played on the legendary Commodore 64, the first Apple
computers and the IBM-PC. Within a short time, it became popular around the world. In 1989 it was released to the
Gameboy console. Tetris is the best-selling video game of all times, with 150 million copies sold since its first release.
Tetris is very easy to play. The aim is to stack differently colored and shaped blocks that fall down into a pit.
You can rotate them in every direction and you get points when you have created a complete row of blocks, which then
disappears. The goal of the game is to make horizontal lines full of blocks without any gaps between them.
Pajitnov created the game as a welcome distraction from everyday work, but Soviet authorities quickly banned
the Tetris. Unlike many other software developers, the Tetris did not make the Soviet scientist rich. Private business was
banned during the Communist era and Pajitnov had to give up the rights to the government.
Tetris is still popular today. Even though its educational value is close to zero, people still keep coming back to
play. Game developer Ubisoft has announced that it is releasing an ultimate version of Tetris in honour of the game‘s
30th birthday.
Adapted from http://www.english-online.at/new-articles/entertainment/tetris-world-most-popular-video-game.htm

5. The best title of the text above is .... 7. According to the text, the Tetris did not make the
A. Pajitnov and Tetris creator rich because ....
B. The History of Tetris A. the game is too easy to play
C. Tetris‘ Thirtieth Birthday B. it was pirated by American company
D. The Best Selling Video Game C. the Soviet authorities forbade the game
E. The Most Popular Game Ever D. it was owned by the Soviet government
E. no private business was allowed in Soviet
6. This kind of text is usually found in a ....
A. PC magazine 8. All of the following statements are true about
B. scientific report Tetris, except ....
C. computer manual A. it has an educational value
D. video game brochure B. it is the world‘s most famous game
E. classified advertisement C. it was created by a Soviet scientist
D. it is always a best-selling video game
E. Pajitnov does not own the game any longer

Questions 9 to 11 are based on the following text.

Over this decade, employment in jobs requiring education beyond a high school diploma will grow more rapidly
than employment in jobs that do not; of the 30 fastest growing occupations, more than half require post – secondary
education. With the average earnings of college graduates at a level that is twice as high as that of workers with only a
high school diploma, higher education is now the clearest (9) ... into the middle class.
In higher education, the U.S has been outpaced internationally. While the United States ranks ninth in the world
in the proportion of young adults enrolled in college, we have fallen to 16th in the world in our share of certificates and
degrees awarded to adults ages 25 – 34 – lagging behind Korea, Canada, Japan and other nations. While more than
half of college students graduate within six years, the (10) ... for low-income students is around 25 percent.
Acknowledging these factors early in his administration, President Obama challenged every American to
commit to at least one year of higher education or post – secondary training. (11) ... that by 2020, America would once
again have the highest proportion of college graduates in the world.

SMA Alfa Centauri Bandung Halaman 211


MODUL PERSIAPAN UN & SBMPTN BAHASA INGGRIS 2017/ 2018

9. ....
A. advantage 11. ....
B. endeavor A. American people will deserve higher education
C. opening for their future
D. direction B. The president has also set up an innovative
E. pathway goal for the country
C. President Obama has reminded Americans of
10. .... their college completion
A. completion rate D. Middle class people of America are
B. academic potential encouraged to go to American colleges
C. learning achievement E. American students and workers became
D. academic absorption encouraged to take further studies
E. logical understanding

Questions 12 to 15 are based on the following text.

Trafficking in human beings is a multi – billion – dollar form of international organized crime, constituting
modern day slavery. Victims are recruited and trafficked between countries and regions using deception or coercion.
They are stripped of their autonomy, freedom of movement and choice, and face various forms of physical and mental
abuse.
There are three main types of human trafficking: trafficking for forced labor, trafficking for sexual exploitation,
trafficking of organs. Closely connected is the issue of people smuggling in which smugglers procure, for financial or
material gain, the illegal entry of an individual into a country of which he is neither a citizen nor a permanent resident.
Generally speaking, once payment is completed, the relationship between the illegal immigrant and the smuggler is
terminated.
Trafficking in human beings is a crime under international law and many national and regional legal sysems.
Given the complexities of the issue, a multitude of strategies are necessary at a range of levels in order to reduce the
problem.

12. Why is trafficking human beings also called 13. ... the illegal entry of an individual into a country of
modern day slavery? It is because .... which he is neither ... (paragraph 2). The
A. The victims are stripped of their autonomy, underlined word means ....
freedom of movement and choice, and face A. in opposition to individual‘s right
various forms of physical and mental abuse B. in accordance with the law
B. The victims are recruited and trafficked C. approval the constitution
between countries and regions using D. in harmony with the law
deception or coercion E. against the law
C. Trafficking in human beings is a crime under
international law and many national and 14. Accodring to the text, the relationship between the
regional legal systems illegal immigrant and the smuggler ....
D. Once payment is completed, the relationship A. is maintained although the payment is
between the illegal immigrant and the completed
smuggler is terminated B. continues even after the payment is completed
E. The victims are never identified rightly by the C. stops as soon as the payment is completed
legal institution D. is protected by the official institution
E. is prolonged for further smuggling

SMA Alfa Centauri Bandung Halaman 212


MODUL PERSIAPAN UN & SBMPTN BAHASA INGGRIS 2017/ 2018

15. They are stripped of their autonomy, freedom of 19. ―Problems of transportation have caused donated
movement and choice, and face various forms of food and clothing for the tsunami victims to be
physical and mental abuse. The underlined refers piled up at the airport‖.
to .... ―I wish ....‖
A. the countries involved in the human beings A. there are more volunteers to help
trafficking B. the victims should not have worried
B. the regions involved in the human beings C. the government can ask for help
trafficking D. the problem will be solved immediately
C. the victims of the human beings trafficking E. there were more helicopters available
D. international illegal organized crimes
E. international organized crimes 20. Many of risk factors ... with pregnancy are more
harmful when the expectant mother is over 35.
Questions 16 to 20 are not based on the text. A. associate
B. associated
16. Right now, the amount of carbon dioxide in our C. associating
environment ... at 400 parts per million, a number D. to associate
that is already large enough to start affecting our E. to be associating
environment.
A. hovers
B. hovered
C. is hovering
D. have hovered
E. has been hovering

17. Several large vertebrates ... by hunting, illegal


trade and habitat loss.
A. have been threatened
B. are being threaten
C. are threatened
D. were threaten
E. is threatened

18. Many climate scientists agree that significant


societal, economic, and ecological damage ... if
global temperatures rose by more than 2˚C (36˚F)
in such a short time.
A. would have been result
B. would result
C. were result
D. was result
E. will result

SMA Alfa Centauri Bandung Halaman 213

You might also like